NCERT Solutions For Class 8 Maths

NCERT Solutions For Class 8 Maths Chapter 11 Direct And Inverse Proportions

NCERT Class 8 Maths Chapter 11 Direct And Inverse Proportions Introduction

Question 1. Mohan prepares tea for himself and his sister. He uses 300 mL of water, 2 spoons of sugar, 1 spoon of tea leaves and, 50 mL of milk. How much quantity of each item will he need, if he has to make tea for five persons?
Solution:

NCERT Solutions For Class 8 Maths Chapter 10 Direct And Inverse Proportions Mohan Prepares Tea For Himself

Quantity of water required for 2 persons = 300 mL

Quantity of water required for 1 person = \(\frac{300}{2} \mathrm{~mL}\)

Quantity of water required for 5 persons \(\frac{300}{2} \times 5 \mathrm{~mL}\)

Quantity of sugar required for 2 persons = 2 spoons

Quantity of sugar required for 1 person \(\frac{2}{2} \text { spoons }\)

Read and Learn More NCERT Solutions For Class 8 Maths

Quantity of sugar required for 5 persons \(\frac{2}{2} \times 5=5 \text { spoons }\)

Quantity of tea leaves required for 2 persons = 1 spoon

Quantity of tea leaves required for 1 person \(\frac{1}{2} \text { spoons }\)

Quantity of tea leaves required for 5 persons \(=\frac{1}{2} \times 5=2 \frac{1}{2} \text { spoons }\)

Quantity of milk required for 2 persons = 50 mL

Quantity of milk required for 1 person = \(\frac{50}{2} \mathrm{~mL}\)

Quantity of milk required for 5 persons = \(\frac{50}{2} \times 5 \mathrm{~mL}=125 \mathrm{~mL}\)

Thus, Mohan will require 750 mL of water, 5 spoons of sugar, \(2 \frac{1}{2}\) Spoons of tea leaves, and 125 mL of milk for 5 persons.

Question 2. If two students take 20 minutes to arrange chairs for an assembly, then how much time would five students have to do the same job?
Solution:

Time taken by 2 students = 20 minutes

Time taken by 1 student = 20 x 2 minutes

Time taken by 5 students = \(\frac{20 \times 2}{5} \text { minutes }=8 \text { minutes }\)

We come across many such situations in our day-to-day life, where we need to see variation in one quantity bringing in variation in the other quantity. Quantities so related are called variables and the relation is called variation.

For example :

  1. If the number of articles purchased increases, the total cost also increases.
  2. The more money deposited in a bank, more is the interest earned.
  3. As the speed of a vehicle increases, the time taken to cover the same distance decreases.
  4. For a given job, the more the number of workers less will be the less time taken to complete the work.

Question 3. Write five more such situations where a change in one quantity leads to  change in another quantity.
Solution:

Five more such situations where a change in one quantity leads to a change in another quantity are as follows :

  1. If the amount of the commodity purchased increases, the total cost also increases.
  2. If the amount of loan taken from a bank increases, then the interest to be paid also increases.
  3. At a particular temperature, if the pressure of gas increases, then the volume decreases.
  4. If the number of pipes to fill a tank increases, the time taken to fill the tank decreases.
  5. If the number of guests increases, the number of days for which a given amount of food will last decreases.

Question 4. How do we find out the quantity of each item needed by Mohan? or, the time five students take to complete the job?
Solution:

We need to study some concepts of variation as follows :

  1. Direct variation
  2. Inverse variation

Two quantities may be linked in two ways :

Both increase or decrease together proportionally.

If one increases, the other decreases proportionally, and vice-versa.

The first way is named a direct variation whereas the second way is named as an inverse variation.

  1. Direct Proportion

If two quantities are related in such a way that an increase in one leads to a corresponding proportional increase in the other, then such a variation is called direct variation.

Thus, two numbers x and y are said to vary in direct proportion if

\(\frac{x}{y}\)= k or x = ky where k is a constant.

In this case, if ÿ1,y2 are the values ofy corresponding to the values x1, x2 of x respectively, then

⇒  \(\frac{x_1}{y_1}=\frac{x_2}{y_2}\)

Question 5. Let the consumption of petrol be x liters and the corresponding distance traveled by km. Now, complete, the following table:

NCERT Solutions For Class 8 Maths Chapter 10 Direct And Inverse Proportions Distance Travelled Be y km

We find that as the value increases, the value also increases in such a way that the ratio \(\frac{x}{y}\) does not change; it remains constant (say It). In this case, it is……(check it !)
Solution:

NCERT Solutions For Class 8 Maths Chapter 10 Direct And Inverse Proportions Distance Travelled Be y km Direct Proportion

In this case, it is \(\frac{1}{15}\)

Question 6. Think of a few more examples for direct proportion.
Solution:

  1. A few more examples of direct proportion are as follows :
  2. Length of the cloth purchased and its total cost.
  3. Number of months and total salary.
  4. Number of hours of production and the amount of the commodity produced.

Take a clock and fix its minute hand, at 12.

Record the angle turned through by the minute hand from its original position and the time that has passed, in the following table

NCERT Solutions For Class 8 Maths Chapter 10 Direct And Inverse Proportions Clock And Fix Minute

What do you observe about T and A ? Do they increase together? Is \(\frac{T}{A}\) same every time ?

Is the angle turned through by the minute hand directly proportional to the time that has passed? Yes; From the above table, you can also see

NCERT Solutions For Class 8 Maths Chapter 10 Direct And Inverse Proportions Hand Directly Proportional

T1 : T2 = A1 : A2, because

T1 : T2 = 15 : 30 = 1:2

A1 : A2 = 90 : 180 = 1:2

Check if T2 : T3 = A2 : A3 and T3 : T4 = A3 : A4

You can repeat this activity by choosing your time interval.

NCERT Solutions For Class 8 Maths Chapter 10 Direct And Inverse Proportions The Minute Hand Is Directly Proportional

We observe about T and A that they increase together and \(\frac{T}{A}\)is the same every time. A Yes, the angle turned by the minute hand is directly proportional to the time that has passed.

On checking, we find that

T2 : T3 = A2 : A3 = 2:3

T3 : T4 = A3 : A4 = 3:4

Ask your friend to fill in the following table and find the ratio of his age to the corresponding age of his mother:

NCERT Solutions For Class 8 Maths Chapter 10 Direct And Inverse Proportions The Ratio Of His Age Of His Friends

What do you observe?

Do F and M increase (or decrease) together? Is — the same every time? No.

You can repeat this activity with other friends and write down your observations.

Solution:

NCERT Solutions For Class 8 Maths Chapter 10 Direct And Inverse Proportions Repeat This Activity With Other Friends

We observe that F and M increase (or decrease) together. But — is not the same every
time.

Note: It is not. necessary that the variables increasing (or decreasing) together are always in direct proportion.

For example :

Height and age of an individual

Height and weight of an individual

Height of mango tree and the number of mango fruits growing on its branches.

Question 7. Observe the following tables and find if x and y are directly proportional:

NCERT Solutions For Class 8 Maths Chapter 10 Direct And Inverse Proportions

Solution:

We have \(\frac{20}{40}=\frac{1}{2}\)

⇒ \(\frac{17}{34}=\frac{1}{2}\)

⇒ \(\frac{14}{28}=\frac{1}{2}\)

⇒ \(\frac{11}{22}=\frac{1}{2}\)

⇒ \(\frac{8}{16}=\frac{1}{2}\)

⇒ \(\frac{5}{10}=\frac{1}{2}\)

⇒ \(\frac{2}{4}=\frac{1}{2}\)

We find that each ratio is the same.

Hence, x and y are directly proportional.

We have \(\frac{6}{4}=\frac{3}{2}\)

⇒ \(\frac{10}{8}=\frac{5}{4}\)

⇒ \(\frac{14}{12}=\frac{7}{6}\)

⇒ \(\frac{18}{16}=\frac{9}{8}\)

⇒ \(\frac{22}{20}=\frac{11}{10}\)

⇒ \(\frac{26}{24}=\frac{13}{12}\)

⇒ \(\frac{30}{28}=\frac{15}{14}\)

We find that all the ratios are not the Same.

Hence, x and y are not directly proportional.

⇒ \(\frac{5}{15}=\frac{1}{3}\)

⇒ \(\frac{8}{24}=\frac{1}{3}\)

⇒ \(\frac{12}{36}=\frac{1}{3}\)

⇒ \(\frac{15}{60}=\frac{1}{4}\)

⇒ \(\frac{18}{72}=\frac{1}{4}\)

⇒ \(\frac{20}{100}=\frac{1}{5}\)

We find that all the ratios are not the same.

Hence, x and y are not directly proportional.

Question 8. Principal = f1,000, Rate = 8% per annum. Fill in the following table and find which type of interest (simple or compound) changes in direct proportion with time.

NCERT Solutions For Class 8 Maths Chapter 10 Direct And Inverse Proportions Changes In Direct Proportion With Time Period

Solution:

1 year \(\frac{\mathrm{P} \times r \times t}{100}\)

⇒ \(₹ \frac{1000 \times 8 \times 1}{100}\)

₹ 80

We have \(\frac{80}{1}=80\)

2 years \(\frac{\mathrm{P} \times r \times t}{100}\)

⇒ \(₹ \frac{1000 \times 8 \times 2}{100}\)

₹ 160

\(\frac{160}{2}=80,\)

3 years \(\frac{\mathrm{P} \times r \times t}{100}\)

⇒\(₹ \frac{1000 \times 8 \times 3}{100}\)

₹ 240

⇒ \(\frac{240}{3}=80\)

We find that each ratio is the same.

Hence, simple interest changes in direct proportion with a period.

Compound Interest (in ?) for different periods

1 year \(\mathrm{P}\left(1+\frac{r}{100}\right)^t-\mathrm{P}\)

⇒ \(1000\left(1+\frac{8}{100}\right)-1000\)

⇒ \(1080-1000=₹ 80\)

We have \(\frac{80}{1}=80\)

2 years \(\mathrm{P}\left(1+\frac{r}{100}\right)^t-\mathrm{P}\)

v\(1000\left(1+\frac{8}{100}\right)^2-1000\)

₹ 166.40

⇒ \(\frac{166.40}{2}=83.20\)

3 years \(\mathrm{P}\left(1+\frac{r}{100}\right)^t-\mathrm{P}\)

⇒ \(1000\left(1+\frac{8}{100}\right)^3-1000\)

₹ 259.712

⇒ \(\frac{259.712}{3}=86.5706\)

We find that all the ratios are not the same.

Hence, compound interest does not change in direct proportion to the period.

Question 9. If we fix the period and the rate of interest, simple interest changes proportionally with the principal. Would there be a similar relationship for compound interest? Why?
Solution:

Case 1

⇒ \(\text { S.I. }=\frac{\mathrm{P} r t}{100}\)

⇒ \(\frac{\text { S.I. }}{\mathrm{P}}=\frac{r t}{100}\)

r and t are fixed

⇒ \(\frac{r t}{100} \text { is constant }\)

⇒ \(\frac{\mathrm{S} . \mathrm{I}}{\mathrm{P}}=\text { constant }\)

S.I. changes proportionally with principal

Case 2

⇒ \(\text { C.I. }=\mathrm{P}\left\{\left(1+\frac{r}{100}\right)^t-1\right\}\)

⇒ \(\frac{\mathrm{C} . \mathrm{I}}{\mathrm{P}}=\left(1+\frac{r}{100}\right)^t-1\)

r and tare fixed

⇒ \(\left(1+\frac{r}{100}\right)^t-1 \text { is constant }\)

⇒ \(\frac{\mathrm{C} . \mathrm{I}}{\mathrm{P}}=\text { constant }\)

C.I. changes proportionally with principal

There is a similar relationship for compound interest

Question 10. Take a map of your State. Note the scale used there. Using a ruler, measure the “map distance’’ between any two cities. Calculate the actual distance between them.
Solution:

Let the scale be 1 cm = 100 km

Let the distance between any two cities on the map be 5 cm.

Then, the actual distance between these two cities = 100 x 5 km = 500 km

Direct And Inverse Proportions Exercise 11.1

Question 1. Following are the car parking charges near a railway station up to

  1. 4 hours ₹ 60
  2. 8 hours ₹ 100
  3. 12 hours ₹ 140
  4. 24 hours ₹ 180

Check if the parking charges are in direct proportion to the parking time.
Solution:

We have

⇒ \(\frac{60}{4}=\frac{60 \div 4}{4 \div 4}=\frac{15}{1}\) HCF (60, 4) = 4

⇒ \(\frac{100}{8}=\frac{100 \div 4}{8 \div 4}=\frac{25}{2}\) HCF (100, 8) = 4

⇒ \(\frac{140}{12}=\frac{140 \div 4}{12 \div 4}=\frac{35}{3}\) HCF (140, 12) = 4

⇒ \(\frac{180}{24}=\frac{180 \div 12}{24 \div 12}=\frac{15}{2}\) HCF (180, 24) = 12

Since all the ratios are not the same, therefore, the parking charges are not in direct proportion to the parking time.

Question 2. A mixture of paint is prepared by mixing 1 part of red pigments with 8 parts of the base. In the following table, find the parts of the base that need to be added

NCERT Solutions For Class 8 Maths Chapter 10 Direct And Inverse Proportions A Mixture Of Paint Is Prepared

Solution:

Let the number of parts of red pigments be x and the number of parts of the base is y.

As the number of parts of red pigments increases, several parts of the base also increase in the same ratio. So it is a case of direct proportion.

We make use of the relation of the type \(\frac{x_1}{y_1}=\frac{x_2}{y_2}\)

Here

x1= 1

y1 = 8 and x2 = 4

Therefore \(\frac{x_1}{y_1}=\frac{x_2}{y_2} \text { gives }\)

⇒ \(\frac{1}{8}=\frac{4}{y_2}\)

y2 = 8 x 4

y2 = 32

Hence, 32 parts of the base are needed to be added to 4 parts of red pigments.

Here,

x1 = 1

y4 = 8 and x3 = 7

Therefore \(\frac{x_1}{y_1}=\frac{x_3}{y_3}\) gives

⇒ \(\frac{1}{8}=\frac{7}{y_3}\)

y3 = 8 x 7

y3 = 56

Hence, 56 parts of the base are needed to be added to 7 parts of red pigments.

Here

x1= 1

y1= 8

x4 = 12

Therefore,\(\frac{x_1}{y_1}=\frac{x_4}{y_4}\) gives

⇒ \(\frac{1}{8}=\frac{12}{y_4}\)

y4= 12 x 8

y4 = 96

Hence, 96 parts of the base are needed to be added to 12 parts of red pigments.

Here,

x1= 1

y1 = 8

x5 = 20

Therefore \(\frac{x_1}{y_1}=\frac{x_5}{y_5}\)gives

⇒ \(\frac{1}{8}=\frac{20}{y_5}\)

y5 = 8 x 20

y5 = 160

Hence, 160 parts of the base are needed to be added to 20 parts of red pigments.

Question 3. In Question 2 above, if part of the red pigment requires 75 mL of the base, how much red pigment should we mix with 1800 mL of the base?
Solution:

Let the number of parts of red pigments be and the amount of base be y mL.

As the number of parts of red pigment increases, the amount of base also increases in the same ratio. So it is a case of direct proportion.

We make use of the relation of the type.

⇒ \(\frac{x_1}{y_1}=\frac{x_2}{y_2}\)

Here x1=1

y2 = 75 and ,y2 = 1800

⇒ \(\frac{x_1}{y_1}=\frac{x_2}{y_2}\) give

⇒ \(\frac{1}{75}=\frac{x_2}{1800}\)

⇒ \(\frac{1}{75} =\frac{x_2}{1800}\)

⇒ \(75 x_2 =1800 \)

⇒ \(x_2 =\frac{1800}{75}=\frac{600}{25}\)

x2 = 24

Hence, 24 parts of the red pigment should be mixed with 1800 mL of base.

⇒ \(\frac{x_1}{y_1}=\frac{x_2}{y_2} \Rightarrow x_1 y_2=x_2 y_1\)

⇒ \(\frac{x_1}{x_2}=\frac{y_1}{y_2}\)

Question 4. A machine in a soft drink factory fills 840 bottles in six hours. How many bottles will it fill in five hours?
Solution:

Let the machine fill x bottles in five hours. We put the given information in the form of a table as shown below :

NCERT Solutions For Class 8 Maths Chapter 10 Direct And Inverse Proportions A Machine In A Soft Drink Factory

The more the number of hours, the more the y2 number of bottles would be filled. So, the number of bottles filled and the number of hours are directly proportional to each other.

⇒ \(\frac{x_1}{x_2}=\frac{y_1}{y_2}\)

⇒ \(\frac{840}{x_2}=\frac{6}{5}\)

6×2 = 840 x 5

⇒ \(x_2=\frac{840 \times 5}{6}=700\)

Hence, 700 bottles will be filled in five hours.

Question 5. A photograph of bacteria enlarged 50,000 limes allies a length of 5 cm as shown in the diagram. What is the actual length of the bacteria? If the photograph is enlarged 20,000 times only, what, would be its enlarged length?

NCERT Solutions For Class 8 Maths Chapter 10 Direct And Inverse Proportions A Photograph Of A Bacteria Enlarged Actual Length

Solution:

Actual length of the bacteria \(\frac{5}{50000} \mathrm{~cm}=\frac{1}{10000} \mathrm{~cm}=10^{-4} \mathrm{~cm}\)

Let the enlarged length be cm. We put the given information in the form of a table as shown below :

NCERT Solutions For Class 8 Maths Chapter 10 Direct And Inverse Proportions A Photograph Of A Bacteria Enlarged

⇒ \(\frac{x_1}{y_1}=\frac{x_2}{y_2}\)

⇒ \(\frac{50000}{5}=\frac{20000}{y_2}\)

50000 y2 = 5 x 20000

⇒ \(y_2=\frac{5 \times 20000}{50000}=\frac{5 \times 2}{5}\)

y2 = 2

Hence, its enlarged length would be 2 cm.

Question 6. In a model of a ship, the mast is 9 cm high, while the mast of the actual ship is 12 m. high. If the length of the ship is 28 m, how long is the model ship?

NCERT Solutions For Class 8 Maths Chapter 10 Direct And Inverse Proportions A Model Of A Ship

Solution:

Let the length of the model ship be x2 cm.

We form a table as shown below :

NCERT Solutions For Class 8 Maths Chapter 10 Direct And Inverse Proportions In A Model Of A Ship

The more the length of the ship, the more would be the length of its mast. Hence, this is a case of direct proportion. That is,

⇒ \(\frac{x_1}{y_1}=\frac{x_2}{y_2}\)

⇒ \(\frac{28}{12}=\frac{x_2}{9}\)

12 x2 = 28 x 9

⇒ \(x_2=\frac{28 \times 9}{12}=\frac{28 \times 3}{4}\)

x2 = 21

Hence, the length of the model ship is 21 m

Question 7. Suppose 2 kg of sugar contains 9 Y.106 crystals. How many sugar crystals are there in (i) 5 kg of sugar? (ii) 1.2 kg ofsugar ?
Solution:

Suppose the amount of sugar is x kg and the number of crystals is y. We put the given information in the form of a table as shown below :

NCERT Solutions For Class 8 Maths Chapter 10 Direct And Inverse Proportions The Amount Of Sugar

As the amount of sugar increases, the number of crystals also increases in the same ratio. So it is an ease of direct proportion. We make use of the relation of the type \(\frac{x_1}{y_1}=\frac{x_2}{y_2}\)

Here, x1 = 2

y1 = 9 x 106

x2= 5

Therefore \(\frac{x_1}{y_1}=\frac{x_2}{y_2}\) gives

⇒ \(\frac{2}{9 \times 10^6}=\frac{5}{y_2}\)

2y2= 5 x 9 x 106

⇒ \(y_2=\frac{5 \times 9 \times 10^6}{2}\)

y2 = 22.5 x 106

y2 = 2.25 x 107

[Standard form]

Hence, there are 2.25 x 107 crystals of sugar in 5 kg of sugar

Here, .x1 = 2

y1 =9 X 106

x3 = 1.2

Therefore \(\frac{x_1}{y_1}=\frac{x_3}{y_3}\) gives

⇒ \(\frac{2}{9 \times 10^6}=\frac{1.2}{y_3}\)

2y3 = 1.2 x 9 x 106

2y3= 10.8 x 106

⇒ \(y_3=\frac{10.8 \times 10^6}{2}\)

y3 = 5.4 x 106

Hence, there are 5.4 x 106 crystals of sugar in 1.2 kg of sugar.

Question 8. Rashmi has a road map with a scale of 1 cm representing 18 km. She drives on a road for 72 km. What would be her distance covered in the map?
Solution:

Let. the distance covered in the map be x cm. We put the given information in the form of a table as shown below :

NCERT Solutions For Class 8 Maths Chapter 10 Direct And Inverse Proportions Rashmi Has A Road Map

The more the actual distance covered on the road, the more the distance on the map. So, it is a case of direct proportion

⇒ \(\frac{x_1}{y_1}=\frac{x_2}{y_2} \Rightarrow \frac{18}{1}=\frac{72}{y_2}\)

⇒ \(y_2=\frac{72}{18}=4\)

Hence, the distance covered on the map would be 4 cm.

Question 9. A 5 m 60 cm high vertical pole casts a shadow 3 m 20 cm long. Find at the same time) the length of the shadow cast by another pole 10 m 50 cm high (ii) the height of a pole which casts a shadow 5 m long.
Solution:

Let the height of the vertical pole be * m and the length of the shadow be y m. We put the given information in the form of a table as shown below :

NCERT Solutions For Class 8 Maths Chapter 10 A 5 m 60 cm High Vertical Pole Casts

As the height of the vertical pole increases, the length of the shadow also increases in the same ratio. So it is a case of direct proportion.

We make use of the relation of the type \(\frac{x_1}{y_1}=\frac{x_2}{y_2} \text {. }\)

Here xl = 5 m 60 cm = 5.60 m

yi= 3 m 20 cm = 3.20 m

x2 = 10 m 50 cm = 10.50 m

Therefore \(\frac{x_1}{y_1}=\frac{x_2}{y_2}\) gives

⇒ \(\frac{5.6}{3.2}=\frac{10.5}{y_2}\)

5.6y2 = 3.2 x 10.5

⇒ \(y_{2 .}=\frac{3.2 \times 10.5}{5.6}=\frac{32 \times 105}{560}\)

⇒ \(\frac{4 \times 105}{70}=\frac{4 \times 15}{10}\)

y2= 6

Hence, the length of the shadow is 6 m.

(ii) Here xx = 5 m 60 cm = 560 cm

y1 = 3 m 20 cm = 320 cm

y3 = 5 m = 500 cm

Therefore \( \frac{x_1}{y_1} =\frac{x_3}{y_3} \text { gives } \frac{560}{320}=\frac{x_3}{500}\)

⇒ \(320 x_3 =560 \times 500\)

⇒ \(x_3 =\frac{560 \times 500}{320}=\frac{560 \times 50}{32}\)

⇒ \(\frac{70}{4} \times 50=\frac{3500}{4}\)

x3 = 875

Hence, the height of the pole is 8 m 75 cm.

Question 10. A loaded truck travels 14 km in 25 minutes. If the speed remains the same, how far can it travel in 5 hours?
Solution:

Two quantities x and y which vary £ in direct proportion have the relation

⇒ \(x=k y \text { or } \frac{x}{y}=k\)

⇒ \(k =\frac{\text { Number of km it can travel }}{\text { time in hours }}\)

⇒ \(\frac{14}{\left(\frac{25}{60}\right)}=\frac{14 \times 60}{25}\)

⇒ \(\frac{14 \times 12}{5}=\frac{168}{5}\)

Now, x is the distance traveled in 5 hours

Using the relation x = ky, obtain

⇒ \(x=\frac{168}{5} \times 5\)

x = 168

Hence, it can travel 168 km in 5 hours.

Question 11. On a squared paper, draw five squares of different sides. Write the following information in a tabular form.

NCERT Solutions For Class 8 Maths Chapter 10 Direct And Inverse Proportions Draw Five Squares Of Different Sides

Find whether the length of a side is in direct proportion to the:

  1. The perimeter of the square.
  2. The area of the square.

Solution:

NCERT Solutions For Class 8 Maths Chapter 10 Direct And Inverse Proportions The Perimeter Of The Square

Since each \(\frac{L}{\mathrm{P}}\) is the same, so we find that the length of a side is in direct proportion to the perimeter of the square.

Since all \(\frac{L}{\mathrm{a}}\) are not the same, therefore, we find that the length of a side is not in direct proportion to the area of the square.

Question 12. The following ingredients are required to make halwa for 5 persons: SujiRawa = 250 g, Sugar = 300 g, Ghee = 200 g, Water = 500 ml. Using the concept of proportion, estimate the changes in the quantity of ingredients, to prepare halwa for your class.
Solution:

Suppose that there are 25 (= 5 x 5) students in the class. Then, this is a case of direct variation. Hence, we shall require five times the quantity of ingredients.

Question 3. Choose a scale and make a map of your classroom., showing windows, doors blackboard, etc. (An example is given here),

NCERT Solutions For Class 8 Maths Chapter 10 Direct And Inverse Proportions Make A Map Of your Classroom

Solution:

Please make yourself.

Question 13. Take a few problems discussed so far under ‘direct variation’. Do you think that they can be solved by the ‘unitary method’?
Solution:

Yes, they can be solved by the unitary method

∴  75 mL of the base is required for

= 1 part of red pigment

∴  1 mL of the base is required for

\(\frac{1}{75}\) part of red pigment

∴  1800 mL of the base is required for

⇒ \(=\frac{1}{75} \times 1800=\frac{120}{5}\)

= 24 parts of red pigment

Number of bottles filled in 6 hours = 840

Number of bottles filled in 1 hour \(\)

⇒ \(\frac{840}{6}=140\)

∴  Number of bottles filled in 5 hours = 140 x 5 = 700

∴  Distance travelled in 25 minutes = 14 km

∴  Distance travelled in 1 minute\(=\frac{14}{25} \mathrm{~km}\)

∴  Distance travelled in 5 hours(= 5 x 60 minutes or 300 minutes)

⇒ \(\frac{14}{25} \times 300 \mathrm{~km}\)

14 x 12 = 168 km

Inverse Proportion

If two quantities are related in such a way that an increase in one quantity leads to a corresponding proportional decrease in the other and vice-versa, then such a variation is called inverse proportion.

Thus, two quantities x and y are said to vary in inverse proportion if xy = k where k is a constant of proportionality

In this case, if. y1, y2 are the values of y corresponding to the values xv x2
f x respectively,

⇒ \(x_1 y_1=x_2 y_2 \quad \text { or } \quad \frac{x_1}{x_2}=\frac{y_2}{y_1}\)

Question 14. Think of more such examples of pairs of quantities that vary in inverse proportion.
Solution:

Some more such examples of pairs of quantities that vary in inverse proportion are as follows :

Number of machines to produce a given number of articles and the number of days required for production.

Number of periods a day in a school and the length of the period.

Number of workers to build a wall and The number of hours to build the wall

Question 15. Take a squared paper and arrange 48 counters on it in different numbers of rows shown below

NCERT Solutions For Class 8 Maths Chapter 10 Direct And Inverse Proportions Squared Paper And Arrange 48 Counters

NCERT Solutions For Class 8 Maths Chapter 10 Direct And Inverse Proportions Squared Paper And Arrange 48 Counters R increases, C decreases

What do you observe? As R increases, C decreases.

  1. Is R1: R2 = C2: C1?
  2. Is R3:R4 = C4:C3 ?
  3. Are R and C inversely proportional to each other?

Try this activity with 36 counters.

Solution:

NCERT Solutions For Class 8 Maths Chapter 10 Direct And Inverse Proportions Squared Paper And Arrange 48 Counters inversely proportional

So \(\frac{\mathrm{R}_1}{\mathrm{R}_2} =\frac{2}{3}\)

⇒ \( \frac{\mathrm{C}_2}{\mathrm{C}_1} =\frac{16}{24}=\frac{2}{3} \)

⇒ \( \frac{\mathrm{R}_1}{\mathrm{R}_2} =\frac{\mathrm{C}_2}{\mathrm{C}_1} \)

⇒ \(\mathrm{R}_1: \mathrm{R}_2 =\mathrm{C}_2: \mathrm{C}_1\)

R1: R2 = C2: C1

⇒ \(\frac{\mathrm{R}_3}{\mathrm{R}_4}=\frac{4}{6}=\frac{2}{3}\)

⇒ \(\frac{\mathrm{C}_4}{\mathrm{C}_3}=\frac{8}{12}=\frac{2}{3}\)

⇒ \(\frac{\mathrm{R}_3}{\mathrm{R}_4}=\frac{\mathrm{C}_4}{\mathrm{C}_3}\)

R3:R4 = C4:C3

Yes! R and C are inversely proportional to each other.

Let us arrange 36 counters in different numbers of rows as shown below :

NCERT Solutions For Class 8 Maths Chapter 10 Direct And Inverse Proportions 36 Counters In Different Number

Let us try this activity with 36 counters as follows :

NCERT Solutions For Class 8 Maths Chapter 10 Direct And Inverse Proportions Squared Paper And Arrange 48 Counters

We observe that as R increases, C decreases.

R1 : R2 = 2:3

C2 : C4= 12 : 18 = 2 : 3

R1:R2 = C2 : C1

R3 : R4 = 4:6 = 2:3

C4 : C3 = 6:9 = 2:3

R3:R4= c4-c3

Yes! R and C are inversely proportional to each other.

Question 16. Observe the following tables and find which pair of variables (here x and y) are in inverse proportion.

NCERT Solutions For Class 8 Maths Chapter 10 Direct And Inverse Proportions Which Pair Of Variables Inverse Proportion

Solution:

x1y1 = 50 x 5 = 250

x2y2 = 40 x6 = 240

So, x1y1 ≠ x2y2

Hence, x and y are not in inverse proportion

x1y1= 100 x 60 = 6000

x2y2 = 200 x 30 = 6000

x3y3 = 300 x 20 = 6000

x4ÿ4 = 400 x 15 = 6000

So , x1y1 = x2y2 = x3y3 = x4y4

Hence, x and y are in inverse proportion.

x1y1= 90 x 10 = 900

x2y2 = 60 x 15 = 900

x3y3 = 45 x 20 = 900

x4ÿ4 = 30 x 25 = 750

As x1y1 = x2y2 = x3y3 ≠ x4y4

So, x and y are not in inverse proportion.

Note: When two quantities x and y are in direct proportion (or vary directly) they are also written as x y.

When two quantities x andy are in inverse proportion (or vary inversely) they are also written as x «c \(\frac{1}{y}\)

Direct And Inverse Proportions Exercise 11.2

Question 1. Which of the following are in inverse proportion 1

  1. The number of workers on a job and the time to complete the job.
  2. The time taken for a journey and the distance traveled at a uniform speed,
  3. Area cultivated land and the crop harvested,
  4. The time taken for a fixed journey and the speed of the vehicle.
  5. The population of a country and the area, of land per person.

Solution:

The number of workers on a job and the time to complete the job are in inverse
proportion, since as the number of workers increases, then the time to complete the job decreases proportionally.

The time taken for a journey and the distance traveled at a uniform speed are not in inverse proportion, since for a longer distance, more time will be required.

The area of cultivated land and the crop harvested is not in inverse proportion, since for more area of cultivated land, more crops will be needed to be harvested.

The time taken for a fixed journey and the speed of the vehicle are in inverse proportion, since the speed of the vehicle, proportionally less would be the time to cover a fixed journey.

The population of the country and the area of land per person are in inverse proportion,
since for more population of the country, the area of land per person would be proportionally less.

Question 2. In a Television game show, the prize money of f1,00,000 is to be divided equally amongst the winners. Complete the following table and find whether the prize money given to an individual winner is directly or inversely proportional to the number of winners.

NCERT Solutions For Class 8 Maths Chapter 10 Direct And Inverse Proportions Area Of Cultivated Land And The Crop Harvested

Solution:

Let the prize (in T) for each winner bey3, y4, y5, y6, and y1 corresponding to the number of winners 4, 5, 8, 10, and 20 respectively. Then,

1 X 1,00,000 = 4 X y3 \(y_3=\frac{1,00,000}{4}=25,000\)

1 x 1,00,000 = 5 x y4 \(y_3=\frac{1,00,000}{5}=20,000\)

1 x 1,00,000 = 8 x y5 \(y_3=\frac{1,00,000}{8}=12,000\)

1 x 1,00,000 = 10 x y6 \(y_3=\frac{1,00,000}{10}=10,000\)

1 x 1,00,000 = 20 x y7 \(y_3=\frac{1,00,000}{20}=5,000\)

NCERT Solutions For Class 8 Maths Chapter 10 Direct And Inverse Proportions As Number Of winners Increases, Amount Of Prize Decreases

As the number of winners increases, amount of prize decreases. So the prize money given to an individual winner is inversely proportional to the number of winners.

Question 3. Rehrnan is making any paira wheel of using spokes in such a way that the angles between any pair of consecutive equal. Help equal him spokes by completing the following table:

NCERT Solutions For Class 8 Maths Chapter 10 Direct And Inverse Proportions Consecutive Using Spokes

  1. Are the number of spokes and the angles formed between the pairs of consecutive spokes in inverse proportion?
  2. Calculate the angle between a pair of consecutive spokes on a wheel with 15 spokes.
  3. How many spokes would be needed, if the angle between a pair of-consecutive spokes is 40°?

Solution:

Let the angles (in degree) between a pair of consecutive spokes be y3, y4, and y5 respectively. Then,

⇒ \(y_3=\frac{360^{\circ}}{8}=45^{\circ}\)

⇒ \(y_4=\frac{360^{\circ}}{10}=36^{\circ}\)

⇒ \(y_5=\frac{360^{\circ}}{12}=30^{\circ}\)

NCERT Solutions For Class 8 Maths Chapter 10 Direct And Inverse Proportions A Pair Of Consecutive Spokes

Yes; The number of spokes and the angles formed between the pairs of consecutive spokes are in inverse proportion.

|4 x 90° = 6 x 60° = 8 x 45° = 10 x 36° = 12 x 30°

Let the angle (in degrees) between a pair of consecutive spokes on a wheel with 15 spokes be y6.

The smaller the number of spikes, the more the angle between a pair of consecutive spokes.

So, this is a case of inverse proportion.

Hence, 4 x 90° = 15 x y

⇒ \(y_6=\frac{4 \times 90^{\circ}}{15}\) x1y1 = x2y2

y6 = 24°

Hence, the angle between a pair of consecutive spokes on a wheel with 15 spokes is 24°.

Let x spokes be needed.

The lesser the number of spokes, the more will be the angle between a pair of consecutive spokes, bo, this is a case of inverse proportion.

Hence, 4 x 90° = x x 40°

⇒ \(x=\frac{4 \times 90^{\circ}}{40}\) x1y1 = x2y2

X = 9

Hence, 9 spokes would be needed, if the angle between a pair of consecutive spokes is 40″,

Question 4. If a box of sweets is divided among 21 children, they will get 5 sweets each. How many would each get, if the number of the children is reduced by 4?
Solution:

Suppose that each would get y2 sweets

Thus, we have the following table :

NCERT Solutions For Class 8 Maths Chapter 10 Direct And Inverse Proportions If A Box Of Sweets

The fewer the number of children, the more the number of sweets each would get. So, this is a case of inverse proportion.

Hence 24 x 5 = 20 x y2

⇒ \(y_2=\frac{24 \times 5}{20}\)

y2 = 6

Hence, each would get 6 sweets, if the number of children is reduced by 4.

Question 5. A farmer has enough food to feed 20 animals in his cattle for 6 days. How long would the food last if there were 10 more animals in his cattle?
Solution:

Suppose that the food would last for two days. We have the following table :

NCERT Solutions For Class 8 Maths Chapter 10 Direct And Inverse Proportions A Farmer Has Enough Food To Feed

We note that the more the number of animals, the lesser will be the number of days for which the food will last. Therefore, this is a case of inverse proportion.

So, 20 x 6 = 30 x y2

⇒ \(y_2=\frac{20 \times 6}{30}\)

y2 = 4

Hence, the food would last for 4 days, if there were 10 more animals in his cattle.

Question 6. A contractor estimates that 3 persons could rewire Dasminder’s house in 4 days. If, he uses 4 persons instead of three, how long should they take to complete the job?
Solution:

Suppose that they take y2 days to complete the job. We have the following table :

NCERT Solutions For Class 8 Maths Chapter 10 Direct And Inverse Proportions A Contractor Estimates

The more the number of persons, the lesser will be the number of days required to complete the job. So, this is a case of inverse proportion.

Hence, 3 x 4 =4 x y2

⇒ \(y_2=\frac{3 \times 4}{4}\)

y2 = 3

Hence, they would take 3 days to complete the job.

Question 7. A batch of bottles was packed in 25 boxes with 12 bottles in each box. If the same batch is packed using 20 bottles in each box, how many boxes would be filled?

NCERT Solutions For Class 8 Maths Chapter 10 Direct And Inverse Proportions A Batch Of Bottles Were Packed

Solution:

Suppose that y2 boxes would be filled. We have the following table:

NCERT Solutions For Class 8 Maths Chapter 10 Direct And Inverse Proportions A Batch Of Bottles Were Packed Lesser The Number Of Bottles

The smaller the number of bottles, the more the number of boxes required to be filled. So, this is a case of inverse proportion.

Hence, 12 x 25 = 20 x y2

⇒ \(y_2=\frac{12 \times 25}{20}=3 \times 5\)

y2= 15

Hence, 15 boxes would be filled if the same batch is packed using 20 bottles in each box.

Question 8. A factory requires 42 machines to produce a given number of articles in 63 days. How many machines would be required to produce the same number of articles in 54 days?
Solution:

Suppose that x2 machines would be required. We have the following table :

NCERT Solutions For Class 8 Maths Chapter 10 Direct And Inverse Proportions I factory requires 42 machines

The smaller the number of machines, the more will be the number of days to produce the same number of articles.

So, this is a case of inverse proportion.

Hence, 42 x 63 = x2 x 54

⇒ \(x_2=\frac{42 \times 63}{54}=\frac{21 \times 7}{3}\)

x2= 49

Hence, 49 machines would be required to produce the same number of articles in 54 days.

Question 9. A car takes 2 hours to reach a destination by traveling at the speed of 60 km/h. How long will it take when the car travels at the speed of 80 km/h?
Solution:

Let it take two hours. We have the following table :

NCERT Solutions For Class 8 Maths Chapter 10 Direct And Inverse Proportions A Car Lakes 2 Hours To Reach A Destination

Lesser the speed, the more the number of hours to reach the destination. So, this is a case of inverse proportion.

Hence, 60 x 2 = 80 x y2

⇒ \(y_2=\frac{60 \times 2}{80}=\frac{3 \times 2}{4}\)

⇒ \(y_2=\frac{3}{2}=1 \frac{1}{2}\)

Thus, \(1 \frac{1}{2}\) hours would be taken when the car travels at the speed of 80 km/h.

Question 10. Two people could fit new windows in a house in 3 days.

  1. One of the people fell ill before the work started. How long would the job take now?
  2. How many people would be needed to fit the windows in one day?

Solution:

Let the job would take y2 days.

We have the following table :

NCERT Solutions For Class 8 Maths Chapter 10 Direct And Inverse Proportions Two Persons Could Fit

The more the number of people, the lesser the number of days to do the job. So, the number of persons and number of days vary in inverse proportion.

So, 2 x 3= 1 x y2

Thus, the job would now take 6 days when one of the persons fell ill before the work started.

Let y3 persons be needed.

We have the following table :

NCERT Solutions For Class 8 Maths Chapter 10 Direct And Inverse Proportions The Persons Fell ill Before The Work

Clearly, the more the number of people, the lesser the number of days to do the job. So, the number of persons and number of days vary in inverse proportion.

So, 3 x 2 = 1 x y3

y3= 6

Thus, 6 people would be needed to fit the windows in one day.

Question 11. A school has 8 periods a day each of 45 minutes duration. How long would each period be, if the school has 9 periods a day, assuming the number of school hours to be the same?
Solution:

Let each period be two minutes long. We have the following table :

NCERT Solutions For Class 8 Maths Chapter 10 Direct And Inverse Proportions A school has 8 periods a day Duration

Wo note that the more the number of periods, the lesser would be the length of each period. Therefore, this is a case of inverse proportion.

So, 8 x 45 = 9 x y2

⇒ \(y_2=\frac{8 \times 45}{9}\)

y2 = 40

Hence, each period would be 40 minutes long if the school had 9 periods a day.

Question 12. Take a sheet of paper. Fold it as shown in the figure. Count the number of parts and the area of a part in each case.

NCERT Solutions For Class 8 Maths Chapter 10 Direct And Inverse Proportions Sheet Of Paper

Tabulate your observations and discuss them with your friends. Is it a case of inverse proportion? Why?

NCERT Solutions For Class 8 Maths Chapter 10 Direct And Inverse Proportions Case Of Inverse Proportion

Solution:

NCERT Solutions For Class 8 Maths Chapter 10 Direct And Inverse Proportions Case Of Inverse Proportion Area of each part

We see that

⇒ \(1 \times 1=2 \times \frac{1}{2}=4 \times \frac{1}{4}=8 \times \frac{1}{8}=16 \times \frac{1}{16}=1 \text { (= constant) }\)

So yes it is a case of inverse proportion because the greater the number of parts lesser the area of each part.

Question 13. Take a few containers of different sizes with circular bases, Fill the same amount of water in each container. note the diameter of each container and the respective height at which the water levels stand. Tabulate your observations. Is it a case of inverse proportion?

NCERT Solutions For Class 8 Maths Chapter 10 Direct And Inverse Proportions A Few Containers In Different Sizes

NCERT Solutions For Class 8 Maths Chapter 10 Direct And Inverse Proportions Height Of Water Level

Solution:

NCERT Solutions For Class 8 Maths Chapter 10 Direct And Inverse Proportions Diameter of Container

Let the same amount of water in each container be V. Then

⇒ \(V=\pi\left(\frac{d_1}{2}\right)^2 h_1=\pi\left(\frac{d_2}{2}\right)^2 h_2=\pi\left(\frac{d_3}{2}\right)^2 h_3\)

d12h1 = d22h2 = d32h3

d1h1= d2h2 = d3h3

So, it is not a case of inverse proportion.

It does not imply that

Direct And Inverse Proportions Multiple-Choice Question And Solutions

Question 1. 10 meters of cloth cost? 1000. What will 4 meters cost?

  1. ₹ 400
  2. ₹ 800
  3. ₹ 200
  4. ₹ 100

Solution: 1. ₹ 400

⇒ \(\frac{10}{1000}=\frac{4}{?} \quad \Rightarrow ?=400\)

Question 2. 15 books weigh 6 kg. What will 6 books weigh?

  1. 1.2 kg
  2. 2.4 kg
  3. 3.8 kg
  4. 3 kg

Solution: 2. 2.4 kg

⇒ \(\frac{15}{6}=\frac{6}{?} \quad \Rightarrow ?=2.4\)

Question 3. Does a horse eat 18 kg of corn in 12 days? How much does he eat in 9 days?

  1. 11.5 kg
  2. 12.5 kg
  3. 13.5 kg
  4. 14.5 kg

Solution: 3. 13.5 kg

⇒ \(\frac{12}{18}=\frac{9}{?} \quad \Rightarrow ?=\frac{18 \times 9}{12}\)

= 13.5

Question 4. 8 g of sandalwood cost ₹ 40. What will 10 g cost?

  1. ₹ 30
  2. ₹ 36
  3. ₹ 48
  4. ₹ 50

Solution: 4. ₹ 50

⇒ \(\frac{8}{40}=\frac{10}{?} \quad \Rightarrow ?=50\)

Question 5. 20 trucks can hold 150 metric tonnes. How much will 12 trucks hold?

  1. 80 metric tonnes
  2. 90 metric tonnes
  3. 60 metric tonnes
  4. 40 metric tonnes.

Solution: 2. 90 metric tonnes

⇒ \(\frac{20}{150}=\frac{12}{?} \quad \Rightarrow ?=90 .\)

Question 6. 120 copies of a book cost ₹ 600. What will 400 copies cost?

  1. ₹ 1000
  2. ₹ 2000
  3. ₹ 3000
  4. ₹ 2400

Solution: 2. ₹ 2000

⇒  \(\frac{120}{600}=\frac{400}{?} \Rightarrow ?=2000 .\)

Question 7. The rent of 7 hectares is ₹ 875. What is the rent of 16 hectares?

  1. ₹ 2000
  2. ₹ 1500
  3. ₹ 1600
  4. ₹ 1200

Solution: 1. ₹ 2000

⇒ \(\frac{7}{875}=\frac{16}{?} \quad \Rightarrow ?=\frac{875 \times 16}{7}\)

= 2000

Question 8. A boy runs 1 km in 10 minutes. How long will it take to run 600 m?

  1. 2 minutes
  2. 3 minutes
  3. 4 minutes
  4. 6 minutes

Solution: 4. 6 minutes

⇒ \(\frac{1000}{10}=\frac{600}{?} \Rightarrow ?=6\)

Question 9. A shot travels 90 m in 1 second. How long will it take to go 225 m?

  1. 2 seconds
  2. 2.5.seconds
  3. 4 seconds
  4. 3.5 seconds

Solution: 2. 2.5.seconds

⇒ \(\frac{90}{1}=\frac{225}{?} \Rightarrow ?=2.5 .\)

Question 10. 3 knives cost ₹ 63. What will 17 knives cost?

  1. ₹ 357
  2. ₹ 375
  3. ₹ 537
  4. ₹ 573

Solution: 1. ₹ 357

⇒ \(\frac{3}{63}=\frac{17}{?} \quad \Rightarrow \quad ?=357\)

Question 11. 15 men can mow 40 hectares of land in 1 day. How much will 6 men mow in 1 day?

  1. 16 hectares
  2. 12 hectares
  3. 20 hectares
  4. 24 hectares

Solution: 1. 16 hectares

⇒ \(\frac{15}{40}=\frac{6}{?} \quad \Rightarrow ?=\frac{40 \times 6}{15}=16 .\)

Question 12. A man walks 20 km in 5 hours. How long would it take to walk 32 km?

  1. 3 hours
  2. 4 hours
  3. 6 hours
  4. 8 hours

Solution: 4. 8 hours

⇒ \(\frac{20}{5}=\frac{32}{?} \quad \Rightarrow ?=8\)

Question 13. What is the cost of 50 sticks at ₹ 24 per score?

  1. ₹ 30
  2. ₹ 40
  3. ₹ 50
  4. ₹ 60

Solution: 4. ₹ 60

⇒ \(\frac{20}{24}=\frac{50}{?} \quad \Rightarrow ?=60 \text {. }\)

Question 14. A train travels 60 km in 1 hour. How long will it take to go 150 km?

  1. 2 hours
  2. 3 hours
  3. 2.5 hours
  4. 4 hours.

Solution: 3. 2.5 hours

⇒ \(\frac{60}{1}=\frac{150}{?} \quad \Rightarrow ?=2.5\)

Question 15. If 3 quintals of coal cost ₹ 6000, what is the cost of 120 kg?

  1. ₹ 1200
  2. ₹ 2400
  3. ₹ 3600
  4. ₹ 4800

Solution: 2. ₹ 2400

⇒ \(\frac{300}{6000}=\frac{120}{?} \Rightarrow ?=2400\)

Question 16. If 20 cows eat as much as 15 oxen, how many cows will eat as much as 36 oxen?

  1. 40
  2. 44
  3. 45
  4. 48

Solution: 4. 48

⇒ \(\frac{15}{20}=\frac{36}{?} \quad \Rightarrow ?=\frac{20 \times 36}{15}\)= 48

Question 17. The fare for a journey of 40 km is ₹ 25. How much can be traveled for ₹ 40?

  1. 32 km
  2. 64 km
  3. 50 km
  4. 60 km

Solution: 2. 64 km

⇒ \(\frac{25}{40}=\frac{40}{?} \Rightarrow ?=\frac{40 \times 40}{25}\)= 74

Question 18. Apala types 200 words in half an hour. How many words will she type in 12 minutes?

  1. 80
  2. 50
  3. 100
  4. 60

Solution: 1. 80

⇒ \(\frac{30}{200}=\frac{12}{?} \quad \Rightarrow ?=80 \text {. }\)

Question 19. A labourer is paid? 400 for 2 days of work. If he works for 5 days, how much will he get?

  1. ₹ 1000
  2. ₹ 800
  3. ₹ 750
  4. ₹ 900

Solution: 1. ₹ 1000

⇒ \(\frac{2}{400}=\frac{5}{?} \quad \Rightarrow ?=1000 .\)

Question 20. A machine in a soft drink factory fills 600 bottles in 5 hours. How many bottles will it fill in 2 hours?

  1. 120
  2. 180
  3. 150
  4. 240

Solution: 4. 240

⇒ \(\frac{5}{600}=\frac{2}{?} \quad \Rightarrow ?=240 .\)

Question 21. If 8 men can do a piece of work in 20 days, in how many days could 20 men do the same work?

  1. 6 days
  2. 8 days
  3. 4 days
  4. 10 days

Solution: 2. 8 days

⇒ \(8 \times 20=20 \times ? \quad \Rightarrow ?=8\)

⇒ \(5 \times 3=15,7 \times 3=21 \)

Question 22. If an amount of food last for 40 days for 120 men, how long will it last for 80 men at the same rate?

  1. 50 days
  2. 60 days
  3. 80 days
  4. 100 days

Solution: 2. 60 days

⇒ \( 40 \times 120=80 \times ? \quad \Rightarrow ?=60 \)

Question 23. If 18 women can reap a field in 7 days, in what time can 6 women reap the same field?

  1. 15 days
  2. 21 days
  3. 30 days
  4. 36 days

Solution: 2. 21 days

⇒ \(18 \times 7=6 \times ? \quad \Rightarrow ?=21 .\)

Question 24. 10 men can dig a trench in 15 days. How long will 3 men take?

  1. 50 days
  2. 60 days
  3. 100 days
  4. 75 days

Solution: 1. 50 days

⇒ \( 10 \times 15=3 \times ? \quad \Rightarrow ?=50\)

Question 25. 3 lambs finish eating turnips in 8 days. In how many days will 2 lambs finish them?

  1. 6
  2. 8
  3. 10
  4. 12

Solution: 4. 12

\(3 \times 8=2 \times ? \Rightarrow ?=12 .\)

Question 26. 6 pipes are required to fill a tank in 1 hour. How long will it take if only 5 pipes of the same type are used?

  1. 75 minutes
  2. 72 minutes
  3. 80 minutes
  4. 90 minutes.

Solution: 2. 72 minutes

⇒ \( 6 \times 60=5 \times? \)

⇒ \(?=72 \text { minutes. }\)

Question 27. 40 cows can graze a field in 16 days. How many cows will graze the same field in 10 days?

  1. 60
  2. 64
  3. 80
  4. 75

Solution: 2. 64

⇒ \(40 \times 16=? \times 10 \quad \Rightarrow ?=64 .\)

Question 28. If x = ky and when y = 4, x = 8 then k =

  1. 1
  2. 2
  3. 3
  4. 4

Solution: 2. 2

⇒ \(8=4 k \quad \Rightarrow k=2 .\)

Question 29. The constant of variation, if x «= y, from the following table, is

x     6     12     15    21

y     2      4       5      7

  1. 1
  2. 2
  3. 3
  4. 4

Solution: 3. 3

⇒ \(2 \times 3=6,4 \times 3=12, \\\)

Question 30. x and y vary inversely with each other. If x = 15 when y- 6, then the value of x when = 15 is

  1. 2
  2. 4
  3. 5
  4. 6

Solution: 4. 4

⇒ \( 15 \times 6=? \times 15 \quad \Rightarrow ?=6 \)

Direct And Inverse Proportions True-False

Write whether the following statements are True or False:

1. The distance traveled by a CNG vehicle and the amount of CNG consumed are directly proportional: True

2. If and q are in inverse proportion, then (p+l) and (q- 1) are also in inverse proportion: True

3. If Apala can finish a work in n days, then the amount of work done by her in one day is \(\frac{1}{y}\): True

4. If two quantities p and q vary inversely, then pq remains constant: True

5. If x and y are in direct proportion, then \(\frac{1}{x} \text { and } \frac{1}{y}\) are also in direct proportion: True

Direct And Inverse Proportions Fill in the Blanks

1. If xy = 1, then x and y vary ______ with each other: Inversely

2. The radius and the circumference of a circle vary ______ with each other.
sheets of the same: Directly

3. If the weight of 10 sheets of an A-4 size paper sheet is 30 g, then ______
the paper would weigh 90 g: 30

4. If the amount of work finished by Meenu in one day is \(\frac{1}{m}\) then the whole work will be finished by her in______days: m

5. If the speed remains constant, then the distance traveled is ______ proportional to the time: Directly

6. The area occupied by 10 postal stamps is 40 cm2. Find the area occupied by 100 such stamps: 400cm2

7. In a camp, there is enough flour for 50 persons for 40 days. How long will the food last, if 30 more people join the camp: 25days

8. 20 persons can reap a field in 15 days. How many more persons should be engaged to reap the same field in 10 days: 30

9. A car is traveling 80 km per hour. Find the distance traveled by car in 3 hours, if the speed remains constant: 240km

10. If x varies inversely as y and x = 20 when y = 30. Then, find y when x = 50: 12

NCERT Solutions For Class 8 Maths Chapter 12 Factorisation

Factorization Introduction

In this chapter, we shall learn about the factorization of algebraic expressions, methods of factorization, and division of algebraic expressions.

Factors Of Natural Numbers

A number when written as a product of its prime factors is said to be in the prime factor form. Similarly, we can express algebraic expressions as products of their factors.

Factors Of Algebraic Expressions

A fundamental factor cannot be expressed further as a product of factors.

What Is Factorisation

When we factorize an algebraic expression, we write it as a product of irreducible factors. These factors may be numbers, algebraic variables, or algebraic expressions. This process is called factorization.

Method Of Common Factors

We factorize each term of the given algebraic expression as a product of irreducible factors and separate the common factors. Then, we combine the remaining factors in each term using the distributive law.

Read and Learn More NCERT Solutions For Class 8 Maths

Question 1. Factorise:

12x + 36

22y – 33z

14pq + 35pqr.

Solution:

We have

12i = 2 x 2 x 3 x x

36 = 2 x 2 x 3 x 3

The two terms have 2, 2, and 3 as

common factors.

Therefore, 12x + 36

= (2 x 2 x 3 x x) + (2 x 2 x 3 x 3)

= 2 x 2 x 3 x (x + 3) using distributive law

= 12 x (x + 3)

= 12(x + 3)

which is the required factor form.

We have

22y = 2 x ll x y

33z = 3 x li x z

The two terms have 11 as a common factor.

Therefore,

22y – 33z

= (11 x 2 x y) – (11 x 3 x z)

= 11 X [(2 X y) – (3 X Z)]

I use the distributive law

= 11 x (2y – 3z)

= ll(2y – 3z)

which is the required factor form.

We have

14pq = 2 x 7 x p x q

35pgr = 5×7 x p x g x r

The two terms have 7, p, and q as common factors.

Therefore, 14pq + 35pqr

= 7 X p x q x 2 + 7 x p x q x 5 x r

= 7 x p x q x [2 + (5 x r)]

using distributive law

= 7pq x (2 + 5r)

= 7pq(2 + 5r).

which is the required factor form.

Note:  We notice that the factor form of an expression has only  one term

Factorization By Regrouping Terms

Sometimes it so happens that all the terms in a given algebraic expression do not have a common factor, but the terms can be grouped so that all the terms in each group have a common factor. In doing so, we get a common factor across all the groups formed. This leads to the required factorization of the given algebraic expression.

Factorization Exercise 12.1

Question 1. Find the common factors of the given terms:

  1. 12x, 36
  2. 2y, 22xy
  3. 14pq, 28p2q2
  4. 2x, 3x2, 4
  5. 6abc, 24ab2, 12a2b
  6. 16x3-4x2, 32x
  7. 10pq, 20qr, 30rp
  8. 3x2y3, 10x3y2, 6x2y2z.

Solution:

1.  12x = 2 × 2 × 3 × x

36 = 2 × 2 × 3 × 3

Common prime factors are 2 (occurs twice) and 3.

Required common factor

= 2 × 2 × 3 = 12

2.  2y = 2 x y

22xy = 2 × 11 × x × y

Common factors are 2 and y.

Required common factor = 2 × y = 2y

3.  14pq = 2 x 7 x p x q

28 p2q2 = 2 × 2 × 7 × p × p × q × q

Common factors are 2, 7, p, and g.

Required common factor

= 2 × 7 × p × q = 14pg

4.  2x = 1 × 2 × x

3ÿ = 1 × 3 × x × x

4 = 1× 2 × 2

Common factor is 1

Required common factor = 1

5.  6abc = 2 x 3 x a x b x c

24ab2 = 2 × 2 × 2 × 3 × a × b × b

12a2b = 2 × 2 × 3 × a × a × b

Common factors are 2, 3, a and b

Required common factor

= 2 × 3 × a × b = 6ab

6. 16x3 = 2 × 2 × 2 × 2 × x × y × x

-4X2 =-1 × 2 × 2 × y × X

32r = 2 ×2 × 2 × 2 × 2 × x

Common factors are 2 (occurs twice) and x (occurs once).

Required common factor = 2 × 2 × x = 4x

7.  10pq = 2 ×5 × p × q

20qr = 2 × 2 × 5 × q × r

30rp = 2 x 3 x 5 x r xp

Common factors are 2 and 5.

Required common factor = 2 x 5 = 10

8.  3x2y3 = 3 X x X x X y X y X y

10x3y2 = 2 x 5 x x x x x x y x y

6 x3 y2 z = 2 x 3 x z x x x y X y X z

Common factors are x (occurs twice) and y (occurs twice)

Required common factor = X X X X y X y = x2y2

Question 2. Factorize the following expressions:

  1. 7x – 42
  2. 6p – 12q
  3. 7a2 + 14a
  4. -16z + 20z3
  5. 20l2m + 30 alm
  6. 5x2y – 15xy2
  7. 10a2 – 15b2 + 20c2
  8. – 4a2 + 4ab – 4ca
  9. x2yz + xy2z + xyz2
  10. ax2y + bxy2 + cxyz.

Solution:

1.  7x = 7 × x

42 = 2 x 3 x 7

7x – 42

= l x x – 2 x 3 x 7

= 7(x-2 x 3)

using distributive law

= 7 (x – 6)

It is the required factor form.

2. 6p – 2 x 3 x p

12g = 2 x 2 x 3 x q

6p – 12q

= 2 x 3 x p – 2 x 2 x 3 x q

= 2 x 3(p – 2 x q)

using distributive law

= 6(p – 2q)

It is the required factor form.

3. 7a2 = 7 x a x a

14a = 2 x 7 x a

7a2 + 14a

= 7 x a x a + 2 x 7 x a

= 7 x a (a + 2)

I use the distributive law

= 7a (a + 2)

It is the required factor form.

4. -16z = (-1) x 2 x 2 x 2 x 2 x z

20Z3= 2 × 2 × 5 × Z × x × Z

A – 16z + 20 ×3 N

= (-1) x 2 x 2 x 2 x 2 x 2 + 2 x 2 x 5 x z x z x z

= (2 x 2 x z) [(-1) x 2 x 2 + 5 X z X z] 5

I am using the distributive law

= 4z(-4 + 5z2)

It is the required factor form.

5.  20l2m = 2 x 2 x 5 x l x l x m

30alm = 2 x 3 x 6 x a x l x m

A 20l2m + 30alm

= 2 x 2 x 5 x l x l x m

+ 2 x 3 x 5 x a x l x m

= 2 x 5 x l x m

x (2 x l + 3 x a)

= 10lm (2l + 3a)

| Using the distributive law

It is the required factor form.

6.  5x2y = 5 × x × x × y

15xy× = 3 × 5 × x × y × y

A 5x2y – 15xy2

= 5 × x × x × y

-3 × 5 × x × y × y

= (5 × x × y)(x – 3 x y)

| Using the distributive law

= 5xy (x – 3y)

It is the required factor form.

7.  10a2 = 2 × 5 × a × a

1562 = 3 × 5 × b × b

20c2= 2 × 2 × 5 × c × c

10a2 – 15b2 + 20c2

= 2 × 5  × a × a – 3 × 5 × b × b +2 × 2 × 5 × c × c

+2 × 2 × 5 × c × c

= 5 (2 × a × a -3 × b × b + 2 × 2 × c × c)

using distributive law

= 5(2a2 – 3b2 + 4c2)

It is the required factor form.

8. 4a2 = 2 x 2 x a x a

4ab = 2 x 2 x a x 6

4ca = 2 x 2 x c x a

– 4a2 + 4ab – 4ca

= (-1) × 2 ×2 ×a × a

+ 2 ×  2 × a × b

-2 × 2 × c × a

= 2 × 2 × a [(- 1) × a + 6 – c)]

I use the distributive law

= 4a (- a. + 6 – c)

It is the required factor form.

9.  x2yz = x × x × y × z

xy2z =x × y × y × z

xyz2 =x × y × z × z

x2yz + xyz + xyz2

= X × X × y × z

+ X × y × y × Z

+ X × y × Z × z

= X × y × z(x + y + z)

I use the distributive law

= xyz{x + y + z)

It is the required factor form.

10.  ax2y = a × x × x × y

bxy2 = b × x × y × y

Xyz = C × X × y × z

ax2y + bxy2 + XYZ

= a X × X × X y + b × x × y X y + C × X × y × z

= X X y (a × x+b × y + c × z)

I use the distributive law

= xy (ax + by + cz).

It is the required factor form.

Question 3. Factorise :

  1.  x2 + xy + 8x + 8y
  2. 15xy – 6x + 5y – 2
  3. ax + bx – ay – by
  4. 15pq + 15 + 9q + 25p
  5.  z – 7 + 7xy – xyz.

Soiution: 

x2 + xy + 8x + 8y

= x(x + y) + 8(x + y)

Taking x common in the first two terms

and 8 commons in the last two terms.

= (x + y)(x + 8)

Taking (x + y) common

It is the required form.

2. 15xy – 6x + 5y – 2

= 3x(5y – 2) + 1(5y – 2)

Taldng 3x common in first two terms and 1 common in last two terms.

= (5y – 2) (3x + 1)

Taking (5y- 2) common

It is the required factor form.

3.  ax + bx – ay – by

= x(a + b) – y(a + b)

I Taking .v common in the first two terms x and -y common in the last two terms.

= (a + b)(x – y)

| Taking (a + b) common

It is the required factor form.

4. 15pq + 15 + 9g + 25p

= 15pq + 9q + 25p + 15

Arranging the terms

= 3q(5p + 3) + 5(5p + 3)

Taking 3g common in the first two terms and 5 common in the last two terms.

= (5p + 3)(3q + 5)

Taking (5p + 3) common

It is the required factor form.

5.  z-7 + 7xy – xyz

= z – 7 – XYZ + Ixy

Arranging the terms

= 1(z – 7) – xy (z – 7)

Taking 1 common in the first two terms

and- xy common in the last two terms.

= (z – 7) (1 – xy).

Taking (z – 7) common

It is the required factor form.

Factorization Using Identities

The following identities prove to be quite helpful in factorization of an algebraic expression :

(a + b)2 = a2 + 2ab + b2

(a – b)2 = a2 – 2ab + b2

(a + b) (a – b) = a2 – b2

12.2.4 Factors Of The Form (X + A) {X + B)

(x + a) (x + b) = x2 + (a. + b) x + ab

To factorize an algebraic expression of type x2 + px + q, we find two factors a and b of q such that

ab = q and a + b-p

Then, the given expression becomes

x2 + (a + b) x + ab = x2 + ax + bx + ab

= x (x + a) + b (x + a) = (x + a) (x + b)

which are the required factors.

Factorization Exercise 12.2

Question 1. Factorize the following expressions:

  1. a2 + 8a + 16
  2. p2 – lOp + 25
  3. 25m2 + 30m + 9
  4. 49y2 + 84yz + 36z2
  5. 4x2 – 8x + 4 K
  6. 121b2 – 88bc + 16c2
  7. (l + m)2 – 4lm      [Hint: Expand {l + m)2 first]
  8. a4 + 2a2b2 + b4

Solution:

1.  a2 + 8a+ 16

= (a)2 + 2 (a)(4) + (4)2

= (a + 4)2

Using Identity I

= (a + 4) (a + 4)

It is the required factor form.

2.   p2 – 10p + 25

= (p)2 – 2(p) (5) + (5)2

= (p-5)2

1 Using Identity II

= (p – 5) (p – 5)

It is the required factor form.

3.  25m2 + 30m + 9

= (5m)2 + 2(5m) (3) + (3)2

= (5m + 3)2

Using Identity I

= (5m + 3) (5m + 3)

It is the required factor form.

4.  49y2 + 84yz + 36z2

= (7y)2 + 2(7y) (6z) + (6z2

= (7y + 6z)2 I Using Identity I

= (7y + 6z) (7y + 6z)

It is the required factor form.

5.  4x2 – 8x + 4

= 4(x2 – 2x + 1)

= 4 [(x)2 – 2(x) (1) + (1)2]

= 4(x – 1)2

Using Identity II

= 4(x – 1) (x – 1)

It is the required factor form.

6.  121b2 – 88bc + 16c2

= (11b)2 – 2(11b) (4c) + (4c)2

= (11b – 4c)2

Using Identity II

= (11b – 4c) (11b – 4c)

It is the required factor form.

7.  (l + m)2 – 4lm

= (l2 + 2lm + m2) – 4lm

I Using Identity I

= l2 + (2lm – 4lm) + m2

I Combining the like terms

= l2 – 2lm + m2

= (l)2 – 2(l)(m) + (m2

= (l – m)2

Using Identity II

= (l – m) (l- m)

It is the required factor form.

8.  a4 + 2a2b2 + b4

= (a2)2 + 2(a2) (b2) + (b2)2

= (a2 + b2)2 I Using Identity I

= (a2 + b2) (a2 + b2)

It is the required factor form.

Question 2. Factorise:

  1. 4p2 – 9q2
  2. 63a2 – 112b2
  3. 49x2 – 36
  4. 16X5 – 144x3
  5. (l + m)2 – (l – m)2
  6. 9x2y2 – 16
  7. (x2 – 2xy + y2) – z2
  8. 25a2 – 4b2 + 28bc – 49c2.

Solution:

1.  4p2 – 9q2

= (2p)2– m2

= (2p – 3q) (2p + 3q)

I am Using Identity III

It is the required factor form.

2.  63a2 – 112b2

= 7 (9a2 – 1662) | Taking 7 common

= 7 {(3a)2 – (46)2}

= 7 (3a – 45) (3a + 45)

Using Identity III

It is the required factor form

3.  49x2 – 36

= (7x)2 – (6)2

= (7x – 6) (7x + 6)

1 Using Identity III

It is the required factor form.

4. 16x5 – 144x3

= 16x3(x2 – 9)

I Taking 16x3 common

= 16x3 {(x)2 – (3)2}

= 16x3 (x – 3) (x + 3)

Using Identity III

It is the required factor form.

5.  (l + m)2 – (l – m)2

= {(l + m) – (l -m)}

x {(l + m) + (l – m)}

Using Identity III

= (l + m – l + m) (l + m + l – m)

= (2m) (2l)

= 4lm

It is the required factor form.

6.  9x2y2 – 16

= (3xy)2 – (4)2

= (3xy – 4) (3xy + 4)

Using Identity II

It is the required factor form

7. (x2 – 2xy + y2) – z2

= (x- y)2 – z2

Using Identity II

= (x – y – z) (x – y + z)

Using Identity III

It is the required factor form.

8.  25a2 – 4b2 + 28b2 – 49c2

= 25a2 – (4b2 – 28bc + 49c2)

= 25a2 – {(2b)2 – 2(2b) (7c) + (7c)2}

= (5a)2 – (2b – 7c)2

Using Identity II

= {5a – (2b – 7c)} {5a + (2b – 7c)}

I Using Identity III

= (5a – 2b + 7c) (5a + 2b – 7c).

It is the required factor form.

Question 3. Factorize the expressions:

  1. ax2 + bx2
  2. 7p2 + 21q2
  3. 2x3 + 2xy2+ 2xz2
  4. am2 + bm2 + bn2 + an2
  5. (Im + l) + m + 1
  6. y(y + z) + 9(y + z)
  7. 5y2 – 20y – 8z + 2yz
  8. lOab + 4a + 5b + 2
  9. 6xy – 4y + 6 – 9x.

Solution:

1.  ax2 = a × x × x

bx = b × x

ax2 + bx = ax × x × +  x = x (a × x + b)

| Using the distributive law

= x (ax + b)

It is the required factor form

2.  7p2 = 7 × p × p

21q2 = 3 × 7 × q × q

7p2 + 21q2 =7 × p × p + 3 × 7 × q × q

= 7(p × p + 3 × q × q)

Using distributive law

= 7(p2 + 3g2)

It is the required factor form

3.  2x3 = 2 × x ×  x × x

2xy2 = 2 × x × y × y

2xz2 = 2 × x × z × z

2X2+ 2y2 + 2z2

= 2 × x × x × x + 2 × x × y × y + 2 × x × z × z

= (2 × x) (x  × X + y × y + Z × z)

Using distributive law

= 2x(x2 + y2 + z2)

It is the required factor form

4.  am2 + bm2 + bn2 + an2

= am2+ bm2 + an2 + bn2

= m2 (a + b) +n2 (a + b)

I Taking m2 commonly in the first two terms

‘ and n2 are common in the last two terms.

= (a + b) (m2 + n2)

I Taking (a + b) common

It is the required factor form.

5.  (lm + l) + m + 1

= l(m + 1) + 1 (m + 1)

Taking l common in the first two terms and 1 common in the last two terms.

= (HI + 1) (l+ 1)

Taking (m + 1) common

It is the required factor form.

6.  y(y + z) + 9(y + z)

= (y + z) (y + 9)

Taking (y + z) common

It is the required factor form.

7. 5y2 – 20y – 8z + 2yz

= 5y2 – 20y + 2yz – 8z

= 5y (y – 4) + 2z(y – 4)

distributive law

= (y – 4) (5y + 2z)

I Taking (y – 4) common

It is the required factor form.

8.  10ab + 4a + 5b+ 2

= (10ab + 4a) + (5b + 2)

I Grouping the terms

= 2a (5b + 2) + 1 (5b + 2)

Using distributive law

= (5b + 2) (2a + 1)

| Taking (56 + 2) common

It is the required factor form.

9.  6xy – 4y + 6 – 9x

= 6xy – 4y – 9x + 6

Grouping the terms

= 2y (3x – 2) – 3(3x – 2)

Using distributive law

= (3x – 2) (2y – 3).

Taking (x – 2) common

It is the required form.

Question 4. Factorise:

  1. a4 – b4
  2. p4– 81
  3. x4 – (y + z)4
  4. x4 – (x – z)4
  5. a4– 2a2b2 + b4

Solution:

1.  a4 – b4

= (a2)2 – (b2)2

= (a2 – b2) (a2 + b2)

Using Identity III

= (a – b) (a + b) (a2 + b2)

| Using Identity III

It is the required factor form.

2.  p4– 81= (p2)2 – (9)2

= (P2 – 9) (p2 + 9)

| Using Identity III

= (P)2-(3)2)(p2 + 9)

= (P – 3) (p + 3) (p2 + 9)

Using Identity III

It is the required factor form.

3.  x4 – (y + z)4

= (x2)2 -{(y + z)2}2

= {x2 – (y + 2)2} {x2+ (y + 2)2}

Using Identity III

= {x – (y + z)} {x + (y + z)} {x2 + (y + z)2}

Using Identity III

= (X – y – Z)(X + y + z) {x2 + (y + z)2}

It is the required factor form.

4.  x4 – (x – z)4

= (x2)2 – {(x – z)}2

= {x2 – (x – z)2}{x2+(x – z)2

Using Identity III

= {x – (x – z)}{x +(x- z)} {x2 + (x – z)2}

Using Identity III

= (x – x + z) (x + X – z)

{x2 + (X – 2)2}

= Z(2X – Z) {X2 + (X – 2)2}

= z(2x – z) (x2 + X2 – 2xz + z2)

Using Identity II

= z(2x – z) (2x2 – 2xz + z2)

It is the required factor form.

5.  a4 – 2a2b2 + b4

= (a2)2– 2(a2) (b2) + (b2)2

= (a2 – b2)2

| Using Identity II

= {(a – 6) ( a + b)}2

Using Identity III

= (a – b)2 (a + b)2

= (a-b) (a – b) (a + b) (a + 6).

It is the required factor form

Question  5. Factorize the following expressions:

  1. p2 + 6p + 8
  2. q2– lOq + 21
  3. p2 + 6p – 16

Solution:

1.  P2 +6p + 8

= p2 + 6p + 9 – 1

= {(p)2 + 2(p)(3) + (3)2}-(l)2

= (p + 3)2 – (l)2

I Using Identity I

= (p + 3 – l)(p + 3 + 1)

Using Identity III

= (p + 2) (p + 4)

It is the required factor form

2.  q2– lOq + 21

= q2 – lOq + 25-4

= {(q)2-2(q)(5) + (5)2}-4

= (q – 5)2 – (2)2

Using Identity II

= (q – 5 – 2) (q – 5 + 2)

Using Identity III

= (q – 7) (q – 3)

It is the required factor form

3.  P2+ 6p – 16

= p2 + 6p + 9 – 25

= (p)2 + 2(p)(3) + (3)2-(5)2

= (P + 3)2 – (5)2

Using Identity I

= (P + 3 – 5) (p + 3 + 5)

I Applying for Identity III

= (p – 2) (p + 8)

It is the required factor form.

Division Of Algebraic Expressions

Here, we shall divide one algebraic expression by another.

Division Of A Monomial By Another Monomial

We shall factorize the numerator and denominator into irreducible factors and cancel out the common factors from the numerator and the denominator.

Question 1. Divide:

  1. 24xy2z3 by 6yz2
  2. 63a2b4c6 by 7a2b2c3

Solution:

1.  24xy2z3+ 6yz2

⇒ \(\frac{2 \times 2 \times 2 \times 3 \times x \times y \times y \times z \times z \times z}{2 \times 3 \times y \times z \times z}\)

⇒ \(\frac{2 \times 2 \times x \times y \times z}{1}=4 x y z\)

2.  63a2b4c6 + 7a2b2c3

⇒ \(\frac{3 \times 3 \times 7 \times a \times a \times b \times b \times b \times b \times c \times c \times c \times c \times c \times c}{7 \times a \times a \times b \times b \times c \times c \times c}\)

⇒ \(\frac{3 \times 3 \times b \times b \times c \times c \times c}{1}\)

⇒ \(=9 b^2 c^3\)

Division Of A Polynomial By A Monomial

We divide each term of the polynomial in the numerator by the monomial in the denominator.

Division Of Algebraic Expressions Continued (Polynomial + Polynomial)

We factorize the algebraic expressions in the numerator and the denominator into irreducible factors and cancel the common factors from the numerator and the denominator.

Factorization Exercise 12.3

Question 1. Carry out the following divisions:

  1. 28x4 + 56x
  2. – 36y3 9y2
  3. 66pq2r3 + llqr2
  4. 34x2y2z2 – 51xy2z2
  5. 12a8b8 + (-6a6b4).

Solution:

1.  28x2 + 56x

⇒ \(\frac{28 x^4}{56 x}\)

⇒ \(\frac{2 \times 2 \times 7 \times x \times x \times x \times x}{2 \times 2 \times 2 \times 7 \times x}\)

⇒ \(=\frac{x \times x \times x}{2}=\frac{x^3}{2}\)

2.  – 36y3 + 9y2

⇒ \(\frac{-36 y^{\circ}}{9 y^2}\)

⇒ \(\frac{(-1) \times 2 \times 2 \times 3 \times 3 \times y \times y \times y}{3 \times 3 \times y \times y}\)

⇒ \(\frac{(-1) \times 2 \times 2 \times y}{1}=\frac{-4 y}{1}=-4 y\)

3.  66pq2P + llqr2

⇒ \(\frac{66 p q^2 r^5}{11 q r^2}\)

⇒ \(\frac{2 \times 3 \times 11 \times p \times q \times q \times r \times r \times r}{11 \times q \times r \times r}\)

⇒ \(\frac{2 \times 3 \times p \times q \times r}{1}=\frac{6 p q r}{1}=6 p q r\)

4.   34x3y3z3 + 5lxy2z3

\(\frac{34 x^3 y^3 z^3}{51 x y^2 z^3}\)

⇒ \(\frac{2 \times 17 \times x \times x \times x \times y \times y \times y \times z \times z \times z}{3 \times 17 \times x \times y \times y \times z \times z \times z}\)

⇒ \(\frac{2 \times x \times x \times y}{3}=\frac{2}{3} x^2 y\)

5. 12a8b8 + (-6a6b4).

⇒ \(\frac{12 a^8 b^8}{-6 a^6 b^4}\)

⇒ \(\frac{2 \times 2 \times 3 \times a \times a \times a \times a \times a \times a \times a \times a \times b \times b \times b \times b \times b \times b \times b \times b}{(-1) \times 2 \times 3 \times a \times a \times a \times a \times a \times a \times b \times b \times b \times b}\)

⇒ \(\frac{-2 \times a \times a \times b \times b \times b \times b}{1}\)

⇒ \(\frac{-2 a^2 b^4}{1}=-2 a^2 b^4 \)

Question 2. Divide the given polynomial by the given monomial:

  1. (5x2 – 6x) + 3x
  2. (3y8 – 4y6 + 5y4) + y4
  3. 8(x3y2z2 + x2y2 + x2y2Z2) 4x2y2z2
  4. (x2+ 2x2 + 3x) x 2x
  5. (P3q2 – P6q3) +p3q3

Solution:

1.  (5x2 – 6x) + 3x

⇒ \(=\frac{5 x^2-6 x}{3 x}=\frac{5 x^2}{3 x}-\frac{6 x}{3 x}=\frac{5}{3} x-2=\frac{1}{3}(5 x-6)\)

2.  (3y8 – 4y6 + 5y4) + y4

⇒ \(=\frac{3 y^8-4 y^6+5 y^4}{y^4}=\frac{3 y^8}{y^4}-\frac{4 y^6}{y^4}+\frac{5 y^4}{y^4}\)

– 3y4 – 4y2 + 5

3.  8(x3y2z2 + x2y2 + x2y2Z2) 4x2y2z2

⇒ \(\frac{8\left(x^3 y^2 z^2+x^2 y^3 z^2+x^2 y^2 z^3\right)}{4 x^2 y^2 z^2}\)

⇒ \(\frac{8 x^2 y^2 z^2(x+y+z)}{4 x^2 y^2 z^2}\) Taking (x? y2 z2) common

⇒ \(\frac{2(x+y+z)}{1}=2(x+y+z)\)

4.  (x2+ 2x2 + 3x) x 2x

⇒ \(\frac{x^3+2 x^2+3 x}{2 x}=\frac{x \times\left(x^2+2 x+3\right)}{2 \times x}\)

⇒ \(\frac{1}{2}\left(x^2+2 x+3\right)\)

5.  (P3q2 – P6q3) +p3q3

⇒ \(\frac{p^3 q^6-p^6 q^3}{p^3 q^3}=\frac{p^3 q^3\left(q^3-p^3\right)}{p^3 q^3}\)

⇒ \(\frac{q^3-p^3}{1}=q^3-p^3\)

Question 3. Work out the following divisions:

  1. (10x – 25) 5
  2. (10x – 25) 4- (2x – 5)
  3. 10y(6y + 21) + 5(2y + 7)
  4. 9x2y2(3z – 24) + 27xy(z – 8)
  5. 96abc(3a – 12) (5b – 30) + 144(a – 4) (b – 6).

Solution:

1. (10x – 25) – 5

⇒ \(\frac{10 x-25}{5}=\frac{5(2 x-5)}{5}\)

⇒ \(\frac{2 x-5}{1}=2 x-5\)

⇒ \((10 x-25) \div(2 x-5)\)

⇒ \(\frac{10 x-25}{2 x-5}=\frac{5(2 x-5)}{2 x-5}=\frac{5}{1}=5\)

2. 10 y(6 y+21)+5(2 y+7)

\(\frac{10 y(6 y+21)}{5(2 y+7)}=\frac{10 y \times 3(2 y+7)}{5(2 y+7)}\)

⇒ \(\frac{2 y \times 3}{1}=\frac{6 y}{1}=6 y\)

⇒ \(9 x^2 y^2(3 z-24)+27 x y(z-8)\)

⇒ \(\frac{9 x^2 y^2(3 z-24)}{27 x y(z-8)}=\frac{9 x^2 y^2 \times 3(z-8)}{27 x y(z-8)}\)

⇒ \( \frac{x y}{1}=x y\)

3. 96abc(3a – 12) (5b – 30) + 144(a – 4) (b – 6)

⇒ \(\frac{96 a b c(3 a-12)(5 b-30)}{144(a-4)(b-6)}\)

⇒ \(\frac{2 \times 2 \times 2 \times 2 \times 2 \times 3 \times a b c \times 3(a-4) \times 5(b-6)}{2 \times 2 \times 2 \times 2 \times 3 \times 3 \times(a-4)(b-6)}\)

|Taking 3 common in (3 a-12) and 5 common in $(5 b-30)

⇒ \(\frac{2 \times 5 \times a b c}{1}=\frac{10 a b c}{1}\)

Question 4. Divide as directed.

  1. 5(2x + 1) (3x + 5) -5- (2x + 1)
  2. 26xy(x + 5) (y – 4) * 13x(y – 4)
  3. 52pqr (p + q)(q + r)(r + p) + 104pq(q + r)(r + p)
  4. 20(y + 4) (y2 + 5y + 3) + 5(y + 4)
  5. x(x + l)(x + 2) (x + 3) + x(x + 1).

Solution:

1.  5(2x + 1) (3x + 5) + (2x + 1)

⇒ \(=\frac{5(2 x+1)(3 x+5)}{2 x+1}=\frac{5(3 x+5)}{1}\)

= 5(3x + 5)

2. 26xy(x + 5) (y – 4) + 13x(y – 4)

⇒ \(=\frac{26 x y(x+5)(y-4)}{13 x(y-4)}=\frac{2 y(x+5)}{1}\)

= 2y(x + 5)

3.  52pqr (p + q) (q + r) (r + p) -s- 104pq(q + r) (r + p)

⇒ \(\frac{52 p q r(p+q)(q+r)(r+p)}{104 p q(q+r)(r+p)}\)

⇒ \(\frac{2 \times 2 \times 13 \times p q r(p+q)(q+r)(r+p)}{2 \times 2 \times 2 \times 13 \times p q(q+r)(r+p)}=\frac{1}{2} r(p+q)\)

4.  20(y + 4) (y2 + 5y + 3) + 5(y + 4)

⇒ \(\frac{20(y+4)\left(y^2+5 y+3\right)}{5(y+4)}\)

⇒ \(\frac{2 \times 2 \times 5 \times(y+4)\left(y^2+5 y+3\right)}{5 \times(y+4)}=\frac{2 \times 2 \times\left(y^2+5 y+3\right)}{1}\)

= 4(y2 + 5y + 3)

5.  x(x + 1) (x + 2) (x + 3) + x(x + 1)

⇒ \(=\frac{x(x+1)(x+2)(x+3)}{x(x+1)}=\frac{(x+2)(x+3)}{1}\)

= (x + 2) (x + 3)

Question 5. Factorize the expressions and divide them as directed,

  1. ( y2 + 7y + 10) + (y + 5)
  2. (m2 – 14m – 32) + (m + 2)
  3. (5p2 – 25p + 20) + (p – 1)
  4. 4yz(z2 + 6z – 16) + 2y(z + 8)
  5. 5pq(p2 – q2) + 2p(p + q)
  6. 12xy(9x2 – 16y2) + 4xy(3x + 4y)
  7. 39y3(50y2 – 98) + 26y2(5y + 7)

Solution:

1.  ( y2 + 7y + 10) + (y + 5)

⇒ \(\frac{y^2+7 y+10}{y+5}=\frac{y^2+2 y+5 y+10}{y+5}\)

Using Identity IV; 2y + 5y = 7y; (2y) (5y) = 10/

⇒ \(\frac{y(y+2)+5(y+2)}{y+5}=\frac{(y+2)(y+5)}{y+5}\)

Taking (y + 2) common

⇒ \(\frac{y+2}{1}=y+2\)

2.  (m2 – 14m – 32) + (m + 2)

⇒ \(\frac{m^2-14 m-32}{m+2}=\frac{m^2-16 m+2 m-32}{m+2}\)

⇒ \(\frac{m(m-16)+2(m-16)}{m+2}=\frac{(m-16)(m+2)}{m+2} \quad \text { |Taking }(m-16) \text { common }\)

⇒ \(\frac{m-16 m+2 m=-14 m ; \quad(-16 m)(2 m)=-32 m^2}{}=m-16\)

3.  (5p2 – 25p + 20) + (p – 1)

⇒ \(\frac{5\left(p^2-5 p+4\right)}{p-1}=\frac{5\left(p^2-p-4 p+4\right)}{p-1}\)

\(\frac{5\{p(p-1)-4(p-1)\}}{p-1}=\frac{5(p-1)(p-4)}{p-1}\)

⇒ \(\frac{5(p-4)}{1}=5(p-4)\)

4.  4yz(z2 + 6z – 16) + 2y(z + 8)

⇒ \(\frac{4 y z\left(z^2+6 z-16\right)}{2 y(z+8)}=\frac{2 \times 2 \times y z\left(z^2+6 z-16\right)}{2 \times y(z+8)}\)

⇒ \(\frac{2 z\left(z^2+6 z-16\right)}{z+8}=\frac{2 z\left(z^2+8 z-2 z-16\right)}{z+8}\)

Using Identity IV: 8z – 2z = 6z; (8z) (- 2z) = – 16z2

⇒ \(\frac{2 z[z(z+8)-2(z+8)]}{z+8}\) Taking (z + 8) common

⇒ \(\frac{2 z(z+8)(z-2)}{z+8}=\frac{2 z(z-2)}{1}\)

= 2z (z – 2)

5.  5pq(p2 – q2) + 2p(p + q)

⇒ \(\frac{5 p q\left(p^2-q^2\right)}{2 p(p+q)}=\frac{5 p q(p+q)(p-q)}{2 p(p+q)}\) Using Identity III

⇒ \(\frac{5}{2} q(p-q)\)

6.  12xy(9x2 – 16y2) + 4xy(3x + 4y)

⇒ \(\frac{12 x y\left(9 x^2-16 y^2\right)}{4 x y(3 x+4 y)}=\frac{3\left(9 x^2-16 y^2\right)}{3 x+4 y}\)

⇒ \(\frac{3\left\{(3 x)^2-\left(4 y^2\right)\right\}}{3 x+4 y}\)

⇒ \(\frac{3(3 x+4 y)(3 x-4 y)}{3 x+4 y}\)

⇒ \(\frac{3(3 x-4 y)}{1}=3(3 x-4 y)\)

7.  39y3(50y2 – 98) + 26y2(5y + 7)

⇒ \(\frac{39 y^3\left(50 y^2-98\right)}{26 y^2(5 y+7)}\)

⇒ \(\frac{3 \times 13 \times y^3 \times 2 \times\left(25 y^2-49\right)}{2 \times 13 \times y^2(5 y+7)}\) I Taking 2 common

⇒ \(\frac{\left.3 y(5 y)^2-(7)^2\right)}{(5 y+7)}=\frac{3 y(5 y+7)(5 y-7)}{(5 y+7)}\) Using Identity III

⇒ \(\frac{3 y(5 y-7)}{1}=3 y(5 y-7)\)

Factorization Multiple-Choice Questions and Solutions

Question 1. The common factor of x2y2 and x3y3 is

  1. x2Y2
  2. x3y3
  3. x2y3
  4. x3y2

Solution: 1. x2Y2

x2y2= X X X X y X y

x3y3 = X X X X x X y X y X y

Question 2. The common factor of x2y2 and x4y is

  1. x4y2
  2. x2y2
  3. x3y2
  4. x3y

Solution: 4. x3y

x3y2= x x x x x y x y

x4y = x X x X X x x x y

Question 3. The common factor of a2m4 and a4m2 is

  1. a4m4
  2. a2m2
  3. a2m4
  4. a4m2

Solution: 2. a2m

a2m4 = a x a x m x m x m x m

a4m2 = a x a x a x a x m x m

Question 4. The common factor of p3q4 and p4q3 is

  1. p4q4
  2. p4q3
  3. p3q4
  4. p3q3

Solution: 3. p3q4

p3q4=p x p x p x q x q x qx q

p4q3 = p x p x p x p x q x q x q,

Question 5. The common factor 12y and 30 is

  1. 6
  2. 12
  3. 30
  4. 6y

Solution: 1. 6

12y = 2 x 2 x 3 x y

30 = 2 x 3 x 5

Question 6. The common factor of 2x, 3x3, 4 is

  1. 1
  2. 2
  3. 3
  4. 4

Solution: 1. 1

2x = 2 x x

3x3 = 3 x T x x x x

4 = 2 x 2.

Question 7. The common factor of 10ab, 30bc, 50ca is

  1. 10
  2. 30
  3. 50
  4. abc

Solution: 1. 10

10a6 = 2 x 5 x a x b

306c = 2 x 3 x 5 x b x c

50ca = 2 x 5 x 5 x c x a

Question 8. The common factor of 14a2b and 35a4b2

  1. a4b2
  2. 35a4b2
  3. 14a2b
  4. 7a2b

Solution: 4. 7a2b

14a2b = 2 x 7 x a x a x b

35a4b2 = 5 x 7 x a x a x a x a x a x b

Question 9. The common factor of 8a2b4c2, 12a4bc4 and 20a3b4 is

  1. a4b4
  2. a2b2
  3. 4a2b2
  4. 4a2b.

Solution: 4. 4a2b.

8a2b4c2 = 2 x 2 x 2 x a x a x b x b x b x b x c x c

12a4bc2 = 2 x 2 x 3 x a X a X a x a x b x c x c

20a3b4 = 2 x 2 x 5 x a x a x a x b x b x b x b

Question 10. The common factor of 6 a3b4c2, 21a2b and 15a3 is

  1. 3a2
  2. 3a3
  3. 6a3
  4. 6a3

Solution: 1. 3a2

6a3b4c3 = 2 x 3 x a’x o x a x b x b x t x 6 x c x c

21a2b = 3 x 7 x o x a x 6

15a3 = 3 x 5 x a x a x a

Question 11. The common factor of 2a2b4c2, 8a4b3c4 and 6a3b4c2 is

  1. 2a2b3c2
  2. 6a2b3c2
  3. 8a2b3c2
  4. a4b4c4

Solution: 1. 2a2b3c2

2a2b4c2 = 2 x a x a X b X b X b X b X c x c

8a4b3c2 = 2 x 2 x 2 x 0 x a x n x a x b x b x b X c X c X c X c

6a3b4c2 = 2 X 3 X a x a X a X b X b X b X b X c X C

Question 12. The common factor of 3a2b4c2, 12b2c4 and 15a3b4c4 is

  1. b4c4
  2. 3b2c2
  3. 15b2c2
  4. 12b2c4

Solution: 2. 3b2c2

3a3b4c3 = 3 x o x a x b x b x 6 x 6 x c x c

12b3c4 = 2 x 2 x 3 x b x. b xcxcxcxc

15a3b4c4 = 3 x 5 x a x a x a x b x b x b x b X c x c x c x c.

Question 13. The common factor of 3a2b4c2, 12b3c4 and 15a3b4b4 is

  1. 12x3
  2. 24x3
  3. 36x3
  4. 48x3

Solution: 1. 12x3

243y4 = 2 x 2 x 2 x 3

x x x x x x x y x y x y x y

36x4z4 = 2 x 2 x 3 x 3 X X X X X X X X X z X z X z X z

48x3y2z = 2 x 2 x 2 x 2 x 3 x x x x x x x y x y x z.

Question 14. The common factor of 72x3y4z4, 120z2d4x4 and 96y3z4d4 is

  1. 96z3
  2. 120z3
  3. 72z3
  4. 24z3

Solution: 4. 24z3

72x3y4z4 = 2 x 2 x 2 x 3 x 3 x x x x x x X y X y X y X y x z x z x z x z

120z2d4x4 = 2 x 2 x 2 x 3 x 5 x z x z x d x d x d x d x x x x x x x x

96y3z4d4 = 2 x 2 x 2 x 2 x 2 x 3 x y X y X y x z X z x z X z x d x d x d x d,

Question 15. The common factor of 36p2q3x4, 48pq3x2, and 54p3q3x4 is

  1. 6pq3x2
  2. 36pq3x2
  3. 54pq3x2
  4. 48pq3x2

Solution: 1. 6pq3x2

36p2q3x4 = 2 x 2 x 3 x 3 x p x p x q x q x q x x x x x x x x

48pq3x2= 2 X 2 X 2 X 2 X 3 x p x q x q x q x x x x

P3q4x4 = p x p x p x q x q x q x x x x x x x x.

Question 16. The factorisation of 12a2b + 15ab2 is

  1. 3ab(4a + 5b)
  2. 3a2b(4a + 5b)
  3. 3ab2(4a + 5b)
  4. 3a2b2(4a + 5b).

Solution: 1. 3ab(4a+ 5b)

12a2b + 16ab2 = 3ab(4a + 5b)

Question 17. The factorisation of 10x2 – 18x3 + 14x4 is

  1. 2×2(7x2 – 9.x + 5)
  2. 2x(7x2 – 9x + 5)
  3. 2(7x2 – 9x + 5)
  4. 2×3(7x2 – 9x + 5).

Solution: 1. 2x2(7x2 – 9x + 5)

10x2 – 18x3+ 14x4 = 2x2 (6 – 9x + 7x2).

Question 18. The factorisation of 6x – 42 is

  1. 6(x – 7)
  2. 3(x – 7)
  3. 2(x – 7)
  4. 6(x + 7)

Solution: 1. 6(x – 7)

6x – 42 = 6(x – 7)

Question 19. The factorisation of 6x + 12y is

  1. 14a3b3(2b2 – 3a2)
  2. 3(3x + 4y)
  3. 2(3x + 12y)
  4. none of these.

Solution: 1. 14a3b3(2b2 – 3a2)

6x + 12y = 6(x + 2y)

Question 20. The factorization of 28a3b2 – 42a2b3 is

  1. 14a3b3(2b2 – 3a2)
  2. 14a2b3(2b2 – 3a2)
  3. 14a3b2(2b2 – 3a2)
  4. none of these.

Solution: 1. 14a3b2(2b2 – 3a2)

28a3b3 – 42a2b3 = 14a3b3(2b2 – 3a2)

Question 21. The factorisation of a3 + a2b + ab2 is

  1. a(a2 + ab + b2)
  2. b(a2 + ab + b2)
  3. ab(a2 + ab + b2)
  4. none of these.

Solution: 1. a(a2 + ab + b2)

a3 + a2b + ab2 = a(a2 + ab+ b2)

Question 22. The factorisation of x2yz + xy2z + xyz2 is

  1. XYZ(x + y + z)
  2. x2yz(x + y + z)
  3. xy2z(x + y + z)
  4. xyz2(x + y + z).

Solution: 1. xyz(x + y + z)

x2yz + xy2z + xyz2 = xyz (x+ y + z)

Question 23. The factorisation of ax2y + bxy2 + cxyz is

  1. xy(ax + by + cz)
  2. axy(ax + by + cz)
  3. bxy(ax + by + cz)
  4. cxy(ax + by + cz).

Solution: 1. xy(ax + by + cz)

ax2y + bxy2 + XYZ

= xy (ax + by + cz)

Question 24. The factorisation of a (x + y + z) + b(x + y + z) + c(x + y + z) is

  1. (a + b + c)(x + y + z)
  2. (ab + be + ca)(x + y + z)
  3. (xy + yz + zx)(a + b + c)
  4. none of these.

Solution: 1. (a + b + c)(x + y + z)

a(x + y + z) + b(x + y + z) + c(x + y + z)

= (x + y + z) (a + b + c)

Question 25. The factorisation of 6xy – 4y + 6 – 9xis

  1. (3x – 2)(2y – 3)
  2. (3x + 2)(2y – 3)
  3. (3x – 2)(2y + 3)
  4. (3x + 2)(2y + 3).

Solution: 1. (3x – 2)(2y – 3)

6xy – 4y + 6 – 9x

= 2y(3x – 2) – 3(-2 + 3x)

= (3x – 2)(2y – 3)

Question 26. The factorisation of x2 + xy + 2x + 2y is

  1. (x + 2)(x + y)
  2. (x + 2)(x – y)
  3. (x – 2)(x + y)
  4. (x – 2)(x – y).

Solution: 1. (x + 2)(x + y)

x2 + xy + 2x + 2y

= x(x + y) + 2(x + y)

= (x + 2) (x + y)

Question 27. The factorization of ax + bx – ay – by is

  1. (x – y)(a + b)
  2. (x + y)(o + b)
  3. (x – y)(a – b)
  4. (x + y)(a – b).

Solution: 1. (x – y)(a + b)

ax + bx – ay – by

= x(a. + b) – y(a + b)

= (x – y)(a + b)

Question 28. The factorization of ab – a – b + 1 is

  1. (a – 1)(b – 1)
  2. (a + 1)(6 + 1)
  3. (a – 1)(6 + 1)
  4. (a + 1)(6 – 1)

Solution: 1. (a – 1)(b – 1)

ab – a – b + 1

= a(b – 1) – 1(b – 1)

= (a – l)(b – 1).

Question 29. The factorisation of x2 + x + xy + y + zx + z is

  1. (x + y + z)(x + 1)
  2. (x + y + z)(x + y)
  3. (x + y + z)(y + z)
  4. (x + y + z)(z + x).

Solution: 1. (x + y + z)(x + 1)

x2+ x + xy + y + zx + z

= x(x + 1) + y(x + 1) + z(x + 1)

= (X + 1)(x + y + z)

Question 30. The factorisation of x2y2 + xy + xy2z + yz + x2yz + xz is

  1. (xy + yz + zx)(xy + 1)
  2. (xy + yz + zx)(yz + 1)
  3. (xy + yz + zx)(zx + 1)
  4. none of these.

Solution: 1. (xy + yz + zx)(xy + 1)

x2y2 + xy + xy2z + yz + x2yz + xz

= xy(xy + 1) + yz(xy + 1) + Zx(xy + 1)

= (xy + yz + zx)(xy + 1).

Question 31. The factorisation of x2 + 8x + 16 is

  1. (x + 2)2
  2. (x + 4)2
  3. (x – 2)2
  4. (x – 4)2

Solution: 2. (x + 4)2

x2 + 8x + 16

= (x)2 + 2 (x)(4) + (4)2 = (x+ 4)2.

Question 32. The factorisation of 4y2 – 12y + 9 is

  1. (2y + 3)2
  2. (2y – 3)2
  3. (3y + 2)2
  4. (3y- 2)2

Solution: 2. (2y – 3)2

4y2 – 12y + 9

= (2y)2 – 2(2y)(3) + (3)2

= (2y – 3)2

Question 33. The factorisation of 49p2 – 36 is

  1. (7p + 6)(7p – 6)
  2. (6p + 7)(6p – 7)
  3. (7p + 6)2
  4. (Ip – 6)2

Solution: 1. (Ip + Q)(lp – 6)

49p2 – 36

= (7p)2 – (6)2 = (7p – 6)(7p + 6)

Question 34. The factorisation of y2 – 7y + 12 is

  1. (y + 3)(y + 4)
  2. (y + 3)(y – 4)
  3. (y – 3)(y + 4)
  4. (y – 3)(y – 4).

Solution: 4. (y – 3)(y – 4).

y2– 7y + 12

= y2 – 3y – 4y + 12

= y(y – 3) – 4(y – 3)

= (y – 3)(y – 4)

Question 35. The factorisation of z2 -4z – 12 is

  1. (z + 6)(z + 2)
  2. (z – 6)(z – 2)
  3. (z – 6)(z + 2)
  4. (z + 6)(z – 2).

Solution: 3. (z – 6)(z + 2)

z2 – 4z – 12

= z2 – 6z + 2z – 12

= z(z – 6) + 2(z – 6)

= (z – 6)(z + 2).

Question 36. The factorisation of am2+ bm2 + bn2 + an2 is

  1. (a + b)(m2 – n2)
  2. (a + b)(m2 + n2)
  3. (a – b)(m2 + n2)
  4. (a – b)(m2 – n2).

Solution: 2. (a + b)(m2 + n2)

am2 + bm2 + bn2 + an2

= m2(a + b) + n2(b + a)

= (a + b)(m2 + n2).

Question 37. The factorisation of (Im + t) + m + 1 is X

  1. (l + l)(m + 1)
  2. (l – 1)(m – 1)
  3. (l + l)(m – 1)
  4. (l – 1)(m + 1).

Solution: 1. (l + 1)(m + 1)

Im + l + m + 1

= l(m + 1) + l(m + 1)

= (l + 1)(m + 1)

Question 38. The factorization of (I + m)2 – 4lm is

  1. (l- m)2
  2. (l+m-2)2
  3. (l + m + 2)2
  4. none of these

Solution: 1. (l- m)2

(I + m)2 – 4Im

= l2 + m2+ 2Im – 4lm

= l2– 2Im + m2 = (l- m)2

Question 39. The factorisation of 1 + p + q + r + pq + qr + pr + pqr is

  1. (1 +p)(1 + q)(1 + r)
  2. (1 – p)1- q)(1- r)
  3. (1 – p)(1 – q)(1 + r)
  4. (1 + p)(1 – q)(1 – r).

Solution: 1.(l +p)(1 + q)(1 + r)

(1 + p) + q + r + pq + qr + pr + pqr = 1 + p + q + pq + r (1 + p + q + pq)

= (1 + r)(1 + p + q + pq)

= (1 + r)(1 + p)(l + q).

Question 40. The value of 0.645 x 0.645 + 2 x 0.645 x 0.355 + 0.355 x 0.355 is

  1. l
  2. 0
  3. -l
  4. 2

Solution: 1. 1

Value = (0.645 + 0.355)2 = (l)2 = 1.

Question 41. The factorisation of 1 + 16x + 64×2 is

  1. (l – 8x)2
  2. (l + 8x)2
  3. (8 – x)2
  4. (8 + x)2

Solution: 2. (1 + 8x)2

1 + 16x + 64X2

= (l)2 + 2(1) (8x) + (8x)2 = (1 + 8x)2

Question 42. The factorisation of a:2 + x + \(\frac{1}{4}\) is

  1. \(\left(\frac{x}{2}-1\right)^2\)
  2. \(\left(\frac{x}{2}+1\right)^2\)
  3. \(\left(x+\frac{1}{2}\right)^2\)
  4. \(\left(x-\frac{1}{2}\right)^2.\)

Solution: 3. \(\left(x+\frac{1}{2}\right)^2\)

⇒ \( x^2+x+\frac{1}{4}=x^2+2(x)\left(\frac{1}{2}\right)+\left(\frac{1}{2}\right)^2\)

⇒ \(\left(x+\frac{1}{2}\right)^2\)

Question 43. The value of 992 is

  1. (90)2 + 2(90)(9) + (9)2
  2. (90)2 _ 2(90)(9) + (9)2
  3. (90)2 + (9)2
  4. None of these

Solution: 1. (90)2 + 2(90)(9) + (9)2

992 = (90 + 9)2

= (90)2 + 2(90)(9) + (9)2

Question 44. The value of 492 is

  1. (50)2– 2(50)(1) + (l)2
  2. (50)2 + 2 (50) (1) + (l)2
  3. (50)2 -(1)2
  4. (50)2+ (1)2

Solution: 1. (50)2- 2(50)(1) + (1)2

492 = (50 – 1)2

= (50)2 _ 2(50)(l) + (1)2

Question 45. The factorisation of\(\left(\frac{x^2}{y^2}-2+\frac{y^2}{x^2}\right)\) x – 0, y * 0 is

  1. \(\left(\frac{x}{y}+\frac{y}{x}\right)^2\)
  2. \(\left(\frac{x}{y}-\frac{y}{x}\right)^2\)
  3. \(\left(\frac{x}{y}-1\right)^2\)
  4. \(\left(\frac{x}{y}+1\right)^2.\)

Solution: 2. \(\left(\frac{x}{y}-\frac{y}{x}\right)^2\)

⇒ \(\frac{x^2}{y^2}-2+\frac{y^2}{x^2}\)

⇒ \(\left(\frac{x}{y}\right)^2-2\left(\frac{x}{y}\right)\left(\frac{y}{x}\right)+\left(\frac{y}{x}\right)^2 \)

⇒ \(\left(\frac{x}{y}-\frac{y}{x}\right)^2\)

Question 46. The value of \( \frac{0.73 \times 0.73-0.27 \times 0.27}{0.73-0.27} is\)

  1. 1
  2. 0
  3. 0.73
  4. 0.27

Solution: 1. 1

Value= \(\frac{(0.73+0.27)(0.73-0.27)}{0.73-0.27}\) = 1

Question 47. The factorisation of x2 – 9 is

  1. (x – 3)2
  2. (x + 3)2
  3. (a + 3)(a – 3)
  4. None of these

Solution: 3. (a + 3)(a – 3)

x2 – 9 = (x)2 – (3)2 = (x – 3) (x + 3).

Question 48. The factorisation of 36xy – 1 is

  1. (6xy – 1)(6xy + 1)
  2. (6xy – 1)2
  3. (6xy + 1)2
  4. ( 6 + xy)2

Solution: 1. (6xy – 1)(6xy + 1)

36x2y2 – 1 = (6xy)2 – (1)2

= (6xy – 1)(6xy + 1).

Question 49. The value of \(\frac{0.564 \times 0.564-0.436 \times 0.436}{0.564-0.436} \text { is }\)

  1. 0
  2. 1
  3. -1
  4. None of these

Solution: 2. 1

Value = \(\frac{(0.564+0.436)(0.564-0.436)}{0.564-0.436}\) = 1

Question 50. The value of (0.68)2 – (0.32)2 is

  1. -l
  2. 0
  3. 1
  4. 0.36.

Solution:  4. 0

Value = (0.68 + 0.32) (0.68 – 0.32) = 0.36.

Question 51. The factorisation of 3x2 + 10x + 8 is

  1. (3x + 4)x + 2)
  2. (3x – 4)(x – 2)
  3. (3x + 4)(x – 2)
  4. (3x – 4)(x + 2).

Solution: 1. (3x + 4)(x + 2)

3X2 + lOx + 8

= 3X2 + 6x + 4x + 8

= 3x(x + 2) + 4(x + 2)

= (x + 2)(3x + 4).

Question 52. The factorisation of 3a2 – 16a + 16 is

  1. (x – 4)(3a – 4
  2. (a + 4)(3a + 4)
  3. (a – 4)(3a + 4)
  4. (a + 4)(3a – 4).

Solution: 1. (x – 4)(3a – 4)

3×2 – 16x + 16

= 3s2 – 12x – 4x + 16

= 3x(x – 4) – 4(x – 4)

= (x – 4) (3x – 4).

Question 53. The factorisation of 6a2 – 5a – 6 is

  1. (2a – 3)(3a + 2)
  2. (2a + 3)(3a + 2)
  3. (2a – 3)(3a – 2)
  4. (2a + 3)(3a – 2).

Solution: 1. (2a – 3)(3a + 2)

6X2 – 5x – 6

= 6×2 – 9x + 4x – 6

= 3x(2x – 3) + 2(2x – 3)

= (2x – 3)(3x + 2).

Question 54. The factorisation of 6 – a – 2a2 is

  1. (2 + a)(3 – 2a)
  2. (2 + a)(3 + 2a)
  3. (2 – a)(3 – 2a)
  4. (2 – a)(3 + 2a).

Solution: 1. (2 + a)(3 – 2a)

6 – x – 2x2

= 6 + 3x – 4x – 2x2

= 3(2 + x) – 2x (2 + x)

= (2 + x)(3 – 2x)

Question 55. If x2 – x – 42 = (x + k)(x + 6), then k =

  1. 6
  2. -6
  3. 7
  4. -7

Solution: 4. -7

x2 – x – 42

= x2 – 7x + 6x – 42

= x(x — 7) + 6(x — 7)

= (x – 7)(x + 6)

= (x + k) (x + 6)

k = -7

Question 56. The value of 3.5 x 3.5 – 2.5 x 2.5 is

  1. -6
  2. 6
  3. 60
  4. 1

Solution: 2. 6

Value = (3.5 + 2.5)(3.5 – 2.5) = 6

Question 57. If \(\left(x-\frac{1}{x}\right)^2=x^2+a+\frac{1}{x^2}\) then a=

  1. -2
  2. 2
  3. 2x
  4. -2x

Solution: 1. -2

⇒ \(\left(x-\frac{1}{x}\right)^2 =x^2-2+\frac{1}{x^2}\)

⇒ \(x^2+a+\frac{1}{x^2} a=-2 .\)

Question 58. If x = 2, y = -1, then the value of x2 + 4xy + 4y2 is

  1. 0
  2. 1
  3. -1
  4. 2

Solution: 1. 0

x2 + 4xy + 4y2

= (x)2 + 2(x)(2y) + (2y)2

– (x + 2y)2 = {2 + 2(- l)}2 = 0

Question 59. The quotient of 28x2 ÷ 14r is

  1. 2
  2. 2x
  3. x
  4. x2

Solution: 2. 2x

⇒ \(\frac{28 x^2}{14 x}=\frac{2 \times 2 \times 7 \times x \times x}{2 \times 7 \times x}=2 x .\)

Question 60. The quotient of 12a8b8 ÷ (- 4a6b6) is

  1. 3a2b2
  2. 3a2b
  3. 3ab2
  4. -3a2b2

Solution: 4. -3a2b2

⇒  \(-\frac{12 a^8 b^8}{4 a^6 b^6}\)

⇒  \(2 \times 2 \times 3 \times a \times a \times a \times a \times a \times a \times a \times a \)

⇒  \(-\frac{\times b \times b \times b \times b \times b \times b \times b \times b}{2 \times 2 \times a \times a \times a \times a \times a \times a} \times b \times b \times b \times b \times b \times b\)

= – 3a2b2

Factorization True-False

Write whether the following statements are True or False:

1. An equation is true for all values of its variables: False

2. The difference of the square of two consecutive natural numbers is equal to their sum: True

3. An identity is true for all values of its variables: True

4. (x + 1) (x – 1) (x2 + 1) = x4 + 1: False

5. The difference between the areas of the two squares with sides 5o and 5b is 25 (a – b) (a + b): True

Factorization Fill In The Blanks

1. The name of the property a (b + c) = ab + ac is: Distributive Property

2. The greatest common factor of 5a. and 151) is: 5

3. The common factor of 2xy and 3zt is: 1

4. The quotient obtained on dividing (x2 – 1) (x – 2) by – (x – 2) is:- (x2 – 1)

5. On dividing (x4+ y4) (x – y) by (x – y), the remainder is: 0

6. Find the value of an in 9a = 502 – 412: 91

7. Ifa + b = 10 and a2 + b2 = 44, then find ab: 28

8. Factorise x3 – 64x: x (x – 8) (x + 8)

9. x – \(\frac{1}{x}\) = 5, then find the value of \(x^2+\frac{1}{x^2}\): 27

10. Simplify : (a + b)2 + (a – b)2: . 2 (a2 + b2)

NCERT Solutions For Class 8 Maths Chapter 13 Introduction To Graphs

Introduction To Graphs Introduction

The purpose of the graph is to show numerical facts in visual form for their better and quicker understanding. It is especially very useful when there is a trend or comparison to be shown.

A Line Graph

It is a graph which displays data that changes continuously over periods.

Question What is the information that you gather from the given histogram? Try to list them out.
Solution:

This histogram illustrates the distribution of weights (in kg) of 40 persons in a locality. The information that we gather from this histogram is as follows :

A maximum number of persons in the locality have their weights (in kg) in the interval 50-55.

The minimum number of persons in the locality have their weights (in kg) in the interval 40- 45

Read and Learn More NCERT Solutions For Class 8 Maths

NCERT Class 8 Maths Chapter 13 Introduction To Graphs Number Of Persons

Introduction To Graphs Exercise 13.1

Question 1. The following graph shows a patient’s temperature in a hospital, recorded every hour.

  1. What was the patient’s temperature at 1 p.m.?
  2. When was the patient’s temperature 38.5°C?
  3. The patient’s temperature was the same two times during the period given. What were these two times?
  4. What was the temperature at 1.30 p.m.? How did you arrive at your answer?
  5. During which period did the patient’s temperature show an upward trend?

NCERT Class 8 Maths Chapter 13 Introduction To Graphs The Patient’s Temperature

Solution:

The patient’s temperature at 1 p.m. was 36.5°C.

The patient’s temperature was 38.5°C at 12 noon

The two times when the patient’s temperature was the same were 1 p.m. and 2 p.m.

The temperature at 1.30 p.m. was 36.5°C.

From the graph, we see that the temperature was constant from 1 p.m. to 2 p.m. Since 1.30 p.m. comes in between 1 p.m. and 2 p.m., therefore we arrived at our answer.

The patient’s temperature showed an upward trend during the periods 9 a.m. to 10 a.m., 10 a.m. to 11 a.m. and 2 p.m. to 3 p.m.

Question 2. The following line graph shows the yearly sales figures for a manufacturing company.

  1. What were the sales in
    1. 2002
    2. 2006?
  2. What were the sales in
    1. 2003
    2. 2005?
  3. Compute the difference between the y sales in 2002and 2006.
  4. In which year was the greatest difference between the sales compared to the previous year?

NCERT Class 8 Maths Chapter 13 Introduction To Graphs The Greatest Scales

Solution:

The sales in

  1. 2002 were ₹ 4 crores and in
  2. 2006 were ₹ 8 crores.

The sales in

  1. 2003 were ₹ 7 crores and in
  2. 2005 were ₹ 10 crores.

The difference between the sales in 2002 and 2006 = ₹ 8 crores – ₹ 4 crores = ₹ 4 crores

The difference between the sales in 2002 and 2003 =₹ 7 crores – ? 4 crores =₹ 3 crores

The difference between the sales in 2003 and 2004=₹ 7 crores – ₹ 6 crores = ₹ 1 crore

The difference between the sales in 2004 and 2005 =₹ crores – ₹ 6 crores = ₹ 4 crores

The difference between the sales in 2005 and 2006

= ₹ 10 crores – ₹ 8 crores = ₹ 2 crores

Therefore, in the year 2005, the difference between the sales as compared to the previous year was the greatest.

Question 3. For an experiment in Botany, two different plants, plant A and plant B were grown under similar laboratory conditions. Their heights were measured at the end of each week for 3 weeks. The results are shown in the following graph.

NCERT Class 8 Maths Chapter 13 Introduction To Graphs The Experiment In Botany

  1. How high was Plant A after
    1. 2 weeks
    2. 3 weeks?
  2. (b) How high was Plant B after
    1. 2 weeks
    2. 3 weeks?
  3. How much did Plant A grow during the 3rd week?
  4. How much did Plant B grow from the end of the 2nd week to the end of the 3rd?
  5. During which week did Plant. A grow most?
  6. During which week did the Plant grow the least?
  7. Were the two plants of the same height during any week shown here? Specify.

Solution:

1.  The height of Plant A

  1. after 2 weeks was 7 cm high and
  2. after 3 weeks it was 9 cm high.

2.  The height of Plant B

  1. after 2 weeks it was 7 cm high and
  2. after 3 weeks it was 10 cm high.

3. Plant A grew 9 cm – 7 cm = 2 cm during the 3rd week.

4.  From the end of the 2nd week to the end of the 3rd week, Plant B grew = 10 cm – 7 cm = 3 cm.

5.  The Plant A grew in 1st week = 2 cm – 0 cm = 2 cm

The Plant A grew in 2nd week = 7 cm – 2 cm = 5 cm

The Plant A grew in 3rd week = 9 cm – 7 cm = 2 cm

Therefore, the Plant A grew mostly in Weeks second week

6.  The Plant B grew in 1st week = 1 cm – 0 cm = 1cm

The Plant B grew in 2nd week = 7 cm – 1 cm = 6 cm

The Plant B grew in 3rd week = 10 cm – 7 cm = 3 cm

Therefore, Plant B grew the least in the first week.

7.  At the end of the week, the two plants shown here were of the same height.

Question 4. The following shows the temperature forecast and the actual temperature for each day of the week:

NCERT Class 8 Maths Chapter 13 Introduction To Graphs The Temperature Fore Cast

  1. On which days was the forecast temperature the same as the actual temperature?
  2. What was the maximum forecast temperature during the week?
  3. What was the minimum actual temperature during the week?
  4. On which day did the actual temperature differ the most from the forecast temperature?

Solution:

  1. The forecast temperature was the same as the actual temperature on Tuesday, Friday and Sunday. This is indicated by the points where the two graphs intersect.
  2. The maximum forecast temperature during the week was 35°C.
  3. The minimum actual temperature during the week was 15°C.

NCERT Class 8 Maths Chapter 13 Introduction To Graphs The Forecast Temperature On Thursday

Since the maximum temperature difference is 7.5°C, therefore, the actual temperature filtered the most from the forecast temperature on Thursday.

Question 5. Use the tables below to draw linear graphs.

The number of days a hillside city received snow in different years

NCERT Class 8 Maths Chapter 13 Introduction To Graphs The Men And Women In A Village In Different Years

Population (in thousands) of men and women in a village in different years

NCERT Class 8 Maths Chapter 13 Introduction To Graphs The Number Of Days A Hill Side City

Solution:

NCERT Class 8 Maths Chapter 13 Introduction To Graphs The Linear Graph Showing Snow Fall In Different Year

NCERT Class 8 Maths Chapter 13 Introduction To Graphs Linear Graph Showing Population Of Men And Women

Question 6. A courier person cycles from a town to a neighbouring suburban area to deliver a parcel to a merchant. His distance from the town at different times is shown in the following graph.

  1. What is the scale taken for the time xis?
  2. How much time did the person take 4 for the trave?
  3. Flow far is the place of the merchant from the tow?
  4. Did the person slop on his way? Explain.
  5. During which period did he ride fast?

NCERT Class 8 Maths Chapter 13 Introduction To Graphs The Courier Person Cycles

Solution:

1.  The scale taken for the time axis (i.e., along the x-axis) is 4 units = 1 hour.

2.  Time of the start of the travel = 8 a.m.

Time of the end of the travel = 11.30 a.m.

The time taken by the person for the travel = Difference between 8 a.m. and 11.30 a.m. = 31/2 hours.

3.  The place of the merchant from the town is 22 km.

4.  Yes. This is indicated by the horizontal part of the graph (10 a.m. -10.30 a.m.)

Distance covered by the person between 8 a.m. and 9 a.m. = (10 – 0) km = 10 km

Distance covered by the person between 9 a.m. and 10 a.m. = (16- 10)=6 km

covered by km the person between 10 a.m. and 10: 30 a.m. = (16 – 16) km = 0 km

Distance covered by the person between 10.30 a.m. and 11.30 a.m. = (22 – 16) km = 6 km

He rides fastest between 8 a.m. and 9 a.m

Question 7. Can there be a time-temperature graph as follows? Justify your answer.

NCERT Class 8 Maths Chapter 13 Introduction To Graphs The Time-Temperature

NCERT Class 8 Maths Chapter 13 Introduction To Graphs The Time-Temperature Increase

NCERT Class 8 Maths Chapter 13 Introduction To Graphs The Time Temperature

Solution:

  1. Yes, it can be It is a time-temperature graph because it shows an increase in temperature with an increase in time. Yes, it can be
  2. it is a time-temperature graph because it shows a decrease in temperature with an increase in time.
  3. It cannot be a time-temperature graph because it shows infinitely many different temperatures at one particular time which is not possible.
  4. Yes, it can be It is a time-temperature graph because it shows a fixed temperature at different times (or increasing times)

Some Applications

In our daily life, we observe two quantities that are interrelated i.e., the change in one quantity is accompanied by a change in the other quantity, for example: the more the number of days a labourer works, the more the wages he gets. More the sugar we purchase, more the amount we have to pay.

In the first, case, the number of days is called the independent variable (or control variable) and the wages are called the dependent variable. Similarly, in the second case, the amount of sugar is the independent variable whereas the money paid is the dependent variable.

Thus, an independent variable is a variable which can take values freely i.e., whose value is not dependent on any other variable whereas a dependent variable is a variable where value depends upon the other variable, i.e., independent variable.

The relation between the independent and dependent variables can be shown by a graph.

Question 1. The number of litres of petrol you buy to fill a car’s petrol tank will decide the amount you have to pay. Which is the independent variable here? Think about it
Solution:

The independent variable here is the amount of petrol we buy to fill a car’s petrol tank.

Question 2. In the given example use the graph to find how much petrol can be purchased for 800.
Solution:

Mark the point representing 800 the on y-axis. From this point, draw a line, parallel to the line graph at point, draw a line parallel to OY to ow cut OX at a point This point represents litres on the x-axis Hence, 16 litres of petrol can be purchased for 800

NCERT Class 8 Maths Chapter 13 Introduction To Graphs The Parallel To Line

Question Is Example a case of direct variation?
Solution:

It is a case of direct variation as

⇒  \(\frac{100}{10}=\frac{200}{20}=\frac{300}{30}=\frac{500}{50}=\frac{1000}{100}\) = 10 (constant)

Exercise 13.2

Question 1. Draw the graph for the following tables of values, with suitable scales on the axes.

1.  Cost of apples

NCERT Class 8 Maths Chapter 13 Introduction To Graphs The Cost Of Apples

2.  Distance travelled by car

NCERT Class 8 Maths Chapter 13 Introduction To Graphs The Deposit

  1. How much distance did the car cover during the period 7.30 a.m. to 8 a.m.?
  2. What was the time when the car had covered a distance of 1000 km since its star?

3.  Interest on deposits for a year

NCERT Class 8 Maths Chapter 13 Introduction To Graphs The Distance Travelled By A Car

  1. Does the graph pass through the origin?
  2. Use the graph to find the interest on 72500 for a year.
  3. To get an interest of f280 per year, how much money should be deposited?

Solution:

NCERT Class 8 Maths Chapter 13 Introduction To Graphs The side Of The Square

Scale:

On the horizontal axis: 2 units = 1 apple

On the  vertical axis: 1 unit = ? 5

Mark the number of apples on the horizontal axis.

Mark cost (in ?)the on vertical axis.

Plot, the points: (1, 5), (2, 10), (3, 15), (4, 20) and (5, 25).

Join the points.

We get a linear graph.

NCERT Class 8 Maths Chapter 13 Introduction To Graphs The horizontal axis

Scale :

On the  horizontal axis: 2 units = 1 hour

On the  vertical axis: 2 units = 40 km

Mark time (in hours)the on horizontal axis.

Mark distances (in km)the on vertical axis.

Plot the points (6 a.m., 40), (7 a.m., 80), (8 a.m., 120) and (9 a.m. 160).

Join the points.

We get a linear graph.

Distance tance covered during 7.30 a.m. to 8 a.m. = 120 km – 100 km = 20 km

The time when the car had covered a distance of 100 k Since its start was 7.30 a.m.

NCERT Class 8 Maths Chapter 13 Introduction To Graphs The Simple Interest

Scale :

On the horizontal axis: 2 units? 1000

On the vertical axis: 2 units  80

Mark deposit (in ?) on the horizontal axis.

Mark simple interest (in ?) on the vertical axis.

Plot the points (1000, 80), (2000, 160), (3000, 240), (4000, 320) and (5000, 400).

Join the points.

We get a linear graph.

Yes, The graph passes through the origin.

Interest o ? 2500 for a year = ? 200

To get an interest of? 280 per year? 3500 should be deposited.

Question 2. Draw a graph for the following:

NCERT Class 8 Maths Chapter 13 Introduction To Graphs The side Of The Squares

Is it a linear graph?

NCERT Class 8 Maths Chapter 13 Introduction To Graphs The Side Squares

Is it a linear graph?

Solution:

NCERT Class 8 Maths Chapter 13 Introduction To Graphs The Pass Through The Origin

Scale:

On the  horizontal axis : 1 unit = 1 cmOn thee  vertical axi : 1 unit = 4 cm

Mark the  side of the square (in cm) on the  horizontal axis

Mark the  perimeter (in cm) on vertical

Plot the points (2, 8), (3,12), (3.5, 14), (5, 20) and (6, 24).

Join the points.

Yes, It is a linear graph.

NCERT Class 8 Maths Chapter 13 Introduction To Graphs It Is a linear

Scale :

On the horizontal axis: 2 units = 2 cm

On the vertical axis: 1 unit = 2 cm

Mark the side of the square (in cm) on the horizontal axis.

Mark the area (in cm2) on the vertical axis.

Plot the points (2, 4), (3, 9), (4, 16), (5, 25) and (6, 36).

Join the points.

The graph we get is not linear.

Multiple-Choice Questions and Solutions

Observe the following bar graph and answer the related questions:

NCERT Class 8 Maths Chapter 13 Introduction To Graphs The Travelling Allowance

Question 1. On which head, is the expenditure maximum?

  1. Travelling allowance
  2. Rent
  3. Appliances
  4. Salary of employees.

Solution: 4. Salary of employees.

Question 2. On which head or head, is the expenditure minimum?

  1. Travelling allowance or rent
  2. Appliances
  3. Salary of employees
  4. Others.

Solution: 1. Travelling allowance or rent

Question 3. On which two heads, is the expenditure the same?

  1. Salary of employees and others
  2. Travelling allowance and rent
  3. Appliances and rent
  4. Appliances and others

Solution: 2. Travelling allowance and rent

Question 4. What is the difference in expenditure (in thousands of rupees) between the salary of employees and rent?

  1. 100
  2. 200
  3. 300
  4. 400

Solution: 3. 300

Question 5. What is the sum of the expenditures (in thousands of rupees) on travelling allowance and rent?

  1. 100
  2. 200
  3. 300
  4. 400

Solution: 2. 200

Observe the following circle graph and answer the related questions:

NCERT Class 8 Maths Chapter 13 Introduction To Graphs The Expenditure Maximum

Question 6. On which head is the expenditure maximum?

  1. Food
  2. Clothes
  3. House rent
  4. Education.

Solution: 1. Food

Question 7. On which head is the expenditure minimum

  1. Education
  2. Food
  3. House rent
  4. Clothes.

Solution: 1. Education

Question 8. If the budget of the family is ₹ 10800, what is the savings?

  1. ₹ 1050
  2. ₹ 950
  3. ₹ 1000
  4. ₹ 1200.

Solution: 1. ₹ 1050

Question 9. What is the difference in spending on clothes and education the budget of the family is ₹ 1000.

  1. ₹ 1200
  2. ₹ 800
  3. ₹ 1000
  4. ₹ 1500.

Solution: 1. ₹ 1200

Question 10. What is the sum of the expenditures on food and education if the budget of the family is ₹ 1080?

  1. ₹ 5000
  2. ₹ 5400
  3. ₹ 8000
  4. ₹ 6000

Solution: 3. ₹ 8000

Observe the following circle graph and answer the related questions:

NCERT Class 8 Maths Chapter 13 Introduction To Graphs The Class Interval

Question 11. In which class intervals, are the maximum number of students?

  1. 0-5
  2. 5-10
  3. 20-25
  4. 15-20

Solution: 3. 20-25

Question 12. In which class intervals, are the minimum number of students?

  1. 0-5
  2. 5-10
  3. 10-15
  4. 15-20

Solution: 1. 0-5

Question 13. In which class intervals, is the number of students 200?

  1. 0-5
  2. 5-10
  3. 20-25
  4. 15-20

Solution: 1. 0-5

Question 14. The difference in the number of students in class intervals 0-5 and 5- 10 is

  1. 100
  2. 200
  3. 300
  4. 400

Solution: 1. 100

Question 15. The sum of the number of students in the class intervals 10-15 and 20-25 is

  1. 800
  2. 900
  3. 600
  4. 400

Solution: 1. 800

Observe the following temperature-timeline graph and answer the related questions:

NCERT Class 8 Maths Chapter 13 Introduction To Graphs The Maximum Temperature - Copy

Question 16. At what time is the temperature maximum?

  1. 13 hours
  2. 15 hours
  3. 11 hours
  4. 19 hours

Solution: 1. 13 hours

Question 17. At what time(s) is the temperature minimum?

  1. 7 hours and 21 hours
  2. 9 hours
  3. 11 hours
  4. 13 hours.

Solution: 1. 7 hours and 21 hours

Question 18. 103°F temperature is the time

  1. 11 hours
  2. 13 hours
  3. 15 hours
  4. 21 hours.

Solution: 1. 11 hours

Question 19. What is the difference of temperatures at 7 hours and 21 hours?

  1. 0°F
  2. 1°F
  3. 2°F
  4. 3°F

Solution: 1. 0°F

Question 20. What is the rise in temperature from 11 hours to 13 hours?

  1. 1°F
  2. 2°F
  3. 3°F
  4. 4°F

Solution: 1. 1°F

Question 21. What is the fall in temperature from 13 hours to 21 hours?

  1. 2°F
  2. 3°F
  3. 4°F
  4. 6°F

Solution: 1. 2°F

Question 22. The coordinates of the origin are

  1. (0,0)
  2. (1,0)
  3. (0,1)
  4. (1,1)

Solution: 1. (0,0)

Question 23. What are the coordinates of a point whose .v-coordinate is 3 andy-coordinate is 4?

  1. (3,3)
  2. (3,4)
  3. (4,3)
  4. (4,3)

Solution: 2. (3,4)

Question 24. What are the coordinates of a point whose x-coordinate is 1 and y-coordinate is 0?

  1. (1,0)
  2. (0,0)
  3. (0,1)
  4. (1,1)

Solution: 1. (1,0)

Question 25. What are the coordinates of a point whose v-coordinate is 0 and whose y-coordinate is?

  1. (0,1)
  2. (0,0)
  3. (1,0)
  4. (1,1).

Solution: 1. (0,1)

Observe the following velocity-lime graph and answer the related questions:

NCERT Class 8 Maths Chapter 13 Introduction To Graphs The Rate Of Interest

Question 26. At what time is the velocity maximum?

  1. 7
  2. 8
  3. 9
  4. 10

Solution: 1. 6

Question 27. At what time is the velocity minimum?

  1. 8
  2. 9
  3. 10
  4. 11

Solution: 4. 11

Question 28. At what times are the velocities equal?

  1. 8 and 12
  2. 9 and 11
  3. 7 and 12
  4. 11 and 13.

Solution: 1. 8 and 12

Question 29. What is the fall in velocity from 7 to 1 1?

  1. 80 km/hour
  2. 90 km/hour
  3. 100 km/hour
  4. 20 km/hour.

Solution: 1. 80 km/hour

Question 30. What is the rise in velocity from 11 to 12?

  1. 10 km/hour
  2. 20 km/hour
  3. 30 km/hour
  4. 60 km/hour.

Solution: 4. 60 km/hour.

Observe the following runs-over graph and answer the related questions:

NCERT Class 8 Maths Chapter 13 Introduction To Graphs The Maximum Runs Of The Scored

Question 31. In which over is the maximum scoreboard scored?

  1. 2
  2. 4
  3. 5
  4. 6

Solution: 4. 6

Question 32. In which over are the maximum runs scored?

  1. 10
  2. 11
  3. 12
  4. 9

Solution: 3. 12

Question 33. What is the difference in runs scored in IV and V overs?

  1. 1
  2. 2
  3. 3
  4. 4

Solution: 1. 1

Question 34. What is the sum of runs scored in I and XII overs?

  1. 1
  2. 2
  3. 3
  4. 4

Solution: 1. 1

Question 35. 3 runs are scored in which over?

  1. 2 and 10
  2. 1 and 5
  3. 7 and 8
  4. 10 and 12

Solution: 3. 7 and 8

Read the graph and answer the related questions:

NCERT Class 8 Maths Chapter 13 Introduction To Graphs The Maximum Books

Question 36. In which year was the rate of interest maximum?

  1. 2005
  2. 2003
  3. 2006
  4. 2002.

Solution: 1. 2005

Question 37. In which year was the rate of interest minimum 

  1. 2006
  2. 2004
  3. 2001
  4. 2005.

Solution: 1. 2006

Question 38. The difference in the maximum and minimum rates of interests

  1. 2%
  2. 4%
  3. 6%
  4. 8%.

Solution: 4. 8%.

Question 39. The rise in interest from 2004 to 2005 was

  1. 2%
  2. 4%
  3. 6%
  4. 8%.

Solution: 2. 4%

Question 40. The fall in interest from 2001 to 2002 was

  1. l%
  2. 2%
  3. 3%
  4. 4%.

Solution: 2. 2%

Read the graph and answer the related questions :

NCERT Class 8 Maths Chapter 13 Introduction To Graphs The Students Appeared In The Year

Question 41. How many students appeared in the year 2000?

  1. 200
  2. 250
  3. 300
  4. 350.

Solution: 2. 250

Question 42. In which year did 50 students appear? 

  1. 1998
  2. 2001
  3. 1997
  4. 1996.

Solution: 4. 1996.

Question 43. In which year did the maximum number of students appear?

  1. 2002
  2. 2000
  3. 2001
  4. 1999.

Solution: 1. 2002

Question 44. What is the maximum number of students that appeared in any year?

  1. 350
  2. 300
  3. 250
  4. 300.

Solution: 1. 350

Question 45. In which two years was the number of students appearing the same?

  1. 1997 and 1998
  2. 1998 and 1999
  3. 1999 and 2000
  4. 2000 and 2001.

Solution: 1. 1997 and 1998

Read the graph and answer the related questions:

NCERT Class 8 Maths Chapter 13 Introduction To Graphs The Maximum Books

Question 46. The maximum books are of which subject?

  1. Hindi
  2. Science
  3. English
  4. Sanskrit.

Solution: 1. Hindi

Question 47. The minimum books are of which subject

  1. Home Science
  2. Sanskrit
  3. Science
  4. English.

Solution: 1. Home Science

Question 18. 500 books are of which subject 

  1. English
  2. Science
  3. Hindi
  4. Maths.

Solution: 4. Maths.

Question 49. How many books are on the subject of Home Science?

  1. 100
  2. 200
  3. 300
  4. 400

Solution: 1. 100

Question 50. How many books are there in Sanskrit and Home Science taken together?

  1. 100
  2. 200
  3. 300
  4. 400

Solution: 3. 300

Read the graph and answer the related questions:

NCERT Class 8 Maths Chapter 13 Introduction To Graphs The Number Of Labourers Maximum

Question 51. In which year was the number of labourers maximum?

  1. 2001
  2. 2002
  3. 2003
  4. 2004

Solution: 4. 2004

Question 52. In which year was the number of labourers minimum?

  1. 2003
  2. 2004
  3. 2005
  4. 2006

Solution: 4. 2006

Question 53. What was the difference in the number of labourers in the years 2002 and 203?

  1. 100
  2. 200
  3. 300
  4. 400

Solution: 2. 200

Question 54. Find the rise in the number of labourers from 2001 to 2004.

  1. 200
  2. 300
  3. 400
  4. 500

Solution: 3. 400

Question 55. Find the sum of the number of labourers in the years 2004 and 2006.

  1. 700
  2. 600
  3. 200
  4. 500

Solution: 1. 700

Read the circle graph and answer the related questions:

NCERT Class 8 Maths Chapter 13 Introduction To Graphs The 1000 students In A School

Question 56. There are in all 101000 students a school. The number of students in the class I are

  1. 500
  2. 250
  3. 125
  4. None of these

Solution: 1. 500

Question 57. The number of students in class II is

  1. 500
  2. 250
  3. 125
  4. 100

Solution: 2. 250

Question 58. In which two classes is the number of students the sesame 

  1. 1 and 2
  2. 1 and 3
  3. 3 and 4
  4. 1 and 4

Solution: 3. 3 and 4

Question 59. The minimum number of students in any class is

  1. 125
  2. 250
  3. 500
  4. 1000

Solution: 1. 125

Question 60. The sum of the number of students in class 3 and class 4 is

  1. 500
  2. 1000
  3. 50
  4. 250

Solution: 4. 250

True-False

Write whether the following statements are ‘D ue or False:

1. The coordinates of the origin are (0, 0): True

2. The .v-coordinate of a point is its distance from r-the axis: False

3. For fixing a point on the graph sheet, we need two coordinates: True

4. The relation between the pendent variable and the dependent variable is shown through a graph: True

Fill in the Blanks

1. Comparison of parts of a whole can be done by_____: pie chart

2. A point which lies on both these is_____: (0,0)

3. The r-coordinate of every point lying on the y-axis is_____: 0

4. The process of fixing a point with the help of x and y coordinates is called_____of the point: Plotting

5. What will you get after joining the points (-1, -1), (0, 0) and (3, 3): A straight line passing through the origin

6. What will you get after joining the points (1, 0), (1, 1) and (1, -1): A straight line not passing through the origin

7. What is the distance of the point (3, 4) from s-axis: 4

8. In the following graph, what are the coordinates of P: (3,2)

NCERT Class 8 Maths Chapter 13 Introduction To Graphs The Coordinates Of P

9. In the following graph, which letter indicates (the point (3, 0): A

NCERT Class 8 Maths Chapter 13 Introduction To Graphs Letter Ibdicates The Point

NCERT Solutions For Class 8 Maths Chapter 1 Rational Numbers

NCERT Solutions For Class 8 Maths Chapter 1 Rational Numbers Introduction

  • The counting numbers 1, 2, 3, 4,………… are called natural numbers.
  • The smallest natural number is 1. There is no greatest natural number.
  • If we add 0 to the collection of natural numbers, what we get is called the collection of whole numbers. are called natural numbers. are whole numbers.
  • Every natural number is a whole number but every whole number is not a natural number, e.g., 0 is a whole number but not a natural number.
  • The positive integers are the same as natural numbers.
  • We get negative integers if we put a negative sign before each positive integer. Thus, – 1, – 2, – 3, – 4,
  • The smallest positive integer is 1. The greatest negative integer is -1.
  • The number ‘O’ is neither positive nor negative. It is greater than every negative integer and smaller than every positive integer. It is simply an integer.
  • Thus, 0, 1, 2, 3, and 4, are negative integers.÷
  • Several p/q where p and q are integers and q ≠ 0 is called a rational number.
  • All the above types of numbers are needed to solve various kinds of simple algebraic equations  

Read and Learn More NCERT Solutions For Class 8 Maths

Properties Of Rational Numbers

The list of properties of rational Numbers can be given as follows :

  1. Closure
  2. Commutativity
  3. Associativity
  4. The role of zero
  5. The role of 1
  6. Negative (or additive inverse) of a number
  7. Reciprocal (or multiplicative inverse)
  8. Distributivity of multiplication over addition for rational numbers

Closure

Whole numbers

Question 1. 4+7 =________________ Is it a whole number
Solution:

4 + 7=11

Yes, It is a whole number.

Question 2. 3×7 = _______________Is it a whole number?
Solution:

3 × 7 = 21

Yes, It is a whole number

Question 3. Check for closure property under all four operations for natural numbers
Solution:

NCERT Solutions For Class 8 Maths Chapter 1 Rational Numbers Natural Numbers

Integers

Question 1. Is – 7 + (- 5) an integer?
Solution:

– 7 + (- 5)

– 7 + (- 5) = – 12

Yes,  Which is an integer.

Question 2. Is 8 + 5 an integer?
Solution:

8 + 5

8 + 5 = 13

Yes,  Which is an integer

Question 3. Is 5 – 7 an integer?
Solution:

5 – 7

5 – 7 = -2

Yes, Which is an integer.

Question 4. Is 8 – (- 6) an integer?
Solution:

8 – (- 6)

8 – (- 6) = 8 + 6

= 14,

Yes, Which is an integer.

Question 5. Is – 5 × 8= do integer?
Solution:

– 5 × 8

– 5 × 8= – 40

Yes,  Which is an integer

Rational numbers

Question 1.\(\frac{-3}{8}+\frac{(-4)}{5}\)= ________ Is it a rational number ?
Solution:

⇒ \(\frac{-3}{8}+\frac{(-4)}{5}\)

=\(\frac{-15+(-32)}{40}\)

= \(\frac{-47}{40}\)

Yes, It is a rational number

Question 2. \(\frac{5}{8}-\frac{4}{5}=\frac{25-32}{40}\) = _________________ it is a ralional number?
Solution:

⇒ \(\frac{5}{8}-\frac{4}{5}\)

= \(\frac{25-32}{40}\)

= \(\frac{-7}{40}\)

Yes, It is a rational number.

Question 3. \(\frac{-5}{9}+\frac{(-7)}{6}\) _______________ Is it a rational number?
Solution:

⇒ \(\frac{3}{7}-\left(\frac{-8}{5}\right)=\frac{3}{7}+\frac{8}{5}\)

= \(\frac{15+56}{35}\)

=\(\frac{71}{35}\) .

Yes, It is a rational number.

Question 4. \(\frac{4}{7}+\frac{6}{11}\) Is it a rational number ?
Solution:

⇒  \(\frac{4}{7}+\frac{6}{11}\)

= \(\frac{44+42}{77}=\frac{86}{77}\)

Yes, It is a rational number

Question 5. \({4}{5} \times \frac{-6}{11}\)___________Is it a rational number?
Solution:

⇒ \(\frac{4}{5} \times \frac{-6}{11}\)

= \(\frac{24}{55}\)

Yes, It is a rational number.

Question 6. \(\frac{2}{7}+\frac{5}{3}\)= _______________Is it a rational number?
Solution:

⇒ \(\frac{2}{7} \div \frac{5}{3}\)

=\(\frac{2}{7} \times \frac{3}{5}\)

= \(\frac{6}{35}\)

Yes, It is a rational number

Question 7. \(\frac{-3}{8}+\frac{-2}{9}\)= ______________ It is a rational numbers
Solution:

⇒ \(\frac{-3}{8} \div \frac{-2}{9}\)

= \(\frac{-3}{8} \times \frac{-9}{2}\)

= \(\frac{27}{16}\)

Yes, it is a rational number.

Question 8. Check it for a few more pairs of rational numbers

1. \(\frac{3}{5}+\frac{7}{13}\)
Solution: 

⇒ \(\frac{3}{5}+\frac{7}{13}\)

= \(\frac{39+35}{65}\)

= \(\frac{74}{65}\)

Rational number

2. \(\frac{3}{7}+\frac{(-4)}{9}\)
Solution:

⇒ \(\frac{3}{7}+\frac{(-4)}{9}\)

= \(\frac{27+(-28)}{63}\)

Rational number

3. \(\frac{-5}{9}+\frac{(-7)}{6}\)
Solution:

⇒ \(\frac{-5}{9}+\frac{(-7)}{6}\)

= \(\frac{-10+(-21)}{18}\)

Rational number

Question 9. Try this for some more pairs of rational numbers

1. \(\frac{-3}{8}-\frac{4}{5}\)
Solution:

⇒ \(\frac{-3}{8}-\frac{4}{5}\)

= \(\frac{-15-32}{40}\)

=\(\frac{-47}{40}\)

Rational number

2. \(\frac{2}{9}-\frac{3}{7}\)
Solution:

⇒ \(\frac{2}{9}-\frac{3}{7}\)

= \(\frac{14-27}{63}\)

= –\(\frac{13}{63}\)

Rational number

3. \(\frac{-3}{4} \times \frac{7}{8}\)
Solution:

⇒ \(\frac{-3}{4} \times \frac{7}{8}\)

= –\(\frac{21}{32}\)

Rational number

4. \(\frac{5}{8} \times \frac{3}{7}\)
Solution:

⇒ \(\frac{5}{8} \times \frac{3}{7}\)

= \(\frac{15}{56}\)

Rational number

Question 10. Fill in the blanks in the following table

NCERT Solutions For Class 8 Maths Chapter 1 Rational Numbers Fill In The Blanks In The Following Table Numbers

Solution:

NCERT Solutions For Class 8 Maths Chapter 1 Rational Numbers Fill In The Blanks In The Following Table Numbers Answer

Commutativity

Whole numbers

Question 1. Recall the commutativity of different operations for whole numbers by filling the following table.

NCERT Solutions For Class 8 Maths Chapter 1 Rational Numbers Recall The Commutativity Of Different Operations

Check whether the commutativity of the. operations hold for natural numbers also.
Solution:

NCERT Solutions For Class 8 Maths Chapter 1 Rational Numbers Commutativity Of The Operations Holds Answer

Commutativity of the operations for natural numbers also.

NCERT Solutions For Class 8 Maths Chapter 1 Rational Numbers Commutativity Of The Operations Of Natural Numbers Also

Integers

Question 1.  Fill in the following table and check the commutativity of different operations for integers:

NCERT Solutions For Class 8 Maths Chapter 1 Rational Numbers Commutativity Of Different Operations For Integers

Solution:

NCERT Solutions For Class 8 Maths Chapter 1 Rational Numbers Commutativity Of Different Operations For Integers Answer

Rational numbers

Question 1. \(\frac{-6}{5}+\left(\frac{-8}{3}\right)\) = \(\ldots \ldots \text { and } \frac{-8}{3}+\left(\frac{-6}{5}\right)=\ldots\) Is \(\frac{-6}{5}+\left(\frac{-8}{3}\right) \)= \(\left(\frac{-8}{3}\right)+\left(\frac{-6}{5}\right) \)
Solution:

⇒ \(\frac{-6}{5}+\left(\frac{-8}{3}\right)\)

= \( \frac{-18+(-40)}{15}=\frac{-58}{15}\)

= \(\frac{-8}{3}+\left(\frac{-6}{5}\right)\)

= \(\frac{-40+(-18)}{15}=\frac{-58}{15}\)

So, \(\frac{-6}{5}+\left(\frac{-8}{3}\right)=\left(\frac{-8}{3}\right)+\left(\frac{-6}{5}\right)\)

Question 2. Is \(\frac{3}{8}+\frac{1}{7}=\frac{1}{7}+\left(\frac{-3}{8}\right)\)?
Solution.

⇒ \(\frac{-3}{8}+\frac{1}{7}=\frac{-21+8}{56} \)

= \(\frac{-13}{56}\)

= \(\frac{1}{7}+\left(\frac{-3}{8}\right)=\frac{8+(-21)}{56}\)

= \(\frac{-13}{56}\)

So, \(\frac{-3}{8}+\frac{1}{7}=\frac{1}{7}+\left(\frac{-3}{8}\right)\).

Question 3. Is \(\frac{2}{3}-\frac{5}{4}=\frac{5}{4}-\frac{2}{3}\)
Solution:

⇒ \(\frac{2}{3}-\frac{5}{4}=\frac{8-15}{12}=\frac{-7}{12} \)

⇒ \(\frac{5}{4}-\frac{2}{3}=\frac{15-8}{12}=\frac{7}{12}\)

⇒  \(\frac{-7}{12} \neq \frac{7}{12} \)

∴ \(\frac{2}{3}-\frac{5}{4} \neq \frac{5}{4}-\frac{2}{3} \)

⇒  \(\frac{1}{2}-\frac{3}{5}=\frac{3}{5}-\frac{1}{2} \)

⇒  \(\frac{1}{2}-\frac{3}{5}=\frac{5-6}{10}=\frac{-1}{10} \)

⇒  \(\frac{3}{5}-\frac{1}{2}=\frac{6-5}{10}=\frac{1}{10} \)

⇒  \(\frac{-1}{10} \neq \frac{1}{10} \)

⇒  \(\frac{1}{2}-\frac{3}{5} \neq \frac{3}{5}-\frac{1}{2}\)

Question 4. \(\frac{-8}{9} \times\left(\frac{-4}{7}\right)=\frac{32}{63} \)
Solution:

⇒ \(\frac{-4}{7} \times\left(\frac{-8}{9}\right)=\frac{32}{63}\)

So, \(\frac{-8}{9} \times\left(\frac{-4}{7}\right)=\frac{-4}{7} \times\left(\frac{-8}{9}\right)\)

Question 5. \(\frac{-5}{9} \times \frac{7}{8}\)
Solution:

= \(\frac{7}{8} \times\left(\frac{-5}{9}\right)\)

= \(\frac{-35}{72}\)

Question 6. \(\frac{-6}{11} \times\left(\frac{-3}{5}\right)\)
Solution:

⇒ \(\frac{-6}{11} \times\left(\frac{-3}{5}\right)\)

=\(\frac{-3}{5} \times\left(\frac{-6}{11}\right)\)

= \(\frac{18}{55}\)

⇒ \(\frac{2}{3} \times \frac{4}{7}=\frac{4}{7} \times \frac{2}{3}=\frac{8}{21}\)

Question 7. Is \(\frac{-5}{4} \div \frac{3}{7}=\frac{3}{7}\div\left(\frac{-5}{4}\right)\)
Solution:

⇒ \(\frac{-5}{4} \div \frac{3}{7}=\frac{3}{7} \div\left(\frac{-5}{4}\right)\)

= \(\frac{-35}{12}\)

And \(\frac{3}{7} \div \frac{-5}{4}=\frac{3}{7} \times\left(\frac{-4}{5}\right) \)

=  \(\frac{-12}{35}\)

Since \(\frac{-35}{12} \neq \frac{-12}{35} \)

∴ \(\frac{-5}{4} \div \frac{3}{7} \neq \frac{3}{7} \div\left(\frac{-5}{4}\right)\)

Question 8. Complete the following table:

NCERT Solutions For Class 8 Maths Chapter 1 Rational Numbers Fill In The Blanks In The Following Table Numbers

Solution: 

NCERT Solutions For Class 8 Maths Chapter 1 Rational Numbers Fill In The Blanks In The Following Table Numbers

Associativity

Whole numbers

Question 1. Recall the associativity of the four operations for whole numbers through this table

NCERT Solutions For Class 8 Maths Chapter 1 Rational Numbers Four Operation For Whole Number

Fill in this table and. verify the remarks given in the last column. Check for yourself the associativity of different operations for natural numbers
Solution:

NCERT Solutions For Class 8 Maths Chapter 1 Rational Numbers Associativity Different Operations For Natural Numbers

NCERT Solutions For Class 8 Maths Chapter 1 Rational Numbers Associativity Different Operations For Natural Numbers.

Integers

Question 1. Is (-2) + [3 + (-4)] = [(-2) + 3] + (-4)?
Solution:

Yes; (-2) + [3 + (-4)]

= [(-2) + 3] + (-4)

= -3.

Addition is associative

Question 2. -Is (-6) +[(-4) +(-5)] = (-6) + (-4)] + (-5)?
Solution:

Yes; (-6) + [(-4) + (-5)]

= [(-6) + (-4)] + (-5)

= -15

Addition is associative

Question 3. Is 5 – (7 – 3) = (5 – 7) – 3?
Solution:

5 – (7 – 3) = 5 – 4

= 1

(5 – 7) – 3 = -2 – 3

= -5

1 ≠ – 5

∴  No; 5 – (7 – 3) ≠ (5 – 7) – 3.

Subtraction is not associative

Question 4. Is 5× [(- 7) × (-8)] = [5× (- 7)] × (- 8) ?
Solution:

Yes; 5 × [(-7) × (-8)]

= [5× (-7)] ×(-8)

= 280.

Multiplication is associative

Question 5. Is (- 4) × [(- 8)×(- 5)]= (-4) x (-8)]× (-5)
Solution:

Yes; (-4) × [(-8) × (-5)]

= [(-4) × (-8)] × (-5)

= -160

Multiplication is associative

Question 6.  Is [(-10) ÷ 2] ÷ (- 5)= (-10) ÷[2 ÷ (-5)]?
Solution:

[(-10) ÷ 2] ÷ (-5)= (-5) ÷ (-5) = 1

(-10) ÷ [2 ÷ (-5)]= (-10) ÷ \(\frac{-2}{5}\)

= (-10) ×\(\frac{-5}{2}\)

= 25

∴ 1 ≠ 25

∴ No; [(-10) ÷ 2] ÷ (-5)≠ (-10) ÷ [2 ÷ (-5)]

The division is not associative

Rational numbers

Question 1. Find \(\frac{-1}{2}+\left[\frac{3}{7}+\left(\frac{-4}{3}\right)\right]\) and \(\left[\frac{-1}{2}+\frac{3}{7}\right]+\left(\frac{-4}{3}\right) \text {. }\) Are the two sums equal?
Solution:

⇒ \(\frac{-1}{2}+\left[\frac{3}{7}+\left(\frac{-4}{3}\right)\right]\)

= \(\frac{-1}{2}+\frac{9+(-28)}{21}=\frac{-1}{2}+\left(\frac{-19}{21}\right)\)

= \(\frac{-21+(-38)}{42}=\frac{-59}{42}\)

And \( {\left[\frac{-1}{2}+\frac{3}{7}\right]+\left(\frac{-4}{3}\right)}\)

= \(\frac{-7+6}{14}+\left(\frac{-4}{3}\right)=\frac{-1}{14}+\left(\frac{-4}{3}\right)\)

= \(\frac{-3+(-56)}{42}=\frac{-59}{42}\)

= \(\frac{-1}{2}+\left[\frac{3}{7}+\left(\frac{-4}{3}\right)\right] \)

= \(\left[\frac{-1}{2}+\frac{3}{7}\right]+\left(\frac{-4}{3}\right)\)

= \(\left[\frac{-1}{2}+\frac{3}{7}\right]+\left(\frac{-4}{3}\right)\)

Question 2. \(\frac{-3}{4}+\left[\frac{2}{3}+\left(\frac{-6}{7}\right)\right]\)
Solution:

⇒ \(\frac{-3}{4}+\left[\frac{2}{3}+\left(\frac{-6}{7}\right)\right]\)

= \(\frac{-3}{4}+\frac{14+(-18)}{21}=\frac{-3}{4}+\left(\frac{-4}{21}\right)\)

= \(\frac{-63+(-16)}{84}=\frac{-79}{84} \)

= \(\left[\frac{-3}{4}+\frac{2}{3}\right]+\left(\frac{-6}{7}\right) \)

= \(\frac{-9+(8)}{12}+\left(\frac{-6}{7}\right)=\frac{-1}{12}+\left(\frac{-6}{7}\right) \)

= \(\frac{-7+(-72)}{84}=\frac{-79}{84}\)

So,yes \(\frac{-3}{4}+\left[\frac{2}{3}+\left(\frac{-6}{7}\right)\right]\)

= \(\left[\frac{-3}{4}+\frac{2}{3}\right]+\left(\frac{-6}{7}\right) \)

Question 3. \( \frac{-1}{4}+\left[\frac{2}{9}\right.\left.+\left(\frac{-5}{11}\right)\right] \)
Solution:

⇒  \( \frac{-1}{4}+\left[\frac{2}{9}\right.\left.+\left(\frac{-5}{11}\right)\right] \)

= \(\frac{-1}{4}+\left(\frac{22-(45)}{99}\right) \)

= \(\frac{-1}{4}+\left(\frac{-23}{99}\right) \)

⇒ \(\left[\frac{-1}{4}\right.\left.+\frac{2}{9}\right]+\left(\frac{-5}{11}\right)\)

= \(\frac{-99+(-92)}{396}=\frac{-191}{396}\)

= \(\frac{-9+8}{36}+\left(\frac{-5}{11}\right)\)

= \(\frac{-1}{36}+\left(\frac{-5}{11}\right)=\frac{-11+(-180)}{396}\)

= \(\frac{-191}{396} \)

⇒ \(\frac{-1}{4}+\left[\frac{2}{9}+\left(\frac{-5}{11}\right)\right] \)

= \(\left[\frac{-1}{4}+\frac{2}{9}\right]+\left(\frac{-5}{11}\right)\)

Question 4. \(\frac{-2}{3}-\left[\frac{-4}{5}-\frac{1}{2}\right]\) Check for yourself
Solution:

= \(\left[\frac{-2}{3}-\left(\frac{-4}{5}\right)\right]-\frac{1}{2}\)

= \(\frac{-2}{3}-\left[\frac{-4}{5}-\frac{1}{2}\right]\)

= \(\frac{-2}{3}-\left(\frac{-8-5}{10}\right)=\frac{-2}{3}-\left(\frac{-13}{10}\right)\)

= \(\frac{-2}{3}+\frac{13}{10}=\frac{-20+39}{30}=\frac{19}{30}\)

And \({\left[\frac{-2}{3}-\left(\frac{-4}{5}\right)\right]-\frac{1}{2}=\left[\frac{-2}{3}+\frac{4}{5}\right]-\frac{1}{2}} \)

= \(\left[\frac{-10+12}{15}\right]-\frac{1}{2}=\frac{2}{15}-\frac{1}{2} \)

= \(\frac{4-15}{30}=\frac{-11}{30}\)

So, No ; \(\frac{-2}{3}-\left[\frac{-4}{5}-\frac{1}{2}\right]\)≠ \(\frac{-2}{3}\)– (\(\frac{-4}{5}\)) – \(\frac{1}{2}\)

Question 5. \(\left(\frac{-7}{3} \times \frac{5}{4}\right) \times \frac{2}{9}\)___________
Solution:

⇒\(\left(\frac{-7}{3} \times \frac{5}{4}\right) \times \frac{2}{9}=\frac{-35}{12} \times \frac{2}{9}\)

=\(\frac{-35}{54}\)

So, Yes ;

Question 6. Is \(\frac{2}{3} \times\left(\frac{-6}{7} \times \frac{4}{5}\right)=\left(\frac{2}{3} \times \frac{-6}{7}\right) \times \frac{4}{5} \)
Solution:

⇒ \(\frac{2}{3} \times\left(\frac{-6}{7} \times \frac{4}{5}\right)\)

= \(\frac{2}{3} \times \frac{-24}{35}=\frac{-48}{105}\)________ (1)

=  \(\frac{2}{3}\)× \(\frac{-6}{7}\)× \(\frac{4}{5}\)

= \(\frac{-12}{21} \times \frac{4}{5}=\frac{-48}{105}\)________ (2)

So, Yes; A = B

Question 7. \(\frac{-6}{5} \times\left(\frac{2}{3} \times \frac{1}{8}\right)\)
Solution:

⇒ \(\frac{-6}{5} \times\left(\frac{2}{3} \times \frac{1}{8}\right)=\frac{-6}{5} \times \frac{1}{12}\)

=\(\frac{-1}{10}\)

And  \(\left(\frac{-6}{5} \times \frac{2}{3}\right) \times \frac{1}{8}=\frac{-4}{5} \times \frac{1}{8}=\frac{-1}{10}\)

So, Yes

⇒ \(\frac{-6}{5} \times\left(\frac{2}{3} \times \frac{1}{8}\right)=\left(\frac{-6}{5} \times \frac{2}{3}\right) \times \frac{1}{8}\)

Question 8. \(\frac{2}{7} \times\left(\frac{-5}{9} \times \frac{2}{3}\right) \)
Solution:

⇒ \(\frac{2}{7} \times\left(\frac{-5}{9} \times \frac{2}{3}\right) \)

=\(\frac{2}{7} \times\left(\frac{-10}{27}\right)=\frac{-20}{189} \)

= \(\left(\frac{2}{7} \times \frac{-5}{9}\right) \times \frac{2}{3}=\frac{-10}{63} \times \frac{2}{3}\)

=\(\frac{-20}{189}\)

So, Yes; A=B

Question 9. \(\frac{1}{2}+\left[\frac{-1}{3}+\frac{2}{5}\right]=\left[\frac{1}{2}+\left(\frac{-1}{3}\right)\right]+\frac{2}{5}\),Is L.H.S. =R.H.S. ? Check for yourself
Solution:

L.H.S.\(\frac{1}{2}+\left(\frac{-1}{3}+\frac{2}{5}\right)\)

= \(\frac{1}{2} \div\left(\frac{-1}{3} \times \frac{5}{2}\right)\)

= \(\frac{2}{5} is \frac{5}{2}\)

= \(\frac{1}{2}+\left(-\frac{5}{6}\right) \)

= \(\frac{1}{2} \times \frac{-6}{5}=\frac{-3}{5} \)

R.H.S= \(\left[\frac{1}{2}+\left(\frac{-1}{3}\right)\right]+\frac{2}{5}\)

= \(\left(\frac{1}{2} \times \frac{-3}{1}\right) \div \frac{2}{5}=\frac{-3}{2}+\frac{2}{5} \)

= \(\frac{-3}{2} \times \frac{5}{2}=\frac{-15}{4}\)

So, No; L.H.S ≠R. H.S.

Question 10. Complete the following table:

NCERT Solutions For Class 8 Maths Chapter 1 Rational Numbers Associative

Solution:

NCERT Solutions For Class 8 Maths Chapter 1 Rational Numbers Associative Answer

Question 11. Do you think the properties commutativity and associativity made the calculations
Sol.

Yes! The properties of commutativity and associativity made the calculations easier

The Rolie Of Zero (0)

Question 1. \(-5+0=\ldots \ldots. \ldots \ldots .=-5\)
Solution:

-5+0=0+(-5)=-5

Question 2.  \(\frac{-2}{7}+\ldots \ldots .=0+\left(\frac{-2}{7}\right)=\frac{-2}{7}\)

⇒ \(\frac{-2}{7}+0=0+\left(\frac{-2}{7}\right)=\frac{-2}{7}\)

Question 3. Do a few more such additions. What do you observe?
Solution:

1. 3 + 0 = 0 + 3 = 3

2. -9 +0 = 0 + (-9) = -9

3.\(\frac{-3}{4}+0=0+\left(\frac{-3}{4}\right)=\frac{-3}{4}\)

We observe that when we add 0 to a whole number, the sum is again that whole number. This happens for integers and rational numbers also.

The Role Of 1

Question 1. \(\frac{-2}{7} \times 1=\ldots \ldots . . \times \ldots . .=\frac{-2}{7}\)
Solution:

– \(\frac{2}{7} \times 1=1 \times \frac{-2}{7}=\frac{-2}{7}\)

Question 2. \(\frac{3}{8} \times \ldots \ldots=1 \times \frac{3}{8}=\frac{3}{8}\)
Solution:

⇒ \(\frac{3}{8} \times 1=1 \times \frac{3}{8}\)

= \(\frac{3}{8}\)

Question 3.  What do you find?
Solution:

We find that when we multiply any rational number by 1, we get back the same rational number as the product

Question 4. Check for a few more rational numbers

1. 7 × 1
Solution:

7 × 1 = 7 = 1 × 7

2.\(\frac{-3}{5} \times 1\)
Solution:

⇒ \(\frac{-3}{5} \times 1\)

=\(1 \times \frac{-3}{5}=\frac{-3}{5}\)

3. \(\frac{7}{9} \times 1\)
Solution:

⇒ \(\frac{7}{9} \times 1\)

= \(1 \times \frac{7}{9}=\frac{7}{9}\)

Question 5. Is 1 the multiplicative identity for integers? For whole numbers?
Solution:

Yes, 1 is the multiplicative identity for integers as well as for whole numbers.

Question 6. If a property holds for rational numbers, will it also hold for integers?

  1. For whole numbers?
  2. Which will?
  3. Which will not?

Solution:

1. Except for the following property, the properties of the rational numbers will also hold good for integers :

If a and b are rational numbers, then a÷b is also a rational number if b ≠ 0 but a÷ b is not necessarily an integer where a and b are integers

Eample: \(\frac{4}{5}\)is a rational number. Here 4 and 5 are integers, while \(\frac{4}{5}\) is not an integer

2. Except for the following properties, the properties of the rational numbers will also hold good for whole numbers:

If a and b are rational numbers, then, a- b is also a rational number. But if a and b are whole numbers, then A b is not necessarily a whole number

Let 4 and 5 be whole numbers, but 4 – 5 = -1 is not a whole number.

2. If a and b are rational numbers, then, a + b is a rational number if b  but if a and b are whole numbers, then a + b is not necessarily a whole number.

Example: 4 and 5 are whole numbers but \(\frac{4}{5}\) is not a whole number

Disberstivity Of Multiplication Over Addition For Rational

For all rational numbers a, b, and c,

a(b + c) = ab + ac.

a(b-c) = ab – ac

Question 1. \(\left\{\frac{7}{5} \times\left(\frac{-3}{12}\right)\right\}+\left\{\frac{7}{5} \times \frac{5}{12}\right\}\)
Solution:

⇒ \(\left\{\frac{7}{\mathbf{5}} \times\left(\frac{-3}{12}\right)\right\}+\left\{\frac{7}{5} \times \frac{\mathbf{5}}{\mathbf{1 2}}\right\}\)

= \(\frac{7}{5} \times\left\{\left(\frac{-3}{12}\right)+\frac{5}{12}\right\} \)

= \(\frac{7}{5} \times\left\{\frac{(-3)+5}{12}\right\}=\frac{7}{5} \times \frac{2}{12} \)

= \(\frac{7}{5} \times \frac{1}{6}=\frac{7}{30} \)

Question 2. \(\left\{\frac{9}{16} \times \frac{4}{12}\right\}+\left\{\frac{9}{16} \times \frac{-3}{9}\right\}\)
Solution:

⇒ \(\left\{\frac{9}{16} \times \frac{4}{12}\right\}+\left\{\frac{9}{16} \times \frac{-3}{9}\right\} \)

= \(\frac{9}{16} \times\left\{\frac{4}{12}+\left(\frac{-3}{9}\right)\right\}\)

= \(\frac{9}{16} \times\left\{\frac{1}{3}+\left(\frac{-1}{3}\right)\right\} \)

= \(\frac{9}{16} \times\left\{\frac{1-1}{3}\right\}=\frac{9}{16} \times 0=0 .\)

NCERT Solutions For Class 8 Maths Chapter 1 Rational Numbers Exercise 1.1

Question 1.  Name the property under multiplication used in each of the following

1. \( \frac{-4}{5} \times 1=1 \times \frac{-4}{5}=-\frac{4}{5} \)

2. \(\frac{13}{17} \times \frac{-2}{7}=\frac{-2}{7} \times \frac{-13}{17}\)

3. \(\frac{-19}{29} \times \frac{29}{-19}=1\)

Solution:

1.1 is the multiplicative identity

2. Commutativity of multiplication

3. Multiplicative inverse.

Question 2. Tell what property allows you tocompute :\(\frac{1}{3} \times\left(6 \times \frac{4}{3}\right) \text { as }\left(\frac{1}{3} \times 6\right) \times \frac{4}{3} \text {. }\)
Solution:

Associativity of multiplication

Question 3. The product of two rational numbers is always a
Solution:

The product of two rational numbers is always a rational number.

NCERT Solutions For Class 8 Maths Chapter 1 Rational Numbers Multiple Choice Questions

Question 1. Which of the following statements is false?

  1. Natural numbers are closed under addition
  2. Whole numbers are closed under the addition
  3. Integers are closed under the addition
  4. Rational numbers are not closed under addition.

Solution: 4. Rational numbers are not closed under addition.

Question 2. Which of the following statements is false?

  1. Natural numbers are closed under subtraction
  2. Whole numbers are not closed under subtraction
  3. Integers are closed under subtraction
  4. Rational numbers are closed under subtraction.

Solution: 1. Natural numbers are closed under subtraction

Question 3. Which of the following statements is true?

  1. Natural numbers are closed under multiplication
  2. Whole numbers are not closed under multiplication
  3. Integers are not closed under multiplication
  4. Rational numbers are not closed under multiplication.

Solution: 1. Natural numbers are closed under multiplication

Question 4. Which of the following statements is true?

  1. Natural numbers are closed under the division
  2. Whole numbers are not closed under the division
  3. Integers are closed under the division
  4. Rational numbers are closed under division.

Solution: 2.  Whole numbers are not closed under the division

Question 5. Which of the following statements is false?

  1. Natural numbers are commutative for addition
  2. Whole numbers are commutative for addition
  3. Integers are not commutative for addition
  4. Rational numbers are commutative for addition.

Solution: 3. Integers are not commutative for addition

Question 6. Which of the following statements is true?

  1. Natural numbers are commutative for subtraction
  2. Whole numbers are commutative for subtraction
  3. Integers are commutative for subtraction
  4. Rational numbers are not commutative for subtraction.

Solution: 4. Rational numbers are not commutative for subtraction.

Question 7. Which of the following statements is false?

  1. Natural numbers are commutative for multiplication
  2. Whole numbers are commutative for multiplication
  3. Integers are not commutative for multiplication
  4. Rational numbers are commutative for multiplication.

Solution: 3. Integers are not commutative for multiplication

Question 8. Which of the following statements is true?

  1. Natural numbers are commutative for division
  2. Whole numbers are not commutative for division
  3. Integers are commutative for division
  4. Rational numbers are commutative for division.

Solution: 2. Whole numbers are not commutative for division

Question 9. Which of the following statements is true?

  1. Natural numbers are associative for addition
  2. Whole numbers are not associative for addition
  3. Integers are not associative for addition
  4. Rational numbers are not associative for addition.

Solution: 1. Natural numbers are associative for addition

Question 10. Which of the following statements is true?

  1. Natural numbers are associative for subtraction
  2. Whole numbers are not associative for subtraction
  3. Integers are associative for subtraction
  4. Rational numbers are associative for subtraction

Solution: 2. Whole numbers are not associative for subtraction

Question 11. Which of the following statements is true?

  1. Natural numbers are not associative for multiplication
  2. Whole numbers are not associative for multiplication
  3. Integers are associative for multiplication
  4. Rational numbers are not associative for multiplication.

Solution: 3. Integers are associative for multiplication

Question 12. Which of the following statements is true?

  1. Natural numbers are associative for division
  2. Whole numbers are associative for division
  3. Integers are associative for division
  4. Rational numbers are not associative for division

Solution: 4. Rational numbers are not associative for division

Question 13. 0 is not

  1. A natural number
  2. A whole number
  3. An integer
  4. A rational number.

Solution: 1. A natural number

Question 14. ½ is

  1. A natural number
  2. A whole number
  3. A n integer
  4. A rational number.

Solution: 4.  A rational number.

Question 15. a + 6 = 6 + a is called

  1. Commutative law of addition
  2. Associative law of addition
  3. Distributive law of addition
  4. None of these.

Solution: 1. Commutative law of addition

Question 16. a × b = b × a is called

  1. Commutative law for addition
  2. Commutative law for multiplication
  3. Associative law for addition
  4. Associative law for multiplication.

Solution: 2. Commutative law for multiplication

Question 17. (a + b) + c = a + (b + c) is called

  1. Commutative law for multiplication
  2. Commutative law for
  3. Additionassociative law for addition
  4. Associative law for multiplication.

Solution: 3. Additionassociative law for addition

Question 18. a × (b × c) = (a × b) × c is called

  1. Associative law for addition
  2. Associative law for multiplication
  3. Commutative law for addition
  4. Commutative law for multiplication.

Solution: 2. Associative law for multiplication

Question 19. a (6 + c) = ab + ac is called

  1. Commutative law
  2. Associative law
  3. Distributive law
  4. None of these

Solution: 3. Distributive law

Question 20. The additive identity for rational numbers is

  1. 1
  2. -1
  3. 0
  4. None of these.

Solution: 3. 0

Question 21. The multiplicative identity for rational numbers is

  1. -1
  2. 1
  3. 0
  4. None of these

Solution: 2. 1

Question 22. How many rational numbers are there between any two given rational numbers?

  1. Only one
  2. Only two
  3. Countless
  4. Nothing can be said.

Solution: 3. Countless

Question 23. If a and 6 are two continuous rational numbers, then

  1. \(\frac{a+b}{2}<a\)
  2. \(\frac{a+b}{2}<b\)
  3. \(\frac{a+b}{2}=a\)
  4. \(\frac{a+b}{2}>b\)

Solution: 2.  \(\frac{a+b}{2}<b\)

NCERT Solutions For Class 8 Maths Chapter 1 Rational Numbers True Or False

1. a – 0 and 0 – a represent the same rational number where a ≠ 0. – False

2. 0 is a rational number – True

3. Between any two given rational numbers, there are countless rational numbers – True

NCERT Solutions For Class 8 Maths Chapter 1 Rational Numbers Fill In The Blanks

1. A number of the form p/q, where p and q are integers and q ≠0 is called a → Rational number

2. Write the number 0 reduced by 1 → (-1)

3. Are the two rational numbers p/q and q≠0 equivalent    →Yes

4. In the rational number p/q, why q≠0  → Since division by zero is not defined

5. Which rational number on the number line is equidistant from 0 and -1  → (\(\frac{-1}{2}\)).

NCERT Solutions For Class 8 Maths Chapter 6 Cubes And Cube Roots

NCERT Solutions For Class 8 Maths Chapter 6 Cubes And Cube Roots Introduction

Hardy – Ramanujan Number

Look at the following relations :

1729 = 1728 + 1 = 12³ + 1³

1729 = 1000 + 729 = 10³ + 9³

1729 is the smallest Hardy – Ramanujan Number (a number expressed as a sum of two cubes in two different ways is known as a Hardy- Ramanujan Number). There are infinitely many such numbers.

Few are 4104 (2, 16; 9, 15), 13832 (18, 20; 2, 24

Q. Check it with the numbers given in the brackets. 4104 (2, 16; 9, 15) and 13832 (18, 20; 2, 24)

2³ + 16³  = 8 + 4096 = 4104

9³ + 15³ = 729 + 3375 = 4104

18³ + 20³ = 5832 + 8 000 = 13832

2³ + 24³ = 8 + 13824 = 13832

Read and Learn More NCERT Solutions For Class 8 Maths

Cubes

Numbers obtained when a number is multiplied by itself three times are known as cube numbers or perfect cubes.

For example: 1, 8, 27,

The cube of a natural number m is denoted by m3 and is expressed as

m³ = m x m x m.

Thus, 1³ = 1 x 1 x 1 = 1

2³ = 2 × 2× 2 = 8

3³ = 3 ×  3 × 3 = 27, and so on

Question 1. How many cubes of side? 1 cm will \ make a cube of side 2 cm?
Solution:

2 × 2 × 2 = 8 cubes of side 1 cm will make a cube of side 2 cm.

Question 2. How many cubes of side 1 cm will make a cube of side 3 cm?
Solution:

3 × 3 × 3 = 27 cubes of side 1 cm will make a cube of side 3 cm

Question 3. Following are the cubes of the numbers from 11 to 20
Solution:

NCERT Solutions For Class 8 Maths Chapter 6 Cubes And Cube Roots The Cubes Of The Numbers Form 11 To 20

Question 4. Complete the following cubes of numbers  From 1 to 10

NCERT Solutions For Class 8 Maths Chapter 6 Cubes And Cube Roots Cubes Of Numbers From 1 To 10

Solution:

NCERT Solutions For Class 8 Maths Chapter 6 Cubes And Cube Roots Cubes Of Numbers From 1 To 10 Answer

Question 5. Are there only ten perfect cubes from 1 to 1000?
Solution:
Yes :

There are only ten perfect cubes from 1 to 1000.

These are: 1, 8, 27, 64, 126, 216, 343, 512, 729 and 1000

Question 6. How many perfect cubes are there from 1 to 100?
Solution:

There are only four perfect cubes from 1 to 100.

 These are: 1, 8, 27, and 64.

Question 7. Observe the cubes of even numbers. Are they all even? What can you say about the cubes of odd numbers?
Solution:

  1. The cubes of even numbers are all even.
  2. The cubes of odd numbers are all odd.

Question 8. Consider a few numbers having 1 as the one’s digit (or unit’s). Find the cube of each of them. What can you say about the one digit of the cube of a number having 1 as the one’s digit? Similarly, explore the one’s digit of cubes of numbers ending in 2, 3, 4,
Solution:

We observe that the one digit of the cube of a number having 1 as the one’s digit is 1. Similarly, the one’s digit of cubes of numbers ending in 0, 2, 3, 4, 5, 6, 7, 8, 9 are 0, 8, 7, 4, 5, 6, 3, 2, 9 respectively

Question 9. Find the one digit ofthe cube ofeach of the following numbers.

  1. 3331
  2. 8888
  3. 149
  4. 1005
  5. 1024
  6. 77
  7. 5022
  8. 53

Solution:

1. 3331

Unit digit of the number = 1

Unit digit of the cube of the number = 1

∴  1 × 1 × 1 = \(\underline{1}\)

2.  8888

Unit digit of the number = 8

Unit digit of the cube of the number = 2.

∴  8 × 8 × 8 = 51\(\underline{2}\)

3.  149

Unit digit of the number = 9

Unit digit of the cube of the number = 9

∴  9 × 9 × 9 = 72\(\underline{9}\)

4.  1005

Unit digit of the number = 5

Unit digit of the cube of the number = 5.

∴  5 × 5 × 5 = 12\(\underline{5}\)

5. 1024

Unit digit of the number = 4

Unit digit of the cube of the number = 4

∴  4 × 4 × 4 × 4 = 6\(\underline{4}\)

4. 77

Unit digit of the number = 7

Unit digit of the cube of the number = 3.

∴  7 × 7 × 7 = 34\(\underline{3}\)

8. 5022

Unit digit of the number = 2

Unit digit of the cube of the number = 8

∴  2 × 2 × 2 = \(\underline{8}\)

8. 53

Unit digit of the number = 3

Unit digit of the cube of the number = 7

∴  3 × 3 × 3 = 2\(\underline{7}\)

Some Interesting Patterns

If in the prime factorization of any number, each prime factor appears three times, then the number is a perfect cube.

For example, 216 =\(\underline{2 \times 2 \times 2} \times \underline{3 \times 3 \times 3} \times \underline{3 \times 3} \times \underline{5 \times 5}\)

= 2³ ×  3³ = (2× 3)³ = 6³ which is a perfect cube.

Question 1. How many consecutive odd numbers will be needed to obtain the sum as 10³?
Solution:  Ten.

Question 2. Express the following numbers as the sum of odd numbers using the above pattern.

Solution: 

1. 6³

Here, n = 6

∴ (n – 1) = 5

We start with n × (n – 1) + 1

= 6× 5 +1

= 31

6³ = 31 + 33 + 35 + 37+ 39 + 41

2. 8³

Here, n = 8

∴ (n – 1) = 7

We start with n × (n – 1) + 1

= 8 × 7  + 1

=  56+1

=  57

8³ = 57 + 59 + 61 + 63 + 65 + 67 + 69 + 71

3. 7³

Here, n = 7

(n -1) = 6

We start with n × (n – 1) + 1,

7 × 6 + 1,

= 42+1

= 43

7³ = 43 + 45 + 47 + 49 + 51 + 53 + 55

Question 3. Consider the following pattern.

  1. 2³ – 1³ = 1 + 2 × 1 × 3
  2. 3³ – 2³ = 1 + 3 × 2 × 3
  3. 4³ – 3³= 1 + 4 × 3 × 3

Using the above pattern, find the value of the following:

  1. 7³ – 6³
  2. 12³ – 11³
  3. 20³ – 19³
  4. 51³  – 50³ 

Sol.

1. 7³ – 6³ 

= 1 + 7 × 6 ×3

=1+126

= 127

2. 12³ – 11³

= 1 + 12 × 11 × 3

=1 + 396

= 397

3.  20³- 19³

T = 1 + 20 × 19 × 3

= 1 + 1140

= 1141

4.  51³ – 50³

1 + 51 × 50 × 3

= 1 + 7650

= 7651

Question 4. Which of the. following are perfect cubes

  1. 400
  2. 3375
  3. 8000
  4. 15625
  5. 90000
  6. 6859
  7. 2025
  8. 10648

1.  400

NCERT Solutions For Class 8 Maths Chapter 6 Cubes And Cube Roots 400 Is Not Perfect Cube

By prime factorisation,

Grouping the factors in triplets

400 = \(\underline{2 \times 2 \times 2} \times\)  2× 5 × 5.

= 2³ × 2× 5× 5

Bylaws of exponents

In the above factorization, 2 and 5 × 5 remain after grouping 2’s in triplets.

∴ 400 is not a perfect cube

2.33375

NCERT Solutions For Class 8 Maths Chapter 6 Cubes And Cube Roots 3375 Is Perfect Cube

Therefore, 400 is not a perfect cube

By prime factorisation,

3375 = \(\underline{3 \times 3 \times 3} \times \underline{5 \times 5 \times 5}\)

Grouping the factors in triplets

= 3³× 5³

Bylaws of exponents

= (3 × 5)³

= 15³, which is a perfect cube.

∴  3375 is a perfect cube.

3. 80000

NCERT Solutions For Class 8 Maths Chapter 6 Cubes And Cube Roots 8000 Is Not Perfect Cube

By prime factorisation,

8000 =  \(\underline{2 \times 2 \times 2} \times \underline{2 \times 2 \times 2} \times \underline{5 \times 5 \times 5}\)

Grouping the factors in triplets

= 2³× 2³ × 5³

Bylaws of exponents

= (2 × 2 ×5)³

= 20³, which is a perfect cube.

∴   8000 is a perfect cube

4.15625

NCERT Solutions For Class 8 Maths Chapter 6 Cubes And Cube Roots 15625 Is Perfect Cube

By prime factorisation,

15625 = \(\underline{5 \times 5 \times 5} \times \underline{5 \times 5 \times 5}\)

Grouping the factors in triplets

= 5³× 5³

Bylaws of exponents

= (5 × 5)³

Bylaws of exponents

25³, which is a perfect cube.

∴  15625 is a perfect cube

5. 9000

NCERT Solutions For Class 8 Maths Chapter 6 Cubes And Cube Roots 9000 Is Not Perfect Cube

By prime factorisation,

9000 = \(\underline{2 \times 2 \times 2} \times\) 3× 3 × \(\underline{5 \times 5 \times 5}\)

Grouping the factors in triplets

In the above factorization, 3×3 remains after grouping 2’s and 5’s in triplets.

∴  9000 is not a perfect cube.

6. 6859

NCERT Solutions For Class 8 Maths Chapter 6 Cubes And Cube Roots 6859 Is Perfect Cube

By prime factorisation

6859 = \(\underline{19 \times 19 \times 19}\)

Grouping the factors in triplets

= 19³

Bylaws of exponents which is a perfect cube

∴  6859 is a perfect cube

7. 2025

NCERT Solutions For Class 8 Maths Chapter 6 Cubes And Cube Roots 2025 Is Not Perfect Cube

By prime factorisation,

2025  = \(\underline{3 \times 3 \times 3} \times\) 3 × 5 × 5

Grouping the factors in triplets

= 3³ × 3 × 5 × 5

Bylaws of exponents

In the above factorization, 3 and 5 ×  5 remain after grouping 3’s in triplets.

∴ 2025 is not a perfect cube

8. 10648

NCERT Solutions For Class 8 Maths Chapter 6 Cubes And Cube Roots 10648 Is Perfect Cube

By prime factorisation

10648 = \(\underline{2 \times 2 \times 2} \times \underline{11 \times 11 \times 11} \times \)

Grouping the factors in triplets

= 2³ × 11³

Bylaws of exponents

= (2 × 11)³

Bylaws of exponents

= 22³, which is a perfect cube.

∴  10648 is a perfect cube

Smallest Multiple That Is A Perfect Cube

Sometimes we have to find the smallest natural number by which a number is multiplied or divided to make it a perfect cube

Question 1. Check which of the following are perfect cubes

  1. 2700
  2. 6000
  3. 64000
  4. 500
  5. 125000
  6. 50000
  7. 21600
  8. 10000
  9. 27000000
  10. 1000

What pattern do you observe in these perfect cubes
Solution:

1. 2700

NCERT Solutions For Class 8 Maths Chapter 6 Cubes And Cube Roots 2700 Is Not Perfect Cube

By prime factorisation,

2700 = 2× 2× \(\underline{3 \times 3 \times 3}\)×  5 ×  5

Grouping the factors in triplets

= 2  × 2 × 3³× 5× 5

Bylaws of exponents

In the above factorization, 2 × 2 and

5× 5 remain after grouping 3’s in triplets.

∴ 2700 is not a perfect cube

2. 16000

NCERT Solutions For Class 8 Maths Chapter 6 Cubes And Cube Roots 16000 Is Not A Perfect Cube

By prime factorisation,

16000 = \(\underline{2 \times 2 \times 2} \times \underline{2 \times 2 \times 2}\)  × 2 × \(\underline{5 \times 5 \times 5}\)

Grouping the factors in triplets

= 2³ × 2³ × 2 ×  5³

Bylaws of exponents

In the above factorization, 2 remains after grouping 2’s and 5’s in triplets.

∴ 16000 is not a perfect cube

3. 64000

NCERT Solutions For Class 8 Maths Chapter 6 Cubes And Cube Roots 64000 Is Perfect Cubes

By prime factorisation,

64000 = \(\underline{2 \times 2 \times 2 } \times \underline{2 \times 2 \times 2} \times \underline{2 \times 2 \times 2} \times \underline{5 \times 5 \times 5}\)

Grouping the factors in triplets

= 2³ ×2³ × 2³× 5³

By laws of exponents

= (2 × 2 × 2 ×5)³

= 40³, which is a perfect cube.

∴  64000 is a perfect cube

4.  900

NCERT Solutions For Class 8 Maths Chapter 6 Cubes And Cube Roots 900 Is Not Perfect Cube

By prime factorisation,

900 = 2 ×2 × 3 × 3 × 5 × 5

In the above factorization, 2 × 2, 3 × 3, and 5 × 5 remain when we try to group the factors in triplets.

∴ 900 is not a perfect cube.

5. 125000

NCERT Solutions For Class 8 Maths Chapter 6 Cubes And Cube Roots 125000 Is A Perfect Cube

By prime factorisation,

125000 = \(\underline{2 \times 2 \times 2 } \times \underline{5 \times 5 \times 5} \times \underline{5 \times 5 \times 5}\)

Grouping the factors in triplets

= 2³ × 5³ × 5³

Bylaws of exponents

= (2 × 5 × 5)³

= 50³, which is a perfect cube.

∴  125000 is a perfect cube.

6. 36000

NCERT Solutions For Class 8 Maths Chapter 6 Cubes And Cube Roots 36000 Is Not Perfect Cubes

By prime factorisation,

3600 = \(\underline{2 \times 2 \times 2 } \times \underline{2 \times 2 } \times \underline{3 \times 3} \times \underline{5 \times 5 \times 5}\)

Grouping the factors in triplets

= 2³ × 2 × 2 × 3 × 3 × 5³

Bylaws of exponents

In the above factorization, 2 × 2 and 3 ×3 remain after grouping 2’s and 5’s in  triplets

∴  36000 is not a perfect cube.

7. 21600

NCERT Solutions For Class 8 Maths Chapter 6 Cubes And Cube Roots 21600 Is Not Perfect Cube

By prime factorisation

21600 = \(\underline{2 \times 2 \times 2 }\) × 2 × 2 ×  \(\underline{5 \times 5 \times 5}\)

Grouping the factors in triplets

= 2³ × 2 × 2×3³×5 × 5

Bylaws of exponents

In the above factorization, 2×2 and 5×5 remain after grouping 2’s and 3’s in triplets

∴  21600 is not a perfect cube.

8. 10000

NCERT Solutions For Class 8 Maths Chapter 6 Cubes And Cube Roots 10000 Is Not Perfect Cube

By prime factorisation,

1000 =\(\underline{2 \times 2 \times 2}\) ×  2 × \(\underline{5 \times 5 \times 5}\) × 5

Grouping the factors in triplets

= 2³ × 2 × 5³ × 5

Bylaws of exponents

In the above factorization, 2 and 5 remain after grouping 2’s and 5’s in triplets.

∴ 10000 is not a perfect cube

9. 27000000

NCERT Solutions For Class 8 Maths Chapter 6 Cubes And Cube Roots 27000000 Is Perfect Cube

By prime factorisation,

27000000

=\(\underline{2 \times 2 \times 2} \times \underline{2 \times 2 \times 2} \times \underline{3 \times 3 \times 3} \times \underline{5 \times 5 \times 5}\times \underline{5 \times 5 \times 5}\)

Grouping the factors in triplet

= 2³×2³ × 3³× 5³ × 5³

Bylaws of exponents

= 300³, which is a perfect cube.

∴  27000000 is a perfect cube.

10. 1000

NCERT Solutions For Class 8 Maths Chapter 6 Cubes And Cube Roots 1000 Is Perfect Cube

By prime factorization,

1000 = \( \underline{2 \times 2 \times 2} \times \underline{5 \times 5 \times 5}\)

Grouping the factors in triplets

= 2³ × 5³

Bylaws of exponents

= (2 × 5)³ I By laws of exponents

= 10³, which is a perfect cube.

Therefore, 1000 is a perfect cube.

We observe the following pattern in these perfect cubes:

  1. If in the end of a number, the number of zeros is not 3 or a multiple of 3, then that number cannot be a perfect cube
  2. If in the end of a number, the number £ of zeros is 3 or a multiple of 3, then JJJ that number may be a perfect cube.
  3. Thus, the number of zero at the end of a perfect cube (if it is so) must essentially be 3 or a multiple of3, failing which the number cannot be a perfect cube.

NCERT Solutions For Class 8 Maths Chapter 6 Cubes And Cube Roots Exercise 6.1

Question 1. Which of the following numbers are not perfect cubes?

  1. 216
  2. 129
  3. 1000
  4. 100
  5. 46656

Solution:

1. 216

NCERT Solutions For Class 8 Maths Chapter 6 Cubes And Cube Roots 216 Is A Perfect Cube

By prime factorisation

216 =  \(\underline{2 \times 2 \times 2}\times \underline{3 \times 3 \times 3}\)

Grouping the factors in triplets

= 2³ × 3³

Bylaws of exponents

= (2 × 3)³

= 6³ which is a perfect cube.

∴  216 is a perfect cube

2. 128

NCERT Solutions For Class 8 Maths Chapter 6 Cubes And Cube Roots 128 Is Not A Perfect Cube

By prime factorisation

128 = \(\underline{2 \times 2 \times 2}\times \underline{2 \times 2 \times 2}\) × 2

Grouping the factors in triplets

= 2³ × 2³ × 2

Bylaws of exponents

In the above factorization,2remainsafter grouping the 2 ‘s in triplets

∴ 128 is not a perfect cube

3. 1000

NCERT Solutions For Class 8 Maths Chapter 6 Cubes And Cube Roots 1000 Is Perfect Cube

By prime factorisation

= \(\underline{2 \times 2 \times 2}\times \underline{5 \times 5 \times 5}\)

Grouping the factors in triplets

= 2³× 5³

Bylaws of exponents

= (2 × 5)³ = 10³

Bylaws of exponents

Which is a perfect cube.

∴ 1000 is a perfect cube

4. 100

NCERT Solutions For Class 8 Maths Chapter 6 Cubes And Cube Roots 100 Is Not A Perfect Cube

By prime factorisation

100 =  \(\underline{2 \times 2 \times }\times \underline{5 \times 5}\)

In (he above factorization, 2 × 2,5  × 5 remains when we try to group the factors in triplets

Therefore, 100 is not a perfect, cube.

5.46656

NCERT Solutions For Class 8 Maths Chapter 6 Cubes And Cube Roots 46656 Is A Perfect Cube

By prime factorisation,

46656 = \(\underline{2 \times 2 \times 2 \times 2 \times 2 \times 2}\) × \(\underline{3 \times 3 \times 3} \times \underline{3 \times 3 \times 3}\).

Grouping the factors in triplets

= 2³ × 2³×3³ × 3³

Bylaws of exponents

= (2 ×2 × 3 × 3)³

Bylaws of exponents

= 36³, which is a perfect cube.

∴  46656 is a perfect cube

Question 2. Find the smallest number by which each of the following numbers must be multi¬ plied to obtain, a perfect cube

  1. 243
  2. 256
  3. 72
  4. 675
  5. 100

Solution:

1. 243

NCERT Solutions For Class 8 Maths Chapter 6 Cubes And Cube Roots243 Is Smallest Number Which Is Numbers Must Be Multiplied to Obtain A Perfect Cube

By prime factorisation

243 =\(\underline{3 \times 3 \times 3}\) × 3 × 3

Grouping the factors in triplets

The prime factor 3 does not appear in a group of three.

Therefore, 243 is not a perfect cube.   To make it a cube, we need one more 3. In that case

243 = \(\underline{3 \times 3 \times 3} \times \underline{3 \times 3 \times 3}\)

= \(3^3 \times 3^3\)

= (3×3)³

729, which is a perfect cube.

Hence, the smallest number by which 243 should be multiplied to obtain a perfect cube is 3.

The resulting perfect cube is 3.

2. 256

NCERT Solutions For Class 8 Maths Chapter 6 Cubes And Cube Roots 256 Is Smallest Number Must Be Multiplied to Obtain A Perfect Cube

By prime factorisation

256 =\(\underline{2 \times 2 \times 2}\underline{2 \times 2 \times 2}\) 2 x 2

Grouping the factors in triplets

In the above factorisation 2× 2  after grouping 2‘s in Iriplols. Therefore, 128 is not a perfect cube. To make it a perfect, cube, we need one 2 more. In that, case

256× 2 = \(\underline{2 \times 2 \times 2} \times \underline{2 \times 2 \times 2} \times 2 \times 2\)

= 2³×2³× 2³

Bylaws of exponents

= (2×2×2)³

Bylaws of exponents

8³ = 512, which is a perfect cube.

Hence, the smallest number by which 256 must be multiplied to obtain a perfect cube is 2

The resulting perfect cube is 512 (= 8³)

3. 72

NCERT Solutions For Class 8 Maths Chapter 6 Cubes And Cube Roots 72 Not Perfect Cube

By prime factorisation,

72 = 2 × 2 × 2 × 3 × 3

Grouping the factors in triplets

The prime factor 3 does not appear in a group of three.

Therefore, 72 is not a perfect cube. To make it a perfect cube, we need one more 3. In that case,

72 × 3 = \(\underline{2 \times 2 \times 2} \times \underline{3 \times 3 \times 3}\)

= 2³ ×3³

Bylaws of exponent

= (2 × 3)³

Bylaws of exponents

6³ = 216 which is a perfect cube.

Hence, the smallest number by which 72 must be multiplied to obtain a perfect cube is 3.

The resulting perfect cube is 216 (= 6³)

4. 675

NCERT Solutions For Class 8 Maths Chapter 6 Cubes And Cube Roots 675 Is Smallest Number Must Be Multiplied to Obtain A Perfect Cube

By prime factorisation,

675 = 5 × 3 × 3 × 5 × 5

Grouping the factors in triplets

The prime factor 5 does not appear in a group of three. Therefore, 675 is not a perfect cube.

To make it a perfect cube, we need one more 5. In that case

675 ×  5 = \(\underline{3 \times 3 \times 3} \times \underline{5 \times 5 \times 5} \)

= 3³× 5³

Bylaws of exponents

= (3×5)³

Bylaws of exponents

= 15³

∴ 3375 which is a perfect cube.

Hence, the smallest number by which 675 must be multiplied to obtain a perfect cube is 5.

The resulting perfect cube is 3375 (= 15³)

5. 100

NCERT Solutions For Class 8 Maths Chapter 6 Cubes And Cube Roots 100 Is Smallest Number Must Be Multiplied to Obtain A Perfect Cube

By prime factorisation,

100 = 2 × 2 × 5 × 5

Grouping the factors in triplets

The prime factors 2 and 5 do not appear in a group of three. Therefore, 100 is not a perfect cube.

To make it a perfect cube, we need one 2 and one 5 more. In that case,

100 × 2 × 5 = \(\underline{2 \times 2 \times 2} \times \underline{5 \times 5 \times 5} \)

= 2³ × 5³

Bylaws of exponents 2

= (2 × 5)³

Bylaws of exponents

= 10³

∴ 1000 which is a perfect cube.

Hence, the smallest number by which 100 must be multiplied to obtain a perfect cube is 2× 5 = 10

The resulting perfect is 1000 (= 10³ )

Question 3. Find the smallest number by which each of the following numbers must be divided to obtain a perfect cube.

  1. 81
  2. 128
  3. 135
  4. 192
  5. 704

Solution:

1. 81

NCERT Solutions For Class 8 Maths Chapter 6 Cubes And Cube Roots 81 Is Smallest Number Must Be Divided To Obtain A Perfect Cube

By prime factorisation

81 =\(\underline{3 \times 3 \times 3}\times\) 3

Grouping the factors in triplets

In the above factorization, 3 remains after grouping the 3’s in triplets. Therefore,

81 is not a perfect cube. If we divide the number by 3, then in the prime factorization of the quotient, this 3 will not remain. In that case,

81 ÷ 3  = \(\underline{3 \times 3 \times 3}\)

= 3³

Bylaws of exponents which is a perfect cube

Hence, the smallest whole number by which 81 must be divided to obtain a perfect cube is 3.

The resulting ideal cube is 27 (= 3³)

2. 128

NCERT Solutions For Class 8 Maths Chapter 6 Cubes And Cube Roots 128 Is Smallest Number Must Be Divided To Obtain A Perfect Cube

By prime factorisation,

128 = \(\underline{2 \times 2 \times 2} \times \underline{2 \times 2 \times 2}\)

Grouping the factors in triplets

In the above factorization, 2 remains after grouping’ the 2’s in triplets.

Therefore, 128 is not a perfect cube. If we divide the number by 2, then in the prime factorization ofthe quotient, this 2 will not remain. In that case,

128 ÷ 2 = \(\underline{2 \times 2 \times 2} \times \underline{2 \times 2 \times 2}\)

= 2³ × 2³

= (2 × 2)³

Bylaws of exponents

= 4³, which is a perfect cube.

Hence, the smallest whole number by  which 128 must be divided to obtain a perfect cube is 2

The resulting ideal cube is 64 (= 4³)

3. 135

NCERT Solutions For Class 8 Maths Chapter 6 Cubes And Cube Roots 135 Is Smallest Number Must Be Divided To Obtain A Perfect Cube

By prime factorisation,

135 = \(\underline{3 \times 3 \times 3} \times 5\)

Grouping the factors in triplets.

The prime factor 5 does not appear in a group of three. So 135 is not a perfect cube

In the factorisation 5 appears only once. If we (u) 704 divide 135 by 5, then the prime factorization of the quotient, will not contain 5.

In that case

135 ÷ 5 = \(\underline{3 \times 3 \times 3}\)

Grouping the factors in triplets

= 27

Bylaws of exponents, which is a perfect cube.

Hence, the smallest whole number by  which 135 must be divided to obtain a perfect cube is 5

The resulting perfect cube is 27 (= 33)

4. 192

NCERT Solutions For Class 8 Maths Chapter 6 Cubes And Cube Roots 192 Is Smallest Number Must Be Divided To Obtain A Perfect Cube

By prime factorisation,

192  = \(\underline{2 \times 2 \times 2} \times \underline{2 \times 2 \times 2} \) × 3

Grouping the factors in triplets

The prime factor 3 does not appear in a group of three. So 192 is not a perfect cube. In the factorization of 192, 3 appears only once. So if we divide the number by 3, then the prime factorization of the quotient will not contain 3.

.In that case,

192 ÷ 3 = \(\underline{2 \times 2 \times 2} \times \underline{2 \times 2 \times 2} \) × 3

= 2³ × 2³

= (2 × 2)³

Bylaws of exponents

= 4³ = 64

Which is a perfect cube.

Hence, the smallest whole number by which 192 must be divided to obtain a perfect cube is 3.

The resulting perfect cube is 64 (= 4³)

5. 704

NCERT Solutions For Class 8 Maths Chapter 6 Cubes And Cube Roots 704 Is Smallest Number Must Be Divided To Obtain A Perfect Cube

By prime factorisation,

704  =  \(\underline{2 \times 2 \times 2} \times \underline{2 \times 2 \times 2} \) × 11

Grouping the factors in triplets

The prime factor 11 does not appear in the group of three.

So, 704 is not a perfect cube.

In the factorization 11 appears only one time.

So if we divide 704 by 11, then the prime factorization of the quotient will not contain 11. In that case

704 ÷ 11 = 2 × 2 × 2 × 2 × 2 × 2

Grouping the factors in triplets

2³ × 2³

Bylaws of exponents

= (2 × 2)³

= 4³

= 64

Which is a perfect cube.

Hence, the smallest whole number by which 704 must be divided to obtain a perfect cube is 11.

The resulting perfect cube is 64 (= 4³).

Question 4. Parikshit makes a cuboid of plasticine of sides 5 cm, 2  How many such cuboids will he need to form a cube? cm, 5 cm.
Solution:

Volume of a cuboid

= 5 × 2 × 5 cm³

Since there is only one 2 and only two 5’s in the prime factorization, so, we need 2 × 2 × 5, i.e.t 20 to make a perfect cube.

Therefore, we need 20 such cuboids to make a cube.

Cube Roots

The cube root is the inverse operation of finding a cube.

2³ = 8 ⇒  2 is the cube root of 8.

The symbol denotes the cube root. Thus,\(\sqrt[3]{8}=2\)

Cube Root Through Prime Factorisation Method

We express the given number into product of its prime factors and make triplets (groups of three) of similar factors. Then, we take one factor from each triplet and multiply. The product so obtained gives the cube root of the given number.

Question 1. State true or false: for any integer m, m²< m³. Why?
Solution:

False, if m is a negative integer.

True, if m is a positive integer (natural number).

Verification. Let us take a negative integer m =-1. Then,

m²= (- 1)2 = (- 1)² ×  (-1) = 1

m³ = (_ 1)3 = (- 1)³ × (-1) × (-1)

= – 1

Clearly, m² > m³

Hence, the given statement is false if m is a negative integer.

We get the same inference for m = – 2, -3,-4, etc.

Again, let us take a positive integer(natural number) m = 2. Then,

m² = (2)² = 2 × 2 = 4

m³ = (2)³ = 2 × 2 × 2 = 8

Clearly, m² < m³

Hence, the given statement is true if m is a positive integer (natural number).

We get the same inference for m = 3, 4,5… etc

Cube Root of a Cube Number Steps

  1. Obtain the given number. Start making groups of three digits starting from the rightmost digit of the number.
  2. The first group will give one’s (unit’s) digit of the required cube root.
  3.  Then, take another group. Find the two closest cube numbers between which this group lies. Take the one’s place of the smaller number as the ten’s place of the required cube root

NCERT Solutions For Class 8 Maths Chapter 6 Cubes And Cube Roots Exercise 6.2

Question 1. Find the cubit root of each of the foUouiu# numbers by prime factorization method :

  1. 64
  2. 512
  3. 10648
  4. 27000
  5. 15625
  6. 13824
  7. 110592
  8. 46656
  9. 175616
  10. 91125

Solution:

1. 64

NCERT Solutions For Class 8 Maths Chapter 6 Cubes And Cube Roots Cube Root Of 64

The prime factorization of 64 is

= \(\underline{2 \times 2 \times 2} \times \underline {2\times 2 \times 2}\)

Grouping the factors in triplets

= 2³× 2³

= (2 × 2)³

= 4³

Bylaws of exponents

∴ \(\sqrt[3]{64}\) = 2 × 2

2.512

NCERT Solutions For Class 8 Maths Chapter 6 Cubes And Cube Roots Cube Root Of 512

The prime factorisation of 512 is

⇒ \(\underline{2 \times 2 \times 2} \times \underline{2 \times 2 \times 2} \times \underline{2 \times 2 \times 2}\)

Grouping the factors in triplets

= 2³× 2³ × 2³

= (2 × 2 ×2)³

= 8³

Bylaws of exponents

∴ \(\sqrt[3]{512}\) = 2 × 2 ×2

= 8³

3. 10648

NCERT Solutions For Class 8 Maths Chapter 6 Cubes And Cube Roots 10648 Is Perfect Cube

Prime factorization of 10648 is

⇒ [late]\underline{2 \times 2 \times 2} \times \underline{11 \times 11 \times 11} [/latex]

Grouping the factors in triplets

= 2³ x 11³

= (2 × 11)³

∴ \(\sqrt[3]{10648}\) = 2 × 11

= 22³

4. 27000

NCERT Solutions For Class 8 Maths Chapter 6 Cubes And Cube Roots 2700 Is Not Perfect Cube

Prime factorisation of 27000 is

⇒ \(\underline{2 \times 2 \times 2} \times \underline{3 \times 3  \times 3 } \times \underline{5 \times 5 \times 5 } \times \underline{5 \times 5}\)

Grouping the factors in triplets

= 2³ × 3³ × 5³

= (2 × 3 × 5)³

Bylaws of exponents

= 30³

∴ \(\sqrt[3]{27000}\) = 2×3 × 5

= 30

5. 15625

NCERT Solutions For Class 8 Maths Chapter 6 Cubes And Cube Roots Prime Factorisation Of 15625

The prime factorization of 15625 is

⇒ \(\underline{5 \times 5 \times 5} \times \underline{5 \times 5  \times 5 }\)

Grouping the factors in triplets

= 5³ × 5³

= (5 × 5)³

Bylaws of  exponents

= 25³

∴ \(\sqrt[3]{15625}\) = 5 × 5

= 25

6. 13824

NCERT Solutions For Class 8 Maths Chapter 6 Cubes And Cube Roots Cube Root Of 13824

Prime factorization of 13824 is

⇒  \(\underline{2 \times 2 \times 2}\times \underline{2 \times 2 \times 2} \times \underline{2 \times 2 \times 2} \times \underline{3 \times 3 \times 3}\)

Grouping the factors in triplets

= 2³ × 2³× 2³ × 3³

=(2× 2 × 2 × 3)³

Bylaws of exponents

= 24³

∴ \(\sqrt[3]{13824}\)  = 2× 2 × 2 × 3

= 24

7. 1110592

NCERT Solutions For Class 8 Maths Chapter 6 Cubes And Cube Roots Cube Root Of 110592

Prime factorisation of 110592 is

⇒  \(\underline{2 \times 2 \times 2}\)× \(\underline{2 \times 2 \times 2}\) × \(\underline{2 \times 2 \times 2}\) × \(\underline{2 \times 2 \times 2}\) × \(\underline{3 \times 3 \times 3}\)

Grouping the factors into triplets

= 2³ × 2³ × 2³ × 2³ × 3

= (2 × 2 × 2 × 2 ×3)³

Bylaws of exponents

= 48³

∴ \(\sqrt[3]{110592}\) =2 × 2 × 2 × 2 ×3

8. 46656

NCERT Solutions For Class 8 Maths Chapter 6 Cubes And Cube Roots Cube Root Of 46656

Prime factorization of 46656 is

⇒ \(\underline{2 \times 2 \times 2} \times \underline{2 \times 2 \times 2} \times \underline{3 \times 3 \times 3} \times \underline{3 \times 3 \times 3}\)

Grouping the factors in triplets

= 2³× 2³ × 3³ × 3³

= (2 × 2 × 3 × 3)³

Bylaws of exponents

= 36³

∴ \(\sqrt[3]{46656}\) = 2 × 2 × 3 × 3

=36

9. 175616

NCERT Solutions For Class 8 Maths Chapter 6 Cubes And Cube Roots Cube Root Of 175616

Prime factorisation of 175616 is

⇒  \(\underline{2 \times 2 \times 2 \times} \times \underline{2 \times 2 \times 2 \times} \times \underline{2 \times 2 \times 2 \times} \times \underline{7 \times 7 \times 7}\)

Grouping the factors in triplets

= 2³ × 2³ × 2³ × 7³

= (2 × 2 × 2 × 7)³

Bylaws of exponents

= 56³

∴ \(\sqrt[3]{175616}\) = 2 × 2 × 2 × 7

= 56

10. 91125

NCERT Solutions For Class 8 Maths Chapter 6 Cubes And Cube Roots Cube Root Of 91125

Prime factorization of 91125 is

⇒ \(\underline{3 \times 3 \times 3 \times} \times \underline{3 \times 3 \times 3 \times} \times \underline{5 \times 5 \times 5}\)

Grouping the factors in triplets

= 3³ ×3³ × 5³

= (3 × 3× 5)³

Bylaws of exponents

= 45³

∴ \(\sqrt[3]{91125}\) =3 × 3× 5

= 45

NCERT Solutions For Class 8 Maths Chapter 6 Cubes And Cube Roots Multiple Choice Questions

Question 1. Which of the following numbers is a perfect cube?

  1. 125
  2. 36
  3. 75
  4. 100

Solution: 1. 125

125 = 5 × 5 × 5 = 5³

Question 2. Which of the following numbers is a cube number?

  1. 1000
  2. 400
  3. 100
  4. 600.

Solution: 1.1000

1000 = 10 × 10 × 10 = 10³

Question 3. Which of the following numbers is not a perfect cube?

  1. 1331
  2. 512
  3. 343
  4. 100

Solution: 4. 100

100 = 2 ×2 × 5 ×5

= 2² × 5².

Question 4. Which of the following numbers is not a cube number?

  1. 10000
  2. 3375
  3. 64
  4. 729.

Solution: 1. 10000

10000 = 2 × 2 ×2 × 2 ×5 ×5× 5 ×5

= 24 × 54

= 23× 2 × 53 × 5

Question 5. The cube of an odd natural number is

  1. Even
  2. Odd
  3. Maybe even, maybe odd
  4. Prime number.

Solution: 2. Odd

3 × 3 × 3 = 27 (odd)

Question 6. The cube of an even natural number is

  1. Even
  2. Odd
  3. Maybe even, maybe odd
  4. Prime number.
  5. Solution: 1. Even

6 × 6 × 6 = 216 (even)

Question Question 7. The one digit of the cube of the number 111 is

  1. 1
  2. 2
  3. 3
  4. 9.

Solution: 1. 1

1 × 1 × 1 =  1

Question 8. The one digit of the cube of the number 242 is

  1. 2
  2. 4
  3. 6
  4. 8.

Solution: 4. 8

2 × 2 × 2= 8.

Question 9. The one digit of the cube of the number 123 is

  1. 3
  2. 6
  3. 9
  4. 7

Solution: 4. 7

3 × 3 × 3 =2\(\underline{7}\)

Question 10. The one’s digit of the cube of the number 144 is

  1. 1
  2. 2
  3. 3
  4. 4.

Solution: 4. 4

4 × 4 ×4 = 6\(\underline{4}\)

Question 11. The one’s digit of the cube of the number 50 is

  1. 1
  2. 0
  3. 5
  4. 4

Solution: 2. 0

0 × 0 × 0 = 0

Question 12. The one digit of the cube of the number 326 is

  1. 2
  2. 3
  3. 6
  4. 4

Solution: 3. 3

6 × 6 × 6 = 21\(\underline{6}\)

Question 13. The one digit of the cube of the number 325 is

  1. 2
  2. 5
  3. 3
  4. 6.

Solution: 2. 5

5× 5× 5= 12\(\underline{5}\)

Question 14. The one digit of the cube of the number 347 is

  1. 3
  2. 4
  3. 7
  4. 1

Solution: 1. 3

7×7×7 = 34\(\underline{3}\)

Question 15. The one digit of the cube of the number 68 is

  1. 1
  2. 2
  3. 6
  4. 8

Solution: 2. 2

8× 8 ×8 = 51\(\underline{2}\)

Question 16. The one digit of the cube of the number 249 is

  1. 2
  2. 4
  3. 9
  4. 1

Solution: 3. 9

9 × 9 × 9 = 72\(\underline{9}\)

Question 17. What is the one’s digit in the cube root of the cube number 1331?

  1. 1
  2. 2
  3. 3
  4. 4

Solution: 1. 1

1 × 1 × 1 = 1

Question 18. What is the one digit in the cube root of the cube number 1000000?

  1. 0
  2. 1
  3. 2
  4. 9

Solution: 1. 0

0 ×0 × 0 = 0

Question 19. What is the one’s digit in the cube root of the cube number 1728?

  1. 1
  2. 2
  3. 3
  4. 9

Solution: 2. 2

2 × 2 × 2 = \(\underline{8}\)

Question 20. What is the one’s digit in the cube root, of the cube number 2197′?

  1. 1
  2. 2
  3. 3
  4. 7

Solution: 3. 3

3× 3 × 3 = 2\(\underline{7}\)

Question 21. What is t lie one’s digit, in the cube root, of the cube number 2744?

  1. 1
  2. 2
  3. 3
  4. 4

Solution: 4. 4

4 × 4 × 4 = 6\(\underline{4}\)

Question 22. What is the one’s digit in the cube root of the cube number 3375?

  1. 2
  2. 3
  3. 5
  4. 4

Solution: 3. 5

5 × 5 × 5 = 12\(\underline{5}\)

Question 23. What is the one’s digit in the cube root of the cube number 4096? 

  1. 2
  2. 6
  3. 4
  4. 9.

Solution: 2. 6

6 × 6 × 6 = 21\(\underline{6}\).

Question 24. What is the one’s digit in the cube root of the cube number 4913?

  1. 7
  2. 9
  3. 3
  4. 6

Solution: 1. 7

7 × 7 ×7 = 34\(\underline{3}\)

Question 25. What is the one’s digit in the cube root of the cube number 5832?

  1. 2
  2. 4
  3. 6
  4. 8

Solution: 4. 8

8 × 8 × 8 = 51\(\underline{2}\)

Question 26. What is the one’s digit in the cube root of the cube number 6859?

  1. 7
  2. 8
  3. 9
  4. 6

Solution: 3. 9

8 × 8 × 8 = 51\(\underline{2}\)

Question 27. What, is the one’s digit in the cube root of the cube number 8000 ?

  1. 0
  2. 2
  3. 4
  4. 8

Solution: 1. 0

0 × 0 × 0 = 0.

Question 28. The number of zeroes at the end of the cube of the number 20 is

  1. 1
  2. 2
  3. 3
  4. 6

Solution: 3. 3

Number of zeroes at the end of the number 20 = 1

Number of zeroes at the end of its cube = 3 × 1 = 3

Question 29. The number of zeroes at the end of the cube root of the cube number 1000 is

  1. 1
  2. 2
  3. 3
  4. 4

Solution: 1. 1

The number of zeroes at the end of the cube = 3

Number of zeroes at the end of the cube root = \(\frac{3}{3}\)

= 1

Question 30. The number of zeroes at the end of the cube of the number 100 is

  1. 1
  2. 2
  3. 4
  4. 6

Solution: 4. 6

Number of zeroes at the end of the number 100 = 2

Number of zeroes at the end of its cube = 3 × 2

= 6

Question 31. The number of zeroes at the end of the cube root Of the cube number 8000000 in

  1. 1
  2. 2
  3. 3
  4. 6

Solution: 2. 2

The number of zeroes at the end of the cube = 6

Number of zeroes at the end of the cube root =\(\frac{6}{3}\)

= 2

Question 32. Find the smallest number by which the number 108 must be multiplied to obtain a perfect, cube. ,

  1. 2
  2. 3
  3. 4
  4. 5

Solution: 1. 2

108 = 2 × 2 × 3 × 3 × 3

= 2 × 2 × 3³

Question 33. Find the smallest number by which the number 250 must be divided to obtain a perfect cube.

  1. 2
  2. 3
  3. 4
  4. 5

Solution: 1. 3

250 = 5 ×5 × 5 × 2 = 5³ × 2

Question 34. Find the smallest number by which the number 72 must be multiplied to obtain a perfect cube.

  1. 2
  2. 3
  3. 4
  4. 6

Solution: 2. 3

72 = 2 × 2 × 2 × 3 × 3

= 2³ × 3 × 3

Question 35. Find the smallest number by which the number 375 must be divided to obtain a perfect cube.

  1. 2
  2. 3
  3. 5
  4. 4.

Solution: 2. 3

375 = 3 × 5 × 5 × 5

= 3× 53.

Question 36. Find the smallest number by which the number 100 must be multiplied to obtain a perfect cube.

  1. 5
  2. 2
  3. 4
  4. 10

Solution: 4. 10

100 = 2 × 2 ×5 × 5.

Question 37. Find the smallest number by which the number 10000 must be divided to obtain a perfect cube.

  1. 2
  2. 5
  3. 10
  4. 100.

Solution: 3. 10

10000 = 2 × 2 × 2 × 2 ×5×5 × 5 × 5

= 2³ × 2 × 5³× 5

Question 38. Find the smallest number by which the number 200 must be multiplied to obtain a perfect cube.

  1. 2
  2. 10
  3. 5
  4. 100

Solution: 3. 5

200 =2 × 2 × 2 × 5 × 5

= 2³ ×5 × 5

Question 39. Find the smallest number by which the number 625 must be divided to obtain a perfect cube.

  1. 3
  2. 5
  3. 25
  4. 125

Solution: 2. 5

625 = 5 × 5 × 5 ×5

= 5³ × 5.

Question 40. Kind the smallest number by which (bo number 128 must the multiplied to obtain a perfect cube.

  1. 2
  2. 4
  3. 3
  4. 8

Solution: 2. 4

128 =2 × 2 × 2 × 2 × 2 × 2 × 2

= 2³ ×2³ × 2

Question 41. Find the smallest number bv which the number 250 must be divided to obtain a perfect cube.

  1. 2
  2. 4
  3. 8
  4. 16

Solution: 2. 4

256 = 2 × 2× 2× 2 × 2× 2 ×  2×2

= 2³ × 2³ × 2 × 2.

Question 42. Find the smallest number by which the number 30 must be multiplied to obtain a perfect cube.

  1. 6
  2. 2
  3. 3
  4. 4

Solution: 1. 6

36 = 2 × 2 × 3 × 3

Question 43. Find the smallest number by which the number 1290 must be divided to obtain a perfect cube.

  1. 6
  2. 2
  3. 4
  4. 3

Solution: 1. 6

1296 = 2 × 2× 2 × 2 × 3 × 3

=  2³× 2 ×3³× 3

Question 44. Find the smallest number by which the number 392 must be multiplied to obtain a perfect cube.

  1. 3
  2. 5
  3.  7
  4. 6

Solution: 3.  7

392 = 2 × 2 × 2 × 7× 7

Question 45. Find the smallest number by which the number 2401 must be divided to obtain a perfect cube.

  1. 1
  2. 0
  3. 5
  4. 9.

Solution: 1. 1

2401 = 7 × 7 × 7 × 7

= 7³ × 7

Question 46. Find the smallest number by which the number 121 must be multiplied to obtain a perfect cube.

  1. 1
  2. 9
  3. 11
  4. 5

Solution: 3.11

121 = 11× 11

Question 47. Find the smallest number by which the number 88 must be divided to obtain a perfect cube.

  1. 11
  2. 5
  3. 7
  4. 9

Solution: 1.11

88 = 2 × 2 ×2 × 11

= 2³ ×11

Question 48. The volume of a cube is 64 cm3. The edge of the cube is

  1. 4 cm
  2. 8 cm
  3. 16 cm
  4. 6 cm

Solution: 1. 4 cm

Edge = \(\sqrt[3]{64}=\sqrt[3]{2 \times 2 \times 2 \times 2 \times 2 \times 2}\)

= \(\sqrt[3]{2^3 \times 2^3}\)

= 2 × 2

= 4.

Question 49. Apala makes a cuboid of plasticine with sides 5 cm, 4 cm, 2 cm. How many such cuboids will be needed to form a cube?

  1. 20
  2. 25
  3. 10
  4. 16

Solution: 2. 25

Volume = 5 × 4 × 2

= 5 × 2× 2 × 2

= 5 ×2³

Question 50. Which of the following is false?

  1. A Cube of any odd number is odd
  2. A perfect cube does not end with two zeroes
  3. The cube of a single-digit number may be a single-digit number
  4. There is no perfect cube that ends with 8.

Solution: 4.   There is no perfect cube that ends with 8.

1728 = 12 × 12  × 12

= 12³

NCERT Solutions For Class 8 Maths Chapter 6 Cubes And Cube RootsTrue Or False

There are four perfect cubes between 1 and 100 – False

2. Ifa2 ends in 9, then a3 ends in 9  – False

3. Ifa2 ends in 5, then a3 ends in 25  – False

4. 999 is not a perfect cube  – True

5. Cube roots of 1 are + 1 and -1   – False

6. Cube of any odd number is even – False

7. A perfect cube does not end with two zeros – True

8. If the square of a number ends with 5, then its cube ends with 25. – False

⇒  15² = 225, 15³= 3375

9. There is no perfect cube that ends – False

⇒   12³ = 1728

10. The cube of a two-digit number may be a three-digit number – False 

⇒   10³ = 1000, 99³= 970299

11. The cube of a two-digit number may have seven or more digits -False 

⇒  10³ = 1000, 99³ = 970299

12. The cube of a single-digit number may be a single-digit number – True

⇒  1³ = 1; 2³ = 8

NCERT Solutions For Class 8 Maths Chapter 6 Cubes And Cube Roots Fill In The Blanks

1. \(\sqrt[3]{x}\)represents the Cube  root of the number x

2. 1 cm³  10³ mm³

3. The cube of a number ending in 9 will end in the digit → 9

4. The cube of 100 will have zeros 6 at the end

5. There are  8 perfect cubes between 1 and 1000.

6. Find the value of \(\sqrt[3]{125} \times \sqrt[3]{-64}\) → 20

7. Find the least number by which 72 should be multiplied to make it a perfect cube → 3

8. Find the least number by which 72 should be divided to make it a perfect cube → 9

9. Find(1.1)³→ 1.331

10. Find the number whose cube is 64000 →  40

NCERT Solutions For Class 8 Maths Chapter 7 Comparing Quantities

NCERT Solutions For Class 8 Maths Chapter 7 Comparing Quantities Recalling Ratios And Percentages Important Points

1. We usually compare two quantities by division, i.e., by using fractions. Comparison by division is called ratio.

Note that two quantities can be compared only when they have the same units. A ratio has no unit.

However, if the two quantities are not in the same units, we convert them into the same units before comparing.

2. Two quantities can also be compared using percentages. By percentage, we mean a fraction where the denominator is 100. The numerator of the fraction is called the rate percent.

For example:  \(\frac{1}{5}\) means 5%. The symbol % is often used to express ‘percent’ (p. c.).

3. To convert the ratio into a percentage, we convert it into a fraction whose denominator is 100. [or we multiply by 100 and employ the % sign.]

4. To convert percentages in \(\frac{5}{100}}\) to fractions, we divide the numerator by 100 and express it in the lowest form.

For example : 5% =\(\frac{1}{20}\)

Read and Learn More NCERT Solutions For Class 8 Maths

.
5. In the unitary method, we find the value of one unit from the given value of some units and then we find the value of the required number of units.

Question 1. In primary school, the parents were asked about the number of hours they spend per day helping their children to do homework. 90 parents helped for hour to \(\frac{1}{2}\) hours to 1 \(\frac{1}{2}\) . The distribution of parents according to the time for which, they said they helped is given in the adjoining figure; 20% helped for more than 1\(\frac{1}{2}\) ho per day; 30% helped for \(\frac{1}{2}\)  an hour to 1 \(\frac{1}{2}\) hours,  50% did not help at all.

NCERT Solutions For Class 8 Maths Chapter 7 Comparing Quantities Parents Accroding To The Time Said Helped

Using this, answer the following:

  1. How many parents were surveyed?
  2. How many said that they did not help
  3. How many said that they helped for more than 1\(\frac{1}{2}\) hour?

Solution:

1. According to the question

⇒ \(\frac{30}{100}\) of number of parents surveyed = 90

Number of parents surveyed = 90

Number of parents surveyed

= \(\frac{90 \times 100}{30}\)

= 300

Hence, 300 parents were surveyed.

2. A number of those parents said that they did not help

= 50% of 300

= \(300 \times \frac{50}{100}\)

= 150

Hence, 150 parents said that they did 30% not help.

3. A number of those parents said that they helped for more than 1\(\frac{1}{2}\) hour

= 20% of 300 = \(300 \times \frac{20}{100}\)

= 60

Hence, 60 parents said that they helped for more than 1/[latxe]\frac{1}{2}[/latex] hours

Note: ‘Of’ means multiplication

NCERT Solutions For Class 8 Maths Chapter 7 Comparing Quantities Exercise 7.1

Question 1. Find the ratio of the following:

  1. The speed of a cycle 15 km per hour to the speed of a scooter 30 km per hour.
  2. 5 m to 10 km
  3. 50 paise to 5

Solution:

1. The speed ofa a cycle 15 km per hour to the speed of a scooter 30 km per hour

= 15 km per hour: 30 km per hour

= 15: 30

= \(\frac{15}{30}=\frac{1}{2}\) Or 1:2

2. 5 m to 10 km

10 km = 10 × 1000 m = 10000 m

∴Ratio of 5 m to 10 km

= 5 m: 10000 m

= 5: 10000

⇒ \(\frac{5}{10000}=\frac{1}{2000}\)

= 1:20000

3. 50 paise to ₹ 5

₹ 5 = 5× 100 = 500 paise

A Ratio of 50 paise to ₹ 5

= 50 paise : 500 paise

= 50: 500

= \(\frac{50}{500}=\frac{1}{10}\) Or 1:10

Question 2. Convert the following ratios to percentages

  1. 3:4
  2. 2: 3.

Solution:

1. 3: 4

= \(\frac{3}{4}=\frac{3}{4} \times \frac{25}{25}\)

Making denominator 100

= \(\frac{75}{100}=75 \%\)

Aliter:

⇒  \(3: 4=\frac{3}{4} \times 100 \%\)

= 75 %

2. 2:3

= \(\frac{2}{3}=\frac{2}{3} \times \frac{100}{100}\)

= \(66 \frac{2}{3} \%\)

Aliter:

2: 3 = \(\frac{2}{3} \times 100 \%\)

= \(\frac{200}{3} \%=66 \frac{2}{3} \%\)

Question 3. 72% of 25 students are interested in mathematics. How many are not interested in mathematics?
Solution:

Total number of students = 25

Percentage of students interested in Mathematics s = 72%

Percentage of students who are not interested in Mathematics

= (100 – 72)% = 28%

Number of those students who are not interested in Mathematics

= 28% of 25

= \(\frac{28}{100} \times 25\)

= 7

Hence, 7 students are not interested in or 1: 10 Mathematics.

Question 4. A football team won 10 matches out ofthe total number of matches they played. Iftheir win percentage was 40, then how many matches did they play in all?
Solution:

If 40 matches were won, then the total number of matches played = 100

∴ If 1 match was won, then the total number of matches played

=\(\frac{100}{40}\)

If 10 matches were won, then the total number of matches played

= \(\frac{100}{40} \times 10\)

= 25

Hence, they played 25 matches in all.

Aliter:

According to the question,

40% of (total number of matches) = 10

⇒  \(\frac{40}{100} \times \text { (total number of matches) }\)=10

Total number of matches = \(\frac{10 \times 100}{40}\) = 25

Hence, they played 25 matches in all.

Question 5. If Chameli had ₹ 600 left after spending 75% of her money, how much did she have in the beginning ?’
Solution:

Percentage of money spent = 75%

Percentage of money left = (100- 75)% = 25%

∴ If Chameli had ₹ 25 left, then the money she had in the beginning

= 100

∴  If Chameli had ₹1 left, then the money she had in the beginning

= \(₹ \frac{100}{25}\)

∴ If Chameli has v600 left, then the money she had in the beginning

= \(₹ \frac{100}{25} \times 600\)

=  ₹ 24000

Hence, the money she had in the beginning was ₹ 2400.

Aliter :

According to this question,

25% of total money = ₹ 600

⇒ \(\frac{25}{100}\) of total money = ₹ 600

⇒  \(₹ \frac{600 \times 100}{25}\)

= ₹ 2400.

Hence, the money she had in the beginning was? 2400

Question 6. If 60% of people in a city like cricket, 30% like football and, the remaining like other games, then what percent of the people like other games? If the total number of people is 50 lakh, find the exact number who like each type of game.
Solution:

Percentage of people who like cricket = 60%

Percentage of people who like football = 30%

∴Percentage of people who like other games

= [100 -(60 +30)]%

= (100-90)% = 10%.

Total number of people = 50 lakh

= 50,00,000

∴ Number of people who like cricket

= 60% of 50,00,000

= \(50,00,000 \times \frac{60}{100}\)

= 30,00,000 = 30 lakh

Number of people who like football

= 30% of 50,00,000

= \(50,00,000 \times \frac{30}{100}\)

Number of people who like the other games

= 10% of 50,00,000

= \(50,00,000 \times \frac{10}{100}\)

= 5,00,000 = 5 lakh

Finding Discounts

Discount = Marked Price – Sale Price

Discount per cent = \(\frac{\text { Discount }}{\text { Marked Price }} \times 100 \%\)

Question 1. 4 shop gives 20% discount. What Would the sale price ofeach of these be?

  1. A dress marked at.₹ 120
  2. A pair of shoes marked at ₹ 750
  3. A bag marked at₹ 250.

Solution:

1. A dress marked at ₹120

Marked price ofthe dress = ₹120

Discount rate= 20%
,
∴ Discount = 20% of ₹ 120

20 MarkedDiscountprice × 100

=\(₹ \frac{20}{100} \times 120\)

= ₹ 24

∴ Sale price ofthe dress x 100 % = 4%.

= Marked price – Discount

= ₹ 120 – ₹ 24 = ₹ 96

2. A pair of shoes marked at ₹ 750

Marked price ofthe pair ofshoes= ₹ 750

Discount rate = 20%

Discounts 20% of ₹ 750

= \(₹ \frac{20}{100} \times 750\)

= ₹150

∴ Sale price ofthe pair of shoes

= Marked price – Discount

= ₹ 750 -₹ 150

= ₹600

3. A bag marked at ₹ 250

The marked price of the bag =₹ 250

Discount rate = 20%

Discount = 20% of  ₹ 250

= \(₹ \frac{20}{100} \times 250\)

= ₹50

∴ The sale price of the bag

= Marked price – Discount

=₹250 – ₹ 50 =₹200.

Question 2. If the table marked at f 15,000 is * available for f14, 400. Find the discount given £ and the discount percent.
Solution:

Marked price ofthe table =₹ 15,000

Sale price ofthe table =  ₹ 14,400

∴  Discount given

= Marked price – Sale price

= ₹ 15,000 – ₹14,400

= ₹ 600

∴  Discount per cent

= \(\left(\frac{\text { Discount }}{\text { Marked price }} \times 100\right) \% \)

= \(\left(\frac{600}{15000} \times 100\right) \%=4 \%\)

Question 3. An almirah is sold at ₹5,225 after allowing a discount of 5%. Find its marked price.
Solution:

Let the marked price bet x.

Discount rate = 5%

Discount = 5% of ₹ x

=\(₹ \frac{5}{100} \times x=₹ \frac{x}{20}\)

∴ Sale price

= Marked price -Discount

= \(₹ x-₹ \frac{x}{20}=₹ \frac{19 x}{20}\)

According to the question,

∴ \(\frac{19 x}{20}=5225\)

x = \(\frac{5225 \times 20}{19}\)

x = ₹ 5500

Hence, the marked price of the almirah is ₹5500

Question 4. Try estimating 20% of the same bill amount.

  1. Round of the bill to the nearest tens.
  2. Find the amount of discount.
  3. Reduce the bill amount by the discount amount

Solution:

Your bill in a shop is ₹  577.80.

1. Round off the bill to the nearest tens of ₹ 577.80, i.e., to ₹ 580.

2. Find 20% of this,

⇒  \(₹ \frac{20}{100} \times 580\)

= ₹  116

3. Therefore estimated 20% of the same bill amount is ₹116 or ₹120 (rounded off to the nearest tens).

Question 5. Try finding 15% of  ₹ 375.
Solution:

Find 10% of ₹ 375, i.e.

⇒ ₹\(₹ \frac{10}{100} \times 375\)

= ₹ 37.5

= ₹  40

(rounded off to the nearest tens)

2. Take half of this, i.e \(\frac{1}{2} \times ₹ 40\)

= ₹ 20

(3) Add (2) and (1) to get ₹ 60.

Therefore, an estimated 15% of 7 375 is

= 60

Sales Tax/Value Added Tax/Goods And Services Tax

Sales tax was charged at a specified rate on the sale price of an item by the state government and was added to the bill amount. It is different for different items and also for different states.

  • Amount of Sales Tax = Tax% of bill amount
  • These days, the prices include the tax known as Value Added Tax (VAT).
  • From July 1, 2017, the Government of India introduced GST which stands for Goods and Services Tax which is levied on the supply of goods or services or both

Question 1. Find the buying price of each of the following when 5%STis added to the purchase

  1. A towel at ₹ 50
  2. Two bars of soap at ₹ 35 each
  3. 5 kg of flour at ₹15 per kg.

Solution:

1. Cost of the towel

Rate of ST = 5%

∴ ST =5% of 7 50

=₹ \(\frac{5}{100} \times 50\)

= ₹ 2.50

∴ Buying price of the towel

= Cost of the towel + ST

= ₹ 50 + ₹ 2.50 =₹ 52.50.

2. The cost of two bars of soap

= ₹35 × 2 = ₹ 70

Rate of  ST= 5%

ST = 5% of ₹ 70

= \(₹ \frac{5}{100} \times 70\)

= ₹ 3.50

∴ Buying price of two bars of soap

= Cost of two bars of soap + ST

= ₹ 70 +₹ 3.50 = ₹ 73.50

3. Cost of 5 kg of flour

= ₹ 15 × 5 = ₹ 75

Rate of ST = 5%

ST = 5% of ₹ 75

= \(₹ \frac{5}{100} \times 75\)

= ₹ 3.75

∴ The buying price of 5 kg of flour + ST

= Cost of 5 kg of flour + ST

= ₹ 75 +₹ 3.75

=₹ 78.75.

Question 2. If 8% is included in the prices, find the original price of

  1. A TV bought for ₹13,500
  2. A shampoo bottle was bought for ₹ 180.

Solution:

1. When the price of a TV including

VAT is ₹108,

original price = ₹ 100

When the price of the TV including VAT is

₹ 1, original price = ₹ \( \frac{100}{108}\)

The price of a TV including VAT is ₹ 13500,

Original price = \(₹ \frac{100}{108} .\)

= ₹ 12500

Hence, the original price of the TV Is ₹ 12500.

Aliter:

Price of TV including VAT

= ₹ 13500

Rate of VAT = 8%

Let, the original price of the TV be ₹x

Then,

x+8 %  of x =₹ 13500

⇒ \(x+\frac{8}{100} x =₹ 13500 \)

⇒ 1+ \(\frac{8}{100}\)x =₹ 13500

⇒  \(\frac{108}{100} x =₹ 13500\)

= ₹ \(\frac{13500 \times 100}{108}\)

Hence, the original price of the T.V. is ₹12500.

2. When the price of a shampoo bottle including VAT is ₹ 108,

Original price = ₹ 100

When the price of a shampoo bottle including VAT is ₹1, the original price

= \(₹ \frac{100}{108} .\)

∴ The price of a shampoo bottle including VAT is? 180,

Original price = \(=₹ \frac{100}{108} \times 180 \)

⇒ \(₹ \frac{100 \times 5}{3}=₹ \frac{500}{3}\)

=\(₹ 166 \frac{2}{3}\)

Hence, the original price ofthe shampoo Hence, the original price ofthe shampoo bottle is \(166 \frac{2}{3}\)

Alitor: Price of shampoo bot t lo including

VAT = ₹ 180

Rate of VAT = 8%

Let the original price of shampoo bottle be ₹x

Then,

x+ 8%of x = ₹180

⇒ \(x+\frac{8}{100} x=₹ 180\)

⇒  \(\left(1+\frac{8}{100}\right) x=₹ 180\)

⇒  \(\frac{108}{100} x=₹ 180\)

x =\(₹ \frac{180 \times 100}{108}\)

x = \(₹ \frac{500}{3}=₹ 166 \frac{2}{3}\)

Hence, the original price ofthe shampoo bottle is \(166 \frac{2}{3}\)

NCERT Solutions For Class 8 Maths Chapter 7 Comparing Quantities Exercise 7.2

Question 1. During a sale, a shop offered a discount of 10% on the marked prices of all the items. W7iat would a customer have to pay for a pair of jeans marked at ₹ 1450 and two shirts marked at f850 each
Solution:

For a pair of jeans

Marked price = ₹ 1450

Discount rate = 10%

Discount = 10% of marked price

= 10% of  ₹ 1450

= \(₹ \frac{10}{100} \times 1450\)

= ₹ 145

Sale price – Marked price – Discount

= ₹1450-₹ 145

= ₹ 1305

For two shirts

Marked price =₹ 850 × 2 = ₹ 1700

Discount rate = 10%

Discount = 10% of marked price

= 10% of  ₹ 1700

=₹ \(\frac{10}{100} \times 1700=₹ 170\)

Sale price

= Marked price – Discount

= ₹1700 – ₹70 =₹1530

Total payment made by the customer

= ₹ 1305 + ₹ 1530 =₹ 2835

Hence, the customer will have to pay. 2835 for a pair of jeans and two shirts.

Question 2. The price ofa W is f13,000. The sales tax charged on it is at the rate of 12%. Find the amount that Vinod will have to pay if he buys it
Solution: 

Price of TV = ₹ 13,000

Sales tax charged on it

= 12% of ₹ 13,000

= \(₹ \frac{12}{100} \times 13,000\)

= ₹ 1560

Amount to be paid

= Price + Sales Tax

= ₹ 13,000 + ₹ 1,560

= ₹14,560.

Hence, the amount that Vinod will haveto pay for the TV if he buys it is ₹14,560

Question 3. Arun bought a pair of skates at a sale where the discount given was 20%. If the amount he pays is  ₹ 1600, find the marked price
Solution:

Let the marked price of the pair of 6  shoes be ₹ 100.

Kate of discount = 20

Discount = 20% of ₹ 100

= ₹ \(\frac{12}{100} \times 100\)

=₹ 20

Amount paid = Marked price -Discount

= ₹ 100 – ₹20 – = ₹ 80

If the amount paid is ₹80,

Then the marked price – \(₹ \frac{100}{80}\)

If the amount paid is ₹ 1, then the marked price = \(₹ \frac{100}{80} \times 1600\)

= ₹ 2000

Hence, the marked price of the pair of shoes is ₹ 2000.
.
Question 4. I purchased a hair dryer for f 5,400 including 8% VAT. Find the price before it was added,
Solution:

Price of hair-dryer including VAT

=  ₹ 5400

VAT rate = 8%

Let the original price be ₹ 100

∴ VAT = ₹ 8

Price including VAT = Original price + VAT

= ₹100 + ₹ 8

= ₹ 108

If the price including VAT is ₹108, then

Original price =  ₹ 100

If the price including VAT is ₹ 1, then

Orginalprice = \(₹ \frac{100}{108}\)

If the price including VAT is  ₹5,400, then

Original price = ₹ \(\frac{100}{108} \times 5,400\)

= ₹ 5000

Hence, ‘the price before VAT was added is ₹5,000

Question 5. An article was purchased for ₹1239 including a GST of 18%. Find the price of the article before GST was added,
Solution:

Let the original price of the article be 100. GST Rate = 18% ,

Price after GST is included

= ₹(100 + 18) = ₹118

If the selling price is? 118 then original price = ₹100

If the selling price is ₹ 1, then

Original Price = \(₹ \frac{100}{108} \times 5,400\)

If the selling price is ₹ 1239, then

original price = \(₹ \frac{100}{118} \times 1239\)

= 100 × 10.5

= ₹ 1050

Hence, the price ofthe article before GST was added is? 1050

Compound Interest

1. Interest:

Interest is the extra money paid by institutions like banks or post offices on money that is deposited (kept) with them. Interest is also paid by people when they borrow money. The money deposited or borrowed is called the principal. Interest is generally given in percent for one year.

2. Simple interest (SI):

The interest is called simple when the principal does not change.

3. The formula for simple interest:

Simple interest on a principal of 1 P at R% rate of interest per year for T years is given by

Simple Interest = \(\frac{\text { Principal } \times \text { Rate } \times \text { Time }}{100}\)

SI =  \(\frac{\text { P R T }}{100}\)

4. Amount :

Amount (A)= Principal (P) + Simple Interest(SI)

Question 1.   Find interest and amount to be paid on 115,000 at 5% per annum after 2 years
Solution:

Interest on₹ 100 for 1 year is = 5

Interest on ₹ for 1 year is

= \(₹ \frac{5}{100}\)

Interest On ₹ for 1 year is

= \(₹ \frac{5}{100} \times 15,000\)

Interest on ₹ 15000 for 2 years is

= \(₹ \frac{5 \times 15,000 \times 2}{100}\)

= ₹  1,500

∴ Amount = Principal + Interest

=  ₹ 15,000 + ₹ 1,500

= ₹ 16,500

Aliter:

Here, P = ₹ 15,000

R = 5% per annum

T = 2 years

∴ \(\text { S.I. }=\frac{\text { PRT }}{1000}\)

= \(₹ \frac{15000 \times 5 \times 2}{100}\)

₹ 1500

Amount = Principal (P) + SimpleInterest (SI)

= ₹ 15000 + ₹ 1500

= ₹ 1,500 = ₹ 16500

Deducing A Formula For Compound Interest

A = \(\mathrm{P}\left(1+\frac{\mathrm{R}}{100}\right)^n\)

where P = Principal

R = Rate of interest per annum compounded annually

n = Number of years

A = Amount

CI = A-P

Question 1. Find Cl on a sum of ₹ 8000 for 2 years at 5% per annum compounded annually.
Solution:

Here,

P =₹ 8000

R = 5%p.a.

n = 2 years

A = \(\mathrm{P}\left(1+\frac{\mathrm{R}}{100}\right)^n\)

= \(8000\left(1+\frac{5}{100}\right)^2\)

= \(8000\left(1+\frac{1}{20}\right)^2\)

= \(8000\left(\frac{21}{20}\right)^2 \)

= \(8000 \times \frac{21}{20} \times \frac{21}{20}\)

= 20 × 21 × 21

=₹ 8820

Cl = A – P

= ₹ 8820 – ₹ 8000

= ₹ 820.

∴ Hence, the compound interest is ₹ 820

Applications Of Compound Interest Formula

We use the compound interest formula to find

  1. Increase (or decrease) in population.
  2. The growth of a bacteria if the rate of growth is known.
  3. The value of an item, if its price increases or decreases in the intermediate years.

Note: For increase, R is positive and for decrease, R is negative.

Question 1. A machinery worth f 10,500 depreciated by 5%. Find its value after one year.
Solution:

P = ₹ 10,500

R = -5% per annum

There is depreciation

n = year

A = \(P\left(1-\frac{R}{100}\right)^n \)

= \(10,500\left(1-\frac{5}{100}\right)\)

= \(10,500\left(1-\frac{1}{20}\right) \)

= \(10,500 \times \frac{19}{20}\)

= 9,975

Hence value after 1 year is ₹ 9,975.

Question 2. Find the population ofa city after 2 years, which is at present 12 lacks, if the rate of increase is 4%.
Solution:

P = 12,00,000

R = 4% per annum

n = 2years

A = \(P\left(1+\frac{R}{100}\right)^n\)

= \(12,00,000\left(1+\frac{4}{100}\right)^2 \)

= \(12,00,000\left(1+\frac{1}{25}\right)^2\)

= \(12,00,000\left(\frac{26}{25}\right)^2 \)

= \(12,00,000 \times \frac{26}{25} \times \frac{26}{25} \)

= 12,97,920

Hence, the population of the city after 2 years is 12,97,920

Question 3. Calculate the amount and compound interest on

  1. 10,800 for 3 years at \(2 \frac{1}{2} \)% perannum compounded annually.
  2. ₹ 18,000 for  \(2 \frac{1}{2} \)% years at  perannum compounded annually.
  3. ₹  62,500 for \(1 \frac{1}{2} \)% years at  perannum compounded half-yearly.
  4. ₹  8,000 for 1 year at   9% per annum compounded half yearly.You could use the year-by-year calculation to verify).
  5. ₹  10,000 for a year at 8% per annum compounded halfyearly

Solution:

1. By using year-by-year calculation

Here, P =₹ 10800

R = ₹ 12% per annum

T = ₹ year

SI on ₹ 10,800 at \(12 \frac{1}{2}\) per annum for

= \(10,800 \times \frac{25}{2} \times \frac{1}{100}\)

= ₹ 1,350

∴ Amount at the end of 1st year

= ₹ 10,800 + ₹ 1,350 (A = P + SI)

= ₹ 12,150

= Principal for 2nd year.

SI on ₹ 12,150 at \(12 \frac{1}{2} \% \)per annum

For 1 year = \(\frac{\text { PRT }}{100}\)

=’\(12,150 \times \frac{25}{2} \times \frac{1}{100}\)

= ₹  1,518.75

∴Amount at the end of the 2nd year

= ₹ 12,150+ ₹ 1,518.75

= ₹ 13,608.75

= Principal for 3rd year

SI on  ₹ 13,668.75 at 12\(\frac{1}{2}\)% per annum

For 1year =\(\frac{P R T}{100}\)

=\(13,668.75 \times \frac{25}{2} \times \frac{1}{100} \)

=\(13,668.75 \times \frac{1}{8}\)

= ₹1,708.59

Amount at the end of the year

= ₹13,668.75 + ₹ 1,708.59

=₹ 15,377.34

This is the required amount. Now, CI = ₹15,377.34 – ₹ 10,800

= ₹ 4,577.34

By using the compound interest formula

P =  ₹ 10,800

R = 12\(\frac{1}{2}\) per annum

= \(\frac{25}{2}\)% per annum

n = 3 years

A = \(\mathrm{P}\left(1+\frac{\mathrm{R}}{100}\right)^n \)

= \(10,800\left(1+\frac{25}{2 \times 100}\right)^3 \)

= \(10,800\left(1+\frac{1}{8}\right)^3\)

= \(10,800\left(\frac{9}{8}\right)^3\)

= \(10,800 \times \frac{9}{8} \times \frac{9}{8} \times \frac{9}{8}\)

= ₹  15,377.34

2. By using year-by-year calculation

Here

P = ₹ 18000

R = ₹ 10% per annum

T = ₹ year

SI on ₹ 18,000 at 10% p.a. for 1 year

= \(\frac{\mathrm{PRT}}{100}\)

= \(\frac{18,000 \times 10 \times 1}{100}\)

= ₹ 1,800

∴ Amount at the end of the year

= ₹ 18,000 + ₹ 1,800

= ₹ 19,800

= Principal for 2nd year

SI on ₹ 19,800 at 10% p.a. for 1 year

= \(\frac{19,800 \times 10 \times 1}{100}\)

= ₹ 1,980

Amount at the end of 2nd year

= ₹ 19,800 + ₹1,980

= ₹ 21,780

= Principal for 3rd year

SI on ₹ 21,780 at 10% p.a. for \(\frac{1}{2}\)year

⇒ \(\frac{\mathrm{PRT}}{100}=\frac{21,780 \times 10 \times 1}{2 \times 100}\)

= ₹1089

∴ Amount at the end of \(2 \frac{1}{2}\) year

= ₹ 21,780 +₹1,089

= ₹ 22,869

This is the required amount.

Now, CI = ₹ 22,869 – ₹ 18,000

= ₹ 4,869

By using the compound interest formula

P = ₹ 18,000

R = 10% p.a.

n = 2 years

A = \(P\left(1+\frac{R}{100}\right)^n \)

= \(18,000\left(1+\frac{10}{100}\right)^2 \)

= \(18,000\left(1+\frac{1}{10}\right)^2 \)

= \(18,000\left(\frac{11}{10}\right)^2\)

=18,000  × \(\frac{11}{10} \times \frac{11}{10}\)

=₹ 21,780

SI on? 21,780 at 10% p.a. for \(\frac{1}{2}\) year

=\(\frac{21,780 \times 10 \times 1}{2 \times 100}\)

= ₹ 1089

Amount at the end of \(\frac{1}{2}\) years

= ₹21,780 + ₹ 1,089

= ₹ 22,869

Cl = A- P

= ₹22,869- ₹ 18,000

= ₹ 4,869

3. By using half-year by half-year calculation

Here, P = ₹ 62500

R = 8% per annum

= \(\frac{8}{2}\) per half-year

T = 1 year

∴ SI on ₹ 62,500 at 8% p.a. for 1st half year

= \(\frac{P R T}{100} \)

= \(\frac{62,500 \times 8 \times 1}{2 \times 100}=\frac{62500}{25}\)

= ₹ 2,500

∴  Amount at the end of 1st half-year

= ₹ 62,500 +₹ 2,500

= ₹65,000

= Principal for 2nd half year ₹ 65,000 at 8% p.a. for 2nd half-year

= \(\frac{P R T}{100}\)

= \(\frac{65,000 \times 8 \times 1}{2 \times 100}\)

= ₹2,600

Amount at the end of 2nd halfyear

= ₹ 65,000 + ₹ 2,600

= ₹ 67,600

= Principal for 3rd halfyear

SI on ₹ 67,600 at 8% p.a. for 3rd half-year

= \(\frac{P R T}{100} \)

⇒ \(\frac{67,600 \times 8 \times 1}{2 \times 100}\)

= ₹ 24704

∴ Amount at the end of 3rd half-year

= ₹ 67,600 + ₹ 2,704

= ₹ 70,304

This is the required amount.

Now, Cl = Amount – Principal

= ₹ 70,304 – ₹ 62,500

= ₹7,804

Cl = ₹ 2,500 + ₹ 2,600 +₹ 2,704

= ₹ 7,804

By using the compound interest 2 formula

P =₹ 62,500

= \(\frac{1}{2} \times 8 \% \text { per half year } \)

= 4% per half-year

n = \(1 \frac{1}{2} \text { year } \)

= \(1 \frac{1}{2} \times 2 \text { half years }\)

3 half years

A =\(\mathrm{P}\left(1+\frac{\mathrm{R}}{100}\right)^n \)

= \(62,500\left(1+\frac{4}{100}\right)^3 \)

= \(62,500\left(1+\frac{1}{25}\right)^3 \)

= \(62,500\left(\frac{26}{25}\right)^3 \)

= \(62,500 \times \frac{26}{25} \times \frac{26}{25} \times \frac{26}{25}\)

= \(4\times 26 \times 676\)

= 4 × 26 × 676

= ₹ 70,304

Cl = A- P = ₹ 70,304- ₹ 62,500

=₹ 7804.

By using the compound interest formula

P =₹  62,500

R = 8%p.a

= \(\frac{1}{2} \times 8 \% \text { per half year }\)

1% per half-year

n =\(\frac{1}{2}\)

= 1 \(\frac{1}{2} \times 2\)

= 3 half years

A = \(P\left(1+\frac{R}{100}\right)^n\)

= \(62,500\left(1+\frac{4}{100}\right)^3 \)

= \(62,500\left(1+\frac{1}{25}\right)^3\)

= \(62,500\left(\frac{26}{25}\right)^3\)

=\(62,500 \times \frac{26}{25} \times \frac{26}{25} \times \frac{26}{25}\)

4. By using the compound interest formula

P = ₹ 8,000

R = 9% p.a.

= \(\frac{9}{2}\) % per half-year

= \(8,000\left(1+\frac{9}{2 \times 100}\right)^2\)

= \(8,000\left(1+\frac{9}{200}\right)^2\)

=\(8,000\left(\frac{209}{200}\right)^2 \)

= \(8,000 \times \frac{209}{200} \times \frac{209}{200} \)

\(=\frac{209 \times 209}{5}\)

= 4 ×  26× 676

= ₹  70,304

Cl = A- P = ₹ 70,304 – ₹ 62,500

= ₹ 7804

By using compound interest formula

P=8000

R=9%p.a

=\(\frac{9}{2} \%\)per halfyear

n=1 year = 1 × 2 half-year

= 2 half years

∴ A =\(\mathrm{P}\left(1+\frac{\mathrm{R}}{100}\right)^n\)

= \(8,000\left(1+\frac{9}{2 \times 100}\right)^2\)

= \(8,000\left(1+\frac{9}{200}\right)^2 \)

= \(8,000\left(\frac{209}{200}\right)^2\)

= \(8,000 \times \frac{209}{200} \times \frac{209}{200} \)

=\(\frac{209 \times 209}{5}\)

= ₹8,736.20

Cl = A-P

= ₹ 8,736.20 – ₹8,000

=₹ 736.20

5. By using the compound interest formula

P = ₹ 10,000

= 8% per annum

= \(\frac{8}{2}\) per half-year

= 4 % per half-year

n = year = 1 × 2 half years

= 2 half years

= \(P\left(1+\frac{R}{100}\right)^n \)

= \(10,000\left(1+\frac{4}{100}\right)^2 \)

=\(10,000\left(1+\frac{1}{25}\right)^2 \)

= \(10,000 \times \frac{26}{25} \times \frac{26}{25}\)

16 × 26 × 26

= ₹ 10,816

∴ Cl = A-P

= ₹ 10,816 – ₹ 10,000

= ₹ 816

Question 4.  Kamala borrowed ₹26,400from a Bank to buy a scooter at a rate of 15% p.a. compounded yearly. What amount will she pay at the end of the year and 4 months to clear the loan 

(Hint: Find A for 2 years, interest is compounded yearly and then find SI on the 2nd year amount for \(\frac{4}{12}\)
Solution:

P = ₹ 26,400

R = 15% p.a.

n = 2 years

A = \(P\left(1+\frac{R}{100}\right)^n \)

= \(26,400\left(1+\frac{15}{100}\right)^2 \)

= \(26,400\left(1+\frac{3}{20}\right)^2 \)

= \(26,400\left(\frac{23}{20}\right)^2\)

= \(26,400 \times \frac{23}{20} \times \frac{23}{20}\)

= \(66 \times 23 \times 23 \)

= ₹ 34,914

S.I. on X 34,914 at 15% p.a. for 4 months

=\(\text { (i.e., } \left.\frac{4}{12} \text { year, i.e., } \frac{1}{3} \text { year }\right) \)

= \(\frac{\mathrm{P} \times \mathrm{R} \times \mathrm{T}}{100} \)

= \(34,914 \times \frac{15}{100} \times \frac{1}{3} \)

= \(\frac{34,914 \times 15 \times 1}{3 \times 100}\)

= 1,745.70

Required amount

= ₹ 34,914 +₹ 1,745.70

= ₹ 36,659.70

Hence, the amount that Kavita will pay is ₹ 36,659.70.

NCERT Solutions For Class 8 Maths Chapter 7 Comparing Quantities Exercise 7.3

Question 1. The population of the place increased to 54,000 in 2003 at a rate of 5% per annum. Find the population in 2001. What, would be its population in 2005?
Solution:

1. Let the population in 2001 be P.

R = 5% p.a.

n = 2 years

A = \(P\left(1+\frac{R}{100}\right)^n=P\left(1+\frac{5}{100}\right)^2 \)

= \(P\left(1+\frac{1}{20}\right)^2=P\left(\frac{21}{20}\right)^2\)

According to the question

⇒ \(\mathrm{P}\left(\frac{21}{20}\right)^2=54,000\)

P = \(54,000\left(\frac{20}{21}\right)^2\)

= \(54,000 \times \frac{20}{21} \times \frac{20}{21} \)

= 48,980(approx.)

Hence, the population in 2001 was 48,980 (approx.)

Initial population in 2003

(P) = 54,000

R = 5% p.a.

n = 2 years (2005 – 2003 = 2)

A=\(\left(1+\frac{R}{100}\right)^n\)

= \(54,000 \times \frac{20}{21} \times \frac{20}{21}\)

= \(54,000\left(1+\frac{1}{20}\right)^2\)

= \(54,000\left(\frac{21}{20}\right)^2\)

= \(54,000 \times \frac{21}{20} \times \frac{21}{20}\)

=\(135 \times 21 \times 21=59,535\)

Hence, the population in 2005 would be 59,535. .1

Question 2. In a Laboratory, the count of bacteria in a certain experiment was increasing at the rate of 2.5% per hour. Find the bacteria at the end of2 hours, if the count was initially 5,06,000
Solution:

The initial count of bacteria

(P) = 5,06,000

R = 2.5% per hour

n = 2 hours

=\(\mathrm{P}\left(1+\frac{\mathrm{R}}{100}\right)^n \)

=\(5,06,000\left(1+\frac{2.5}{100}\right)^2 \)

= \(5,06,000\left(1+\frac{1}{40}\right)^2 \)

= \(5,06,000\left(\frac{41}{40}\right)^2 \)

= \(5,06,000 \times \frac{41}{40} \times \frac{41}{40} \)

= \(\frac{1265}{4} \times 41 \times 41\)

= 531616.25

= 5,31,616 (approx.)

Hence, the bacteria count at the end UJ of 2 hours is 5,31,616 (approx.).

Question 3. A scooter was bought at ₹42,000. Its value depreciated at the rate of 8% per annum. Find its value after one year.
Solution:

NCERT Solutions For Class 8 Maths Chapter 7 Comparing Quantities Scooter Bought

Initial value of the scooter

P =₹ 42,000

R = -8% per annum

I v There is depreciation

A = \(P\left(1-\frac{R}{100}\right)^n \)

=\(42,000\left(1-\frac{8}{100}\right)^1\)

= \(42,000\left(1-\frac{2}{25}\right)\)

= \(42,000 \times \frac{23}{25}\)

= \(1680 \times 23=₹ 38,640\)

= 1680 × 23 = ₹ 38,640

Hence, its value after 1 year is ₹ 38,640.

NCERT Solutions For Class 8 Maths Chapter 7 Comparing Quantities Multiple Choice Questions

Question 1. The ratio of 50 paise to  ₹ 1 is

  1. 1:2
  2. 2:1
  3. 1:1
  4. 1:5.

Solution: 1. 1:2

1 = 100 paise

50paise : 100 paise or 50 : 100 or 1:2

Question 2. The ratio of 10 m to 1 km is

  1. 1:10
  2. 10:1
  3. 1:100
  4. 100 :1.

Solution: 3.1:100

1 km = 1000 m

10 m: 1000 m or 10: 1000 or 1: 100

Question 3. The ratio of 10 km per hour to 30 km per hour is

  1. 3:1
  2. 1: 2
  3. 1: 3
  4. 2:1.

Solution: 3. 1: 3

10 : 30 = 1:3

Question 4. The ratio 1: 4 converted to percentage is

  1. 50%
  2. 25%
  3. 75%
  4. 4%.

Solution: 2. 25%

1: 4 = \(\frac{1}{4} \times 100 \%=25 \% \text {. }\)

Question 5. The ratio 4:25 converted to percentage is

  1. 8%
  2. 4%
  3. 16%
  4. 25%.

Solution: 3. 16%

4: 25 = \(\frac{4}{25} \times 100 \%=16 \%\)

Question 6. The fraction ^ converted to a percentage is

  1. 20%
  2. 30%
  3. 40%
  4. 50%.

Solution: 3.40%

⇒ \(\frac{2}{5}\) = \(\frac{2}{5} \times 100 \%=40 \% \text {. }\)

Question 7. The fraction\(\frac{1}{8}\) converted to percentage

  1. 8
  2. 12%
  3. 25%
  4. 8%
  5. 16%.

Solution: 1. 8

⇒ \(\frac{1}{8}=\frac{1}{8} \times 100 \%=12 \frac{1}{2} \%\)

Question 8. Out of 40 students in a class, 25% passed. How many students passed?

  1. 20
  2. 10
  3. 30
  4. 40.

Solution: 2.10

Out of 100, passed = 25

Out of40, passed =\(\frac{25}{100} \times 40\)

⇒ \(25 \% \text { of } 40=\frac{25}{100} \times 40\)

= 10

Question 9. Out of 100 students in a class, 30% like to watch T.V. How many students like to watch T.V.?

  1. 70
  2. 50
  3. 60
  4. 30.

Solution: 4.30

Number of students who like to watch

TV =30% of\(100=\frac{30}{100} \times 100\)

= 30

Question 10. There are 50 students in a class of which 40 are boys and the rest are girls. The ratio of the number of boys and the number of girls is

  1. 2: 3
  2. 1:5
  3. 4:1
  4. 2:5.

Solution: 3. 4:1

Number of Girls = 50 – 40 – 10

Required ratio = 40 : 10 = 4 : 1

Question 11. 40% of 50 students in a class are good at Science. How many students are not good at Science?

  1. 20
  2. 30
  3. 10
  4. 40.

Solution: 2. 30

Out of 100, good at Science = 40

Out of 50, good at Science

Number of students good at science

= \(40 \% \text { of } 50=\frac{40}{100} \times 50=20\)

Or

Number of students good at science

= 40% of 50

= \(40 \% \text { of } 50=\frac{40}{100} \times 50=20\)

Number of students not good at science

= 50-20 = 30

Question 12. Apala has ₹ 200 with her. She spent 80% amount she had. How much money is left with her?

  1. 10
  2. 20
  3. 30
  4. 40.

Solution: 4.40

Out of  100, money spent =? 80

Out of? 200, money spent

= \(\frac{80}{100} \times 200\)

Money spent = 80% of₹ 200

= \(200 \times \frac{80}{100}\)

= ₹ 160

Money left = ₹ 200 – ₹  160

= ₹ 40.

Question 13. A toy marked at t 40 is available for 132. What percent discount is given on the marked price?

  1. 10%
  2. 20%
  3. 25%
  4. 40%.

Solution: 2.20%

Discount = ₹  40 – ₹ 32 = ?8

Discount on ₹  40 = ? 8

Discount on ₹  100

⇒\(\frac{80}{40}\) x100

= ₹ 20

Question 14. Find the simple interest on? 1000 for 2 years at 8% per annum.

  1. ₹ 80
  2. ₹ 40
  3. ₹ 120
  4. ₹ 60.

Solution: 4. ₹ 60

S.I = \(\frac{1000 \times 2 \times 8}{100}\)

= ₹ 160

Question 15. A sofa-set was bought for l 10000. Its value depreciated at the rate of 10% per annum. Find its value after one year.

  1. ₹11000
  2. ₹19000
  3. ₹10000
  4. ₹1000.

Solution: 2. ₹19000

Depreciation in 1 year =\(10000 \times \frac{10}{100}\)

= 1000

Value after 1 year

= 10000 – 1000 =₹ 9000.

Question 16. There are 1275 trees in Chaudhary Farm. Out of these 36% trees are of fruits. How many trees of fruits are there in Chaudhary Farm?

  1. 459
  2. 549
  3. 945
  4. 954.

Solution: 1.459

Required number = \(\frac{1275 \times 36}{100}\)

= 459

Question 17. The quantity of protein in a particular variety of pulse is 25%. Find the amount of protein in 4 kg of pulse.

  1. 1 kg
  2. 2 kg
  3. 3 kg
  4. kg.

Solution: 1. 1 kg

Amount of protein =\(4 \times \frac{25}{100}\)

= 1kg

Question 18. In a mixture, the amount of zinc is 45%. Find the amount of zinc in the 400 g mixture.

  1. 60 g
  2. 120 g
  3. 180 g
  4. 200 g.

Solution: 3. 180 g

Amount of zinc =\(400\times \frac{45}{100}\)

= 80kg

Question 19. In a school out of 340 students, 55% of students are of Science. The remaining students are of Commerce. Find the number of students of Commerce.

  1. 135
  2. 153
  3. 315
  4. 140.

Solution: 2. 153

Number of Science students

= \(340 \times \frac{55}{100}\) = 187

Number of students of Commerce

= 340 – 187 = 153

Question 20. The salary of Manish is ₹ 10000. His salary gets increased by 10%. Find his increased salary.

  1. ₹ 9000
  2. ₹ 11000
  3. ₹ 8000
  4. ₹ 12000.

Solution: 2.₹ 11000

Increase in salary

= ₹  \(10000 \times \frac{10}{100}=₹ 1000\)

Question 21. A shopkeeper purchased 2 refrigerators for ₹ 9800 and  ₹ 8200 respectively. He sold them for ₹ 16920. Find loss%.

  1. 2%
  2. 4%
  3. 5%
  4. 6%.

Solution: 4.6%

Total C.P. =₹ 9800 + ₹ 8200 = ₹18000

S.P. = ₹ 16920

Loss = ₹18000 – ₹16920 = ₹1080

=\(\frac{1080}{18000} \times 100 \%=6 \%\)

Question 22. In selling a plot of land for ₹ 61200, a profit of 20% is made. The cost price of the plot is

  1. ₹ 51000
  2. ₹ 50000
  3. ₹  49000
  4. ₹  52000.

Solution: 1. ₹ 51000

If S.P. is ₹120(100 + 20),

C.P. =₹ 100

If S.P. is ₹ 61200,

C.P =\(\frac{100}{120} \times 61200\)

=  ₹ 51000

Question 23. The simple interest on X 2000 for 4 years is ? 400. The rate percent of interest is

1. \(\frac{2000 \times 100}{400 \times 4}\)

2. \(\frac{400 \times 4}{2000 \times 10} \)

3. \(\frac{400 \times 100}{2000 \times 4}\)

4. \( \frac{2000 \times 4}{400 \times 100} \)

Solution: 3.\(\frac{400 \times 100}{2000 \times 4}\)

⇒ \(\frac{2000 \times 4 \times \mathrm{R}}{100}=400\)

R= \(\frac{400 \times 100}{2000 \times 4}\)

Question 24. The simple interest of ₹ 500 at the rate of 5% is ₹ 100. This interest is of the time

  1. 1 year
  2. 4 years
  3. 10 years
  4. 20 years.

Solution: 2. 4 years

⇒ \(\frac{500 \times 5 \times \mathrm{T}}{100}=100\)

⇒ \(\mathrm{T}=\frac{100 \times 100}{500 \times 5}=4\)

Question 25. The S.I. of ₹ 100 for 1 year at the rate of 3 paise per rupee per month is

  1. ₹ 30
  2. ₹ 36
  3. ₹ 24
  4. ₹ 8.

Solution: 2.₹ 36

S.I. = 100 × 1 × (3 ×  12)

= 3600 paise

= ₹ 36

NCERT Solutions For Class 8 Maths Chapter 7 Comparing Quantities True-False

1. VAT is always calculated on the selling price – True

2. A machinery worth .₹ P is depreciated by R% per annum. Its value after 1 year will be \(\mathrm{P}\left(1+\frac{\mathrm{R}}{100}\right)\)– False

3. Comparison of two quantities by division is called ratio-True

NCERT Solutions For Class 8 Maths Chapter 7 Comparing Quantities Fill In the Blanks

1. The overhead changes are added to → CP

2. If the conversion period is not specified, then it is taken as  Oneyear

3. If a discount of ₹ 4y is available on the market price of ₹ 8y, then the discount percent – 50%

4. If — % of a number is 154, then the number is  4900

5. Find 10% of [100- 20% of 300]  4

6. Explain 0.1234 as a percentage  12.34

7 A jacket work for? 4000 is offered for sale at? 3600. What per cent discount is offered during the sale  10%

8. Apala got 100 marks out of 200 and Meenu got 120 works out of 300. Whose performance is better  Apalas.

NCERT Solutions For Class 8 Maths Chapter 5 Squares And Square Roots

NCERT Solutions For Class 8 Maths Chapter 5 Squares And Square Roots Important Points

1. The numbers expressed as the product of the number with itself are called square numbers or perfect squares.

For example, 1 = 1×1 = 1²

4 = 2 ×2 = 2²

9 = 3 × 3 = 3²

16 = 4 × 4 = 4²

25 = 5 × 5 = 5²………………..

2. If a natural number m can be expressed as n², where n is also a natural number, then, m is called a square number.

For example, 100 is the square of 10, as

Read and Learn More NCERT Solutions For Class 8 Maths

100 – 10 × 10 – 10².

So, 100 is a square number.

121 is the square of 11, as

121 – 11 × 11 = n²

So, 121 is a square number.

Question 1. Consider the following numbers and their squares.

NCERT Solutions For Class 8 Maths Chapter 5 Squares And Square Roots Numbers And Their Squares

From the above table, can we enlist the square numbers between 1 and 100? Are there any natural square numbers up to 100 left out ? From the above table, can we enlist the square numbers between 1 and 100? Are there any natural square numbers up to 100 left out?
Solution:

NCERT Solutions For Class 8 Maths Chapter 5 Squares And Square Roots Numbers And Their Squares Answers

From, the above table, we can enlist the square numbers between 1 and 100.

Such numbers are listed below:

4, 9, 16, 25, 36, 49, 64, and 81 Also, we see that no natural square numbers up to 100 are left out.

Question 2. Find the perfect square numbers between

  1. 30 and 40
  2. 50 and 60.

Solution:

1. We know that  5²= 5 × 5 = 25,  6²= 6 ×  6 = 36 and 7² = 7 ×  7 = 49.

So, the perfect square number between 30 and 40 is 36.

2. We know that 7²2 = 7 × 7 = 49, and 8² = 8 × 8 = 64.

So there is no perfect square number between 50 and 60.

Properties Of Square Numbers

  1. All square numbers end with 0, 1, 4, 5, 6, or 9 only at the unit’s place, i.e., all square numbers have the unit’s digit as 0, 1, 4, 5, 6, or 9 only.
  2. No square number ends with 2, 3, 7 or 8 at the unit’s place.
  3. Square numbers can have only an even number of zeros at the end.
  4. No square number ends with an odd number of zeros at the end.
  5. If a square number ends in 6 then the number whose square it is will end in 4 or 6.

Question 1. Can we say that if a number ends in 0, 1, 4, 5, 6, or 9, then it must be, a square number? Think about it,
Solution:

If a number ends with the digit 0, 1, 4, 5, 6, or 9 at its unit’s place, then it is not necessarily a square number.

For example:

The numbers 10, 21, 34, 325, 146, and 209 are not square numbers although they end with 0, 1, 4, 5, 6, and 9 respectively at their unit’s places.

Note 1: The non-perfect, square numbers end with the digit 2, 3, 7, or 8. i.e., if a number ends with the digit 2, 3, 7, or 8 at its unit place, then it is necessarily a non-perfect square number.

Examples:  32, 42, 43, 53, 63

Note 2: If a number ends with an odd number of zeros at the end, then it is necessarily a non-perfect square number.

Examples: 70, 690, 1430

Question 2.  Can we say whether the following numbers are perfect squares? How do we know?

  1. 1057
  2. 23453
  3. 7928
  4. 222222
  5. 1069
  6. 2061.

Write five numbers which you can decide by looking at their unit’s digit that they are not square numbers.
Solution:

1. 1057 

The number 1057 is not a perfect square because it ends with 7 at the unit’s place whereas the square numbers end with 0, 1, 4, 5, 6, or 9 only at the unit’s place.

2. 23453

The number 23453 is not a perfect square because it ends with 3 at the unit’s place whereas the square numbers end with 0, 1, 4, 5, 6, or 9 only at the unit’s place.

3. 7928

The number 7928 is not a perfect square because it ends with 8 at the unit’s place whereas the square numbers end with 0, 1, 4, 5, 6, or 9 only at the unit’s place.

4. 222222

The number 222222 is not a perfect square because it ends with 2 at the unit’s place whereas the square numbers end with 0, 1, 4, 5, 6, or 9 only at the unit’s place.

5. 1069

The number 1069 ends with 9 at the unit’s place. But we cannot say that 1069 is a perfect square number. It may or may not be a square number.

Also, 30 ×  30 = 900

31 × 31 = 961

32 × 32 = 1024

33  × 33 = 1089

This shows that there is no natural number lying between 1024 and 1089 which is a square number.

As 1024 < 1069 < 1089, i.e., the number 1069 lies between 1024 and 1089, so 1069 is not a square number.

4. The number 2061 ends with 1 at the unit’s place. But we cannot say that 2061 is a perfect square number. It may or may not be a perfect square number.

Also, 45 × 45 = 2025

46 × 46 = 2116

This shows that there is no natural number lying between 2025 and 2116 which is a square number. Since 2025 < 2061 < 2116, i.e., the number 2061 lies between 2025 and 2116, therefore 2061 is not a square number.

The number 2061 ends with 1 at & the unit’s place. But we cannot say that 2061 is a j perfect square number. It may or may not be an O-perfect square number.

Also, 45 × 45 = 2025

46 ×46 = 2116

This shows that there is no natural number lying between 2025 and 2116 which is a square number. Since 2025 < 2061 < 2116, i.e., the number 2061 lies between 2025 and 2116, therefore 2061 is not a square number.

Question 3. Write five numbers that you can decide by looking at their unit’s digit that they are not square numbers.
Solution:

The five numbers that we can decide by looking at their one digit that they are not square numbers are 1032, 453, 5567, 13258, and 13293.

[Note that the non-perfect square numbers end with the digits 2, 3, 7, or 8 at the unit’s place]

Question 4.  Write five numbers that you can not decide just by looking at their unit digit (or unit place) whether they are square numbers or not
Solution:

100000, 3246, 56556, 12331 and 799

Note that the numbers ending with the digits 0, 1, 4, 5, 6, or 9 at the unit’s place may or may not be perfect square numbers.

Question 5. Which of 123², 77², 82², and 161², 109²would end with the digit 1?
Solution:

We know that if a number has 1 or 1 is 9 in the unit’s place, then its square ends with the digit 1.

So, 161² and 109² would end with the digit 1

Question 6. Write the next two square numbers respectively. after 441 which ends in 1 and their corresponding numbers
Solution:

From Table 1 on page 91 ). We find that 441 = 21 x 21= 21s. So, the next two square numbers that end in are

29² (= 841) and 31² ( = 961)and their corresponding numbers are 29 and 31respectively.

[Note that if a number has 1 or 9 in the unit’s place, then its square ends with the digit 1.]

Question 7. Which of the following numbers has the digit 6 at the unit place ?

  1. 19²
  2. 26²
  3. 36²
  4. 34².

Solution:

Note that when a square number ends with digit 6, then the number whose square it is, will have either 4 or 6 at the unit’s place. So,

1. The digit at the unit’s place = 9

∴ 19² would not have 6 in place.

2. The digit at the unit’s place = 4

∴ 24² would have 6 at unit’splace.

3. The digit at the unit’s place = 6

∴ 26² would have 6 at the unit place

Question 8. Can you find more such rules by observing the numbers and their squares
Solution:

Yes; We can find more such rules by observing the numbers and their squares

These are as follows:

  • A square number will end with digit 4 only and only when the number, of which it is the square, ends either with digit 2 or with digit 8.
  • A square number will end with the digit 9 only and only when the number of which, it is square, ends either with digit 3 or with the digit 7.
  • Asquare number will end with the h digit fi only and only when Ilie number, of which it is square, ends with digit f>.
  • A square number will end with digit 1 only mid only when ( ho number, of which it is the Hqunro, ends will) digit 4 or 0,
  • A square number will end with a digit Only and only when the number, of which it is the square, ends with the digit 0.

Question 9. What will be the one digit in the square of the following number?

  1. 1234
  2. 26387
  3. 52698
  4. 99880
  5. 21222
  6. 9106

Solution:

1. 1234

Ending digit = 4 and 4² = 4× 4 = 16

∴ The one digit in the square of the number 1234 will be 6.

2. 26387

Ending digit = 7 and 7² = 7 × 7 = 49

∴ The one digit in the square ofthe number 26387 will be 9.

3. 52698

Ending digit = 8 and 8² = 8×8 = 64

∴ The one digit in the square of the number 52698 will be 4.

4. 99880

Ending digit = 0 and 0² = 0×0 = 0

∴ The one digit in the square ofthe number 99880 will be 0.

5.  21222

Ending digit = 2 and 2² = 2×2 = 4

∴ The one’s digit in the square of the number 21222 will be 4

6. 9106

Ending digit = 6 and 6² = 6 × 6 = 36

∴ The one’s digit in the square of the number 9106 will be 6

Question 10.  If a number contains 3 zeros at the end, how many zeros will its square have? What do you notice about the number of zeros at the end ofthe number and the number of zeros at the end of its square? Can we say that square numbers can only have an even number of zeros at the end?
Solution:

If a number contains 3 zeros at the end, then its square will have 3 × 2 = 6 zeros at the end

The number of zeros at the end of a number

=\(\frac{1}{2}\) ×The number of zeros at the end of its square.

Yes; we can say that square numbers can only have an even number of zeros at the end.

Question 11.  With numbers and their squares. What can you say about the squares of even numbers and squares of odd numbers? 
Solution:

  1. The squares of even numbers are always even numbers.
  2. The squares of odd numbers are always odd numbers.

Question 12. Square of which of the following numbers would be an odd number/an even number? Why?

  1. 727
  2. 158
  3. 269
  4. 1980

Solution:

1. 727

The square of 727 will be an odd number because the number 727 is odd

2. 158

The square of 158 will be an oven number because be number

∴  158 is an even number

3. 269

The square of 269 will be an odd number because the number ‘

∴ 269 is an odd number

4. 1980

The squares of 1980 will be an even number because the number

∴ 1980 is an even number

Question 13. What will be the number of zeros in the square of the following numbers?

  1. 60
  2. 400

Solution:

1. 60

Number of zeros at the end of (the number 60 = 1)

∴ The number of zeros in the square of (be number 60 will be 2 ×1 = 2.

2. 400

Number of zeros at (the end of the number 400 = 2

∴  The number of zeros in the square of the number 400 will be 2 × 2 = 4

Note that the number of zeros at the end of the square of a number = 2 × number of zeros at the end of that number

Some More Interesting Patterns

1. A triangular number is one whose dot patterns can be arranged as triangles. If we combine two consecutive triangular numbers, we get a square number.

2. There are 2n non-perfect square numbers between the squares of the numbers n and (n + 1) which is 1 less than the difference of two squares.

3. The sum of the first n odd natural numbers is n².

4. We can express the square of any odd number as the sum of two consecutive positive integers

Note: \(n^2=\frac{n^2-1}{2}+\frac{n^2+1}{2}\)

Example: \((11)^2=\frac{121-1}{2}+\frac{121+1}{2}\)

= 60 + 61

=121

5. If (n + 1) and (n – 1) are two consecutive even or odd natural numbers, then (n + 1) x (n – 1) = n² – 1

Question 1. How many numbers are there between 6² and 7²
Solution:

Let n = 6

Then, n + 1 = 6 + 1 = 7

∴ There are 2n = 2 × 6 = 12

Nonperfect square numbers between 62 and 72

Question 2. Check for n = 5, n = 6, n = 7 etc., and verify
Solution:

1. n = 5

.’. n + 1 = 6

There are 2n = 2 × 5 = 10 non perfect square numbers between 5² and 6²

These are 26, 27, 28, 29, 30, 31, 32, 33, 34 and 35.

2. n = 6

n + 1 = 7

There are 2n = 2× 6 = 12 nonperfect square numbers between 62 and 72.

These are 37, 38, 39, 40, 41, 42, 43,44, 45, 46, 47 and 48

3. n = 7

n + 1 = 8

There are 2n = 2 × 7 = 14

Nonperfect square numbers between 7² and 8².

These are 50, 51, 52, 53, 54, 55, 56, 57, 58, 59, 60, 61, 62 and 63

Question 3. How many natural numbers lie between 9² and 10² Between 11² and 12²
Solution:

1. Here, n = 9

2n = 2 × 9 = 18.

So, 18 natural numbers lie between 9² and 10².

2. Here, n = 11

2n = 2 × 11 = 22.

So, 22 natural numbers lie between 11² and 12²

Question 4. How many nonsquare numbers lie between the following pairs of numbers?

  1. 100² and 101²
  2. 90² and 91²
  3. 1000² and 1001²

Solution:

1. Here, n = 100

∴ 2n = 2 × 100 = 200.

So, 200 nonsquare numbers lie between the pair of numbers 100² and 101²

2. Here, n = 90

∴ 2n = 2×90 = 180.

So, 180 nonsquare numbers lie between the pair of numbers 90² and 91².

3. Here, n = 1000

∴ 2n = 2 × 1000 = 2000.

So, 2000 nonsquare numbers lie between the pair of numbers 1000² and 1001²

Question 5. 1 + 3 + 5 + 7 […] = 16 = 42 1+3 + 5 + 7+ 9[..]= 25 = 52 1+3 + 5+7 + 9 + 11 […] =36 = 62
Solution:

⇒ 1 + 3 + 5 + 7

[Sum of first four odd numbers]

= 16 = 4²

⇒  1 +3 + 5 + 7 + 9

[Sum of first five odd numbers]

= 25

= 5²

1 + 3 + 5 + 7 + 9+11

[Sum of first six odd numbers]

= 36

= 36

Question 6. Find whether each of the following numbers is a perfect square or not.

  1. 121
  2. 55
  3. 81
  4. 49
  5. 69

Solution:

1. Successively subtract consecutive odd natural numbers starting with l. i.e .. 1. 3, 5, 7. 5), … from 121.

121-1 = 120

120-3 = 117

117-5 = 112

112-7 = 105

105-9 = 96

96-11 = 85

85- 13 = 72

72-15 = 57

57 – 17 = 40

40 – 19 = 21

21 – 21 = 0

This means that

121=1 + 3 + 5 + 7 + 9+11 + 13+15 + 17+19 + 21

i.e., we can express 121 as the sum of consecutive odd numbers starting with 1.

∴ 121 is a perfect square number. number.

2. Successively subtract consecutive odd natural numbers starting with 1, i.e., 1, 3, 5, 7, 9, … from 55.

55 – 1 = 54

54 – 3 = 51

51 – 5 = 46

46 – 7 = 39

39 – 9 = 30

30 – 11 = 19

19 – 13 = 6

6 – 15 = -9

This shows that we are not able to express 55 as the sum of consecutive odd numbers starting with 1

∴ 55 is not a perfect square number

3. Successively subtract consecutive odd natural numbers starting with 1, i.e.,  1, 3, 5, 7, 9, … from 81

81 – 1 = 80

80 – 3 = 77

77 – 5 = 72

72 – 7 = 65

65 – 9 = 56

56 – 11 = 45

45 – 13 = 32

32 – 15 = 17

17 – 17 = 0

This means that

81 =1 + 3 + 5 + 7 + 9+11 + 13 + 15 + 17.

i.e., we can express 81 as the sum of consecutive odd numbers starting with 1.

∴  81 is a perfect square number

4. Successively subtract consecutive odd natural numbers starting with 1, i.e., 1, 3, 5, 7, 9, … from 49

49 – 1 = 48

48 – 3 = 45

45 – 5 = 40

40 – 7 = 33

33 – 9 = 24

24 – 11 = 13

13 – 13 = 0

This means that

9 =1 + 3 + 5 + 7 + 9+11 + 13

We can express 49 as the sum of consecutive odd numbers starting with 1.

∴  49 is a perfect square number.

5. Successively subtract consecutive odd natural numbers starting with 1, i.e. 3. 5. 7. 9. … from 69.

69 – 1 = 68

68 – 3 = 65

65 – 5 = 60

60 – 7 = 53

53 – 9 = 44

44-11= 33

33 – 13 = 20

20 – 15 = 5

5 – 17 = -12

This shows that we are not able to express 69 as the sum of consecutive odd numbers starting with 1
.
∴ 69 is not a perfect square number

Question 7. Express the following as the sum of two consecutive integers.

  1. 21²
  2. 13²
  3. 11²
  4. 19²

Solution:

1. Here,n=21

∴  \(\frac{n^2-1}{2}=\frac{21^2-1}{2}=\frac{441-1}{2} \)

= \(\frac{440}{2}=220 \\\)

And \(\frac{n^2+1}{2}=\frac{21^2+1}{2}=\frac{441+1}{2}\)

= \(\frac{442}{2}=221 \)

∴ 21² = 220+221

= 441

2. Here n= 13

∴ \(\frac{n^2-1}{2}=\frac{13^2-1}{2}=\frac{169-1}{2} \)

=\(\frac{168}{2}=84 \)

And \(\frac{n^2+1}{2}=\frac{13^2+1}{2}=\frac{169+1}{2} \)

= \(\frac{170}{2}=85 \)

13² = 84 + 85

= 169

3. Here n= 11

∴  \(\frac{n^2-1}{2}=\frac{11^2-1}{2}=\frac{121-1}{2}\)

= \(\frac{120}{2}=60 \)

And \(\frac{n^2+1}{2}=\frac{11^2+1}{2}=\frac{121+1}{2} \)

= \(\frac{122}{2}=61 \)

11² = 60+61

= 121 .

4. Here n= 19

⇒ \(\frac{n^2-1}{2}=\frac{19^2-1}{2}=\frac{361-1}{2} \)

=\(\frac{360}{2}=180 \)

And\(\frac{n^2+1}{2}=\frac{19^2+1}{2}=\frac{361+1}{2} \)

= \(\frac{362}{2}=181\)

19²  =180+181

=361 .

Question 8. Do you think the reverse is also true, i.e., is the sum of any two consecutive positive integers is perfect square ofa number? Give an example to support your answer
Solution:

No; The reverse is not always true.

For example :

1. Consider two consecutive positive integers 3 and 4. Their sum is 3 + 4 = 7 which is not a perfect square ofa number.

2. Consider two consecutive positive integers 8 and 9. Their sum is 8 + 9 = 17 which is not a perfect square ofa number

Question 9. Write the square, making use of the above pattern.

  1. 111111²
  2. 1111111²

Solution:

1. 11111²

11111²= 1234565432

2. 1111111²

1111111² = 1234567654321

Question 10. Can you find the square, of the following numbers using the above pattern?

  1. 6066667²
  2. 66666667²

Solution:

1. 6666667²

66666672 = 44444448888889

2. 66666667²

66666667² = 4444444488888889

NCERT Solutions For Class 8 Maths Chapter 5 Squares And Square Roots Exercise 5.1

Question 1. What will be the unit digit of the squares of the following numbers?

  1. 81
  2. 799
  3. 1234
  4. 52698
  5. 12796
  6. 272
  7. 3853
  8. 26387
  9. 99880
  10. 55555

Solution:

1. Unit’s digit of 81 = 1

⇒  1×1= 1

∴ The unit digit of the square of the number 81 will be 1.

2. Unit’s digit of 272 = 2

⇒ 2×2= 4

∴  The unit digit of the square of the number 272 will be 4.

3. Unit’s digit of 799 = 9

⇒  9×9= 81

∴  The unit digit of the square of the number 799 will be 1.

4.  Unit’s digit of 3853 = 3

⇒   3×3= 9

∴  The unit digit of the square of the number 3853 will be 9.

5. Unit’s digit of 1234 = 4

⇒ 4×4 = 16

∴  The unit digit of the square of the number 1234 will be 6.

6. Unit’s digit of 26387 = 7

⇒   7×7 = 49

∴  The unit digit of the square of the number 26387 will be 9.

7. Unit’s digit of 52698 = 8

⇒  8× 8 = 64

∴  The unit digit of the square of the number 52698 will be 4.

8. Unit’s digit of 99880 = 0

⇒  0 × 0 = 0

∴  The unit digit of the square of the number 99880 will be 0.

9. Unit’s digit of 12796 = 6

⇒  6 × 6 = 36

∴  The unit digit of the square of the number 12796 will be 6.

10. Unit’s digit of 55555 = 5

⇒ 5× 5 = 25

∴  The unit digit of the square of the number 55555 will be 5.

Question 2. The following numbers are not perfect squares. Give reasons.

  1. 1057
  2. 23453
  3. 798
  4. 22222
  5. 64000
  6. 89722
  7. 222000
  8. 505050

Solution:

1. 1057 

The number 1057 is not a perfect square because it ends with 7 at the unit’s place whereas the square numbers end with 0. I, 4. 5, 6 or 9 only at unit’s place.

2.  23453

The number 23453 is not a perfect square because it ends with 3 at the unit’s place whereas the square numbers end with 0, 1. 4, 5, 6, or 9 only at the unit’s place.

3.7928 

The number 7928 is not a perfect square because it ends with 8 at the unit’s place whereas the square numbers end with 0, 1, 4, 5, 6, or only 9 at the unit’s place.

4. 222222 

The number 222222 is not a perfect square because it ends with 2 at the unit’s place whereas the square numbers end with 0, 1, 4, 5, 6 or 9 only at the unit’s place.

5. 64000

The number 64000 is not a square number because it has 3 (an odd number of) zeros at the end whereas the number of zeros at the end of a square number ending with zeros is always even

6. 89722

The number 89722 is not a square number because it ends in 2 at the unit’s place whereas the square numbers end with 0, 1, 4, 5, 6, or 9 only at the unit’s place.

7. 222000

The number 222000 is not a square number because it has 3 (an odd number) zeros at the end whereas the number of zeros at the end of a square number ending with zeros is always even.

8. 505050

The number 505050 is not a square number because it has 1 (an odd number) zeros i at the end whereas the number of zeros at the end of a square number ending with zeros is always even.

Question 3. The squares of which of the following would be odd numbers?

  1. 431
  2. 2826
  3. 7779
  4. 82004

Solution:

1. 431

431 is an odd number

∴ Its square will also be an odd number

2.  2826

2826  is an oven number

∴  Its square will not be an odd number.

3. 779

779 is an odd number

∴  Its square will be an odd number.

4. 82004

82004  is an even number

∴  Its square will not be an odd number.

[Note that the squares of odd numbers are odd numbers and the squares of even numbers are even numbers

Question 4. Observe the following pattern and find the missing digits :

  1. 112²= 121
  2. 101² = 10201
  3. 1001² = 1002001
  4. 100001² = 1…………2 ……………..1
  5. 10000001² =………………………………

Solution:

4. 100001² = 10000200001

5. 10000001² = 100000020000001

Question 5. Observe the following pattern and supply the missing numbers

  1. 11² =  121
  2. 101² = 10201
  3. 10101² = 102030201
  4. 1010101² = ………………………
  5. …………………………..² = 10203040504030201

Solution:

4. 1010101² = 1020304030201

5. 101010101² = 10203040504030201

Question 6. Using the given pattern, find the jJJ missing numbers

  1. 1² + 2²+ 2² = 3²
  2. 2² + 3² + 6² = 7²
  3. 3² + 4² + 12² = 13²
  4. 4² + 5² + ………………²= 21²
  5. 5²+ ………………..² +  30²= 31²
  6. 6² + 7² + …………….² = …………..²

Solution:

4. 4² + 5² + 20² = 21²

5. 5² + 6²+ 30² = 31²

6. 6² + 7² + 4²= 43²

Question 7. Without adding, find the sum :

  1. 1+3 + 5 + 7 + 9
  2. 1+3 + 5 + 7 + 9 + 11 + 13 + 15+ 17 + 1
  3. 1 + 3 + 5+ 7 + 9 + 11 +13 + 15 + 17 + 19 + 21 + 23

Solution:

1. 1 + 3 + 5 + 7 + 9

= Sum of first five odd natural numbers

= 5² = 25

2. 1 + 3 + 5+ 7 + 9 + 11 + 13 + 15+17+19

= Sum of first ten odd natural numbers

= 10² = 100

1 + 3 + 5 + 7 + 9+11+13 + 15+17+19 + 21 + 23

= Sum of first twelve odd natural

= 12² = 144.

[Note that the sum of the first n odd natural numbers is n².]

Question 8.

  1. Express 49 as the sum of 7 odd numbers.
  2.  Express 121 as the sum of 11 odd numbers

Solution:

1. 49 =  7 ×  7 = 7²

= 1 + 3 + 5 + 7 + 9+11 + 13

2. 121 = 11 × 11 = 11²

= 1 + 3 + 5 + 7 + 9+11 + 13 + 15 + 17 + 19 + 21

Question 9. How many numbers lie between squares of the following numbers?

  1. 12 and13
  2. 25 and 26
  3. 99 and 100.

Solution:

1. Here, n = 12 and n + 1 = 13

∴ 2n, = 2 × 12 = 24

So, 24 numbers lie between squares of the numbers 12 and 13.

2. Here, n = 25 and n + 1 = 26

∴ 2n = 2 × 25 = 50

So, 50 numbers lie between squares of the numbers 25 and 26.

3. Here, n = 99 and 11+ 1 = 100

∴ 2n = 2 x 99 = 198

So, 198 numbers he between squares of the numbers 99 and 100
.
[Note that 2n natural numbers lie between the squares of the numbers n and (n + 1

Finding The Square Of A Number

23s = (20 + 3)² = (20 + 3) (20 + 3)

= 20 (20 + 3) + 3 (20 + 3)

= 202 + 20 × 3 + 3 × 20 + 32

= 400 + 60 + 60 + 9 = 529

Other Patterns In Squares

Let o5 be a number with the unit digit 5.

Then, (a.5)² = a (a + 1) hundred + 25

Question 1. Now can you find the square of 95?
Solution:

(95)² = 9 (9 + 1) ×100 + 25

= 9 × 10 × 100 + 25

= 9000+ 25

= 9025

Question 2. Find the the. squares of Ike following numbers containing 5 in unit’s place:

  1. 15
  2. 95
  3. 105
  4. 205

Solution:

1. 15² = (1×2) hundred + 25

= 200 + 25 = 225

2. 952 = (9 ×10) hundred + 25

= 9000 + 25 = 9025

3.  1052 = (10 × 11) hundred + 25

= 11000 + 25 = 11025

4. 2052 = (20 × 21) hundred + 25

= 42000 + 25 = 42025

Pythagorean Triplets Points

1. If a, b, and c are three numbers such that any one of the following three relations holds :

  1. a² + b² = c²
  2. b² + c² = a²
  3. c² + a² = b²

Then the numbers a, b, and c are said to form a Pythagorean triplet.

For example : 3, 4, 5 is a Pythagorean triplet because

3² + 42 = 9 + 16 = 25 = 5² .

Can you find more such triplets?

Yes; we can find more such triplets.

For example: 8, 15, 17; 9, 12, 15; 12, 35, 37, etc.

2. For any natural number m > 1, (2m, m² – 1, m² + 1) forms a Pythagorean triplet.

Try to find some more Pythagorean triplets using this form.

Some more Pythagorean triplets using this form are as follows :

6, 8, 10; taking m = 3

8, 15, 17; taking m. = 4

10, 24, 26; taking m = 5

NCERT Solutions For Class 8 Maths Chapter 5 Squares And Square Roots Exercise 5.2

Question 1. Find the square of the following numbers

  1. 32
  2. 35
  3. 86
  4. 93
  5. 71
  6. 46

Solution:

1. 32 = 30 + 2

⇒ 32 = 30 + 2

Therefore, 32² = (30 + 2)²

= (30 + 2) (30 + 2)

= 30(30 + 2) +2(30 + 2)

= 900 + 60 + 60 + 4

= 1024

2. 35 =30+5

⇒ 35 =30+5

Therefore, 35² = (30 + 5)²

= (30 + 5) (30 + 5)

= 30 (30 + 5) + 5 (30 + 5)

= 900+ 150+ 150 + 25

= 1225

3. 86 = 80 + 6

⇒  86 = 80 + 6

Therefore, 86² = (80 + 6)²

= (80 + 6) (80 + 6)

= 80 (80 + 6) + 6 (80 + 6)

= 6400 + 480 + 480+36

= 7396

4. 93 = 90 + 3

⇒  93 = 90 + 3

Therefore, 93² = (90 + 3)²

= (90 + 3) (90 + 3)

= 90 (90 + 3) + 3 (90 + 3)

= 8100 + 270+270+9

= 8649

5. 71 = 70+1

⇒   71 = 70+1

Therefore, 71² = (70 + 1)²

= (70+ 1) (70+ 1)

= 70(70+ 1)+ 1(70+ 1)

= 4900+70 + 70+1

= 5041

6. 46 =40 + 6

⇒ 46 =40 + 6

Therefore, 46² = (40 + 6)²

= (40 + 6) (40 + 6)

= 40 (40 + 6) + 6 (40 + 6)

= 1600 + 240 + 240 + 36

= 2116

Question 2. Write a Pythagorean triplet whose one number is

  1. 14
  2. 16
  3. 18.

Solution:

1. Let 2m. = 6

⇒  m = – = 3

m² – 1 = 3² – 1 = 9 – 1 = 8

And m² + 1 = 3² + 1 = 9 + 1 = 10

So, a Pythagorean triplet, whose one member is 6, is 6, 8, 10.

2. Let 2m = 14

⇒  m = \(\frac{14}{2}\) = 7

∴ m²- 1=7² -1 = 49 – 1= 48

and m² + 1 = 7² + 1 = 49 + 1 = 50

So, a Pythagorean triplet, whose one member is 14, is 14, 48, 50.

3. Let 2m = 16

⇒  m =  \(\frac{16}{2}\)

∴  m²- 1= 8² – 1 = 64 – 1 = 63

m² + 1 = 8² + 1 = 64 + 1 = 65

So, a Pythagorean triplet, whose one number is 16, is 16, 63, 65.

4.  Let 2m =18

⇒  m =  \(\frac{18}{2}\)

∴  m²- 1= 9² – 1 = 81- 1 = 80

m² + 1 = 9² + 1 = 81 + 1 = 82

So, a Pythagorean triplet, whose one the number is 18, is 18, 80, 82.

Square Roots

The square root of a number ‘a is that number which when multiplied by itself gives that number ‘a as a product.

Thus, if b is the square root of a;

Then b × b = a or b² = a

Symbolically, we write

b = \(\sqrt{a}\)

Note: b = \(\sqrt{a}\) ⇔  b² = a i.e., is the square root iofaaisthesquare ofb

Finding Square Roots

To /meet a number whose square is known is known as finding the squarewot.

Finding the square root is the inverse (opposite) operation of squaring.

There are two integral square roots of a perfect square number.

For example : 4 = (2)² = (- 2)²

∴ \(\sqrt{4}\) = 2 and – 2 both. Here, we shall take up only the positive square rootof a natural number.

Thus, \(\sqrt{4}\) = 2 (not- 2)

The positive square root of a number is denoted by the symbol V.

For example,

3² = 9

⇒ \(\sqrt{9}\)

= 3

Question 1. 11² = 121. What is the square root
Solution:

The square root of 121 is 11.

Question 2. 14² = 196. What is the square root of
Solution:

The square root of 196 is 14

Question 3.

  1. (- 1)² =1. Is-  1 a square root of 1?
  2. (- 2)² = 4.Is – 2 a square root of 4?
  3. (- 9)²= 81. Is – 9a square root of 81?

Solution:

  1. Yes; – 1 is a square root of 1.
  2. Yes; – 2 is a square root of 4.
  3. Yes; -9 is a square root of 81.

Finding Square Root Through Repeated Subtraction

We subtract successive odd numbers starting from 1 from the given square number till we get zero. The number of times, we have to make subtractions, gives the square root of the given square number.

Question 1. By repeated subtraction of odd (iii) 0) numbers starting from 1, find whether the following numbers are. perfect squares or not? If the number is a perfect square then find its square root.

  1. 121
  2. 55
  3. 36
  4. 49
  5. 90.

Solution:

1.  121

  1. 121 – 1 = 120
  2. 120 – 3 = 117
  3. 117 – 5 = 112
  4. 112- 7 = 105
  5. 105 – 9 = 96
  6. 96 – 11 = 85
  7. 85 – 13 = 72
  8. 72 – 15 = 57
  9. 57 – 17 = 40
  10. 40 – 19 = 21
  11. 21 – 21 = 0

Since from 121 we subtracted successive odd numbers starting from 1 and obtained 0 at the 11th step,

∴ \(\sqrt{121}\) = 11.

2. 55

  1. 55 – 1 = 54
  2. 54 – 3 = 51
  3. 51 – 5 = 46
  4. 46 – 7 = 39
  5. 39 – 9 = 30
  6. 30 – 11 = 19
  7. 19 – 13 = 6
  8. 6 – 15 = -9

This shows that we are not able to get zero at any step while subtracting successive odd numbers starting from 1 to 55.

∴  55 is not a perfect square

3.36

  1. 36 – 1 = 36
  2. 35 – 3 = 32
  3. 32 – 5 = 27
  4. 27 – 7 = 20
  5. 20 – 9 = 11
  6. 11 – 11 = 0

Since from 36 we subtracted successive odd numbers starting from 1 and obtained 0 at the 6th step, therefore, = 6.

4. 49

  1. 49 – 1 = 48
  2. 48 – 3 = 45
  3. 45 – 5 = 40
  4. 40 – 7 = 33
  5. 33 – 9 = 24
  6. 24 – 11 = 13
  7. 13 – 13 = 0

Since from 49 we subtracted successive odd numbers starting from 1 and obtained 0 at the 7th step,

∴ \(\sqrt{49}\) =7

  1. 90 – 1 = 89
  2. 89 – 3 = 86
  3. 86 – 5 = 81
  4. 81 – 7 = 74
  5. 74 – 9 = 65
  6. 65 – 11 = 54
  7. 54 – 13 = 41
  8. 41 – 15 = 26
  9. 26 – 17 = 9
  10. 9 – 19 = – 10

This shows that we are not able to get zero at any step while subtracting successive odd numbers starting from 1 to 90.

∴ 90 is not a perfect square.

Finding Square Root Through Prime Factorisation

We find the prime factors of the given perfect square and arrange in pairs. Then, we choose one factor from each pair and multiply it together. The product thus obtained gives the required square root.

Note: A square number has complete pairs of prime factors.

NCERT Solutions For Class 8 Maths Chapter 5 Squares And Square Roots Exercise 5.3

Question 1. What could be the possible ‘one’s’digits ofthe square root of each ofthe following numbers?

  1. 9801
  2. 99856
  3. 99856
  4. 657666025.

Solution:

1. 9801

∴1 × 1 = 1 and 9 × 9 = 81

The possible one’s digit of the square root ofthe number 9801 could be1 or 9.

2. 99856

∴ 4 × 4= 16 and 6 × 6 = 36

The possible one digit of the square root ofthe number 99856 could be 4 or 6.

3. 99856

∴ 1 × 1 = 1 and 9 × 9 = 81

The possible one digit of the square root of the number 998001 could be 1 or 9.

4. 657666025

∴  5 × 5 = 25

The possible one digit of the square root of the number 657666025 could be 5

Question 2. Without doing any calculation, find the numbers which are surely not perfect squares.

  1. 153
  2. 408
  3. 257
  4. 441.

Solution:Solution:

1. 153

The number 153 is surely not a perfect square because it ends in 3 whereas the square numbers end with 0, 1, 4, 5, 6, or 9 only.

2. 257

The number 257 is surely not a perfect square because it ends in 7 whereas the square numbers end with 0, 1, 4, 5, 6 or 9

3. 408

The number 408 is surely not a perfect square because it ends in 8 whereas the square numbers end with 0, 1, 4, 5, 6, or 9 only.

4. 441

The number 441 may be a perfect square as the square numbers end with 0, 1, 4, 5, 6, or 9 only.

Question 3. Find the square roots of 100 and 169 by the method of repeated subtraction.
Solution:

1.100

  1. 100 – 1 = 99
  2. 99 – 3 = 96
  3. 96 – 5 = 91
  4. 91 – 7 = 84
  5. 84 – 9 = 75
  6. 75 – 11 = 64
  7. 64 – 13 = 51
  8. 51 – 15 = 36
  9. 36 – 17 = 19
  10. 19 – 19 = 0

Since from 100, we subtracted successive odd numbers starting from 1 and obtained 0 at the 10th step, therefore, \(\sqrt{100}\)= 10.

2. 169

  1. 169 – 1 = 168
  2. 168-3=165
  3. 165-5=160
  4. 160 – 7 = 153
  5. 153-9 = 144
  6. 144-11=133
  7. 133 – 13 = 120
  8. 120 – 15 = 105
  9. 105 – 17 = 88
  10. 88 – 19= 69
  11. 69 – 21 = 48
  12. 48 – 23 = 25
  13. 25 – 25 = 0

Since from 169, we subtracted successive odd numbers starting from 1 and obtained 0 at I the 13th step, therefore,\(\sqrt{169}\)= 13.

Question 4. Find the square roots of the following numbers by the Prime Factorisation Method

  1. 729
  2. 400
  3. 1764
  4. 4096
  5. 7744
  6. 9604
  7. 5929
  8. 9216
  9. 529
  10. 8100

Solution:

1. The prime factorization of 729 is

729 = 3 × 3 × 3 × 3 × 3 × 3.

NCERT Solutions For Class 8 Maths Chapter 5 Squares And Square Roots The Prime Factorisation 729

By pairing the prime factors, we get

729 = \(3 \times 3 \times 3 \times 3 \times 3 \times 3\)

So, \(\sqrt{729}=3 \times 3 \times 3\) = 27

2. The prime factorization of 400 is

400 = 2× 2× 2× 2× 5× 5.

NCERT Solutions For Class 8 Maths Chapter 5 Squares And Square Roots The Prime Factors 400

By pairing the prime factors we get

400 =  \(2 \times 2 \times 2 \times 2 \times 5 \times 5\)

∴ \(\sqrt{400}=2 \times 2 \times 5\)

= 20

3.   The prime factorisation 2 1764of 1764 is 2

1764 = 2 × 2 × 3×3 × 7 × 7

NCERT Solutions For Class 8 Maths Chapter 5 Squares And Square Roots The Prime Factorisation is 1764

By pairing the prime factors, we get

1764 = \(2 \times 2 \times 3 \times 3 \times 7 \times 7\)

So,  \(\sqrt{1764}=2 \times 3 \times 7\)

= 42

4. The prime factorization 2 512 of 4096 is

4096 = 2 × 2 ×  2 ×  2 ×  2 ×  2× 2 × 2× 2×2× 2× 2

NCERT Solutions For Class 8 Maths Chapter 5 Squares And Square Roots The Prime Factorisation Is 4096

By pairing the prime factors, we get

4096 =  \(\underline{2 \times 2} \times \underline{2 \times 2} \times \underline{2 \times 2}\) × \(\underline{2 \times 2} \times \underline{2 \times 2} \times \underline{2 \times 2}\)

So, \(\sqrt{4096}=2 \times 2 \times 2 \times 2 \times 2 \times 2\)

= 64

5. The prime factorisation of 7744 is

7744 = 2 × 2× 2 × 2 × 2 × 2 ×11× 11.

NCERT Solutions For Class 8 Maths Chapter 5 Squares And Square Roots The Prime Factorisation Is 7744

By pairing the prime factors, we get

7744 = \(\underline{2 \times 2} \times \underline{2 \times 2} \times \underline{2 \times 2}\)

So,\(\sqrt{7744}=2 \times 2 \times 2 \times 11\)

6. The prime factorisation of 9604 is

9604 = 2 × 2 × 7 × 7 × 7 ×7.

NCERT Solutions For Class 8 Maths Chapter 5 Squares And Square Roots The Prime Factorisation Is 9604

By pairing the prime factors, we get

9604 = \(\underline{2 \times 2} \times \underline{7 \times 7} \times \underline{7 \times 7} \)

So, \(\sqrt{9604}=2 \times 7 \times 7=98\)

7. The prime factorization of 5929 is

5929 =  7 × 7 × 11 × 11

NCERT Solutions For Class 8 Maths Chapter 5 Squares And Square Roots The Prime Factorisation Is 5929

By pairing the prime factors, we get

5929 = \(\underline{7 \times 7} \times \underline{11 \times 11}\)

So, \(\sqrt{5929} =7 \times 11=77\)

8. The prime factorisation of 9216 is

9216 = 2 × 2 × 2 × 2 × 2× 2 × 2 × 2× 2 × 2 × 3 × 3

NCERT Solutions For Class 8 Maths Chapter 5 Squares And Square Roots The Prime Factorisation Is 9216

By pairing the primary factors get

9216 =  \( \underline{2 \times 2} \times \underline{2 \times 2} \)

So, \(\times \underline{2 \times 2} \times \underline{2 \times 2} \times \underline{2 \times 2} \times \underline{3 \times 3}\)

⇒ \(\sqrt{9216}=2 \times 2 \times 2 \times 2 \times 2 \times 3\)

= 96

9. The prime factorization of 529 is

529 = 23 × 23.

NCERT Solutions For Class 8 Maths Chapter 5 Squares And Square Roots The Prime Factorisation 529

By pairing the prime factors, we get

529 = \(\underline{23 \times 23} \)

So,\(\sqrt{529}\) = 23

10. The prime factorisation of 8100 is

8100 = 2 × 2 × 3 × 3 × 3 × 3 × 5 × 5.

NCERT Solutions For Class 8 Maths Chapter 5 Squares And Square Roots The Prime Factors 8100

By pairing the prime factors, we get

8100 =\(\underline{2 \times 2} \times \underline{3 \times 3} \times \underline{3 \times 3} \times \underline{5 \times 5}\)

So, \(\sqrt{8100}\) = 2×3 × 3 × 5

= 90.

Question 5. For each ofthe following numbers, find the smallest whole number by which it should be multiplied to get a perfect square number. Also, find the square root ofthe square number so obtained

  1. 252
  2. 180
  3. 10008
  4. 2028
  5. 1458
  6. 768

Solution:

1. The prime factorization of 252 is

252 = 2 × 2 × 3× 3× 7

NCERT Solutions For Class 8 Maths Chapter 5 Squares And Square Roots The Prime Factorisation 252

By pairing the prime  factors, we get

252= \(\underline{2 \times 2} \times \underline{3 \times 3} \times 7\)

As the prime factor 7 has no pair, 252 is not a perfect square.

If 7 gets a pair, then the number will be a perfect square. So, we multiply 252 by 7 to get

252 × 7 = \(\underline{2 \times 2} \times \underline{3 \times 3} \times \underline{7 \times 7}\)

Now each prime factor has a pair. Therefore, 252× 7 = 1764 is a perfect square. Thus the required smallest number is 7.

Also,\(\sqrt{1764}\) =2 × 3× 7

= 42

2. The prime factorisation of 180 is

180 = 2 × 2 × 3 × 3×5

NCERT Solutions For Class 8 Maths Chapter 5 Squares And Square Roots The Prime Factors 180

By pairing the prime factors, we get

180 = \(\underline{2 \times 2} \times \underline{3 \times 3} \times \underline{3 \times 3} \times \) x 5

As the prime factor 5 has no pair, 180 is not a perfect square.

If 5 gets a pair, then the number will be a perfect square. So, we multiply 180 by 5 to get

180 × 5= \(\underline{2 \times 2} \times \underline{3 \times 3} \times \underline{3 \times 3} \times \underline{5 \times 5}\)

Now each prime factor has a pair. Therefore, 180 × 5 = 900 is a perfect square. Thus the required smallest number is 5

Also, \(\sqrt{900}\)

= 2 ×  3× 5 = 30

3. The prime factorization of 1008 is

1008 = 2 × 2 × 2×2 × 3 × 3 × 7

NCERT Solutions For Class 8 Maths Chapter 5 Squares And Square Roots The Prime Factors 1008

By pairing the prime factors, we get

1008 = \(\underline{2 \times 2} \times \underline{2 \times 2} \times \underline{3 \times 3} \times 7\)

As the prime factor 7 has no pair, 1008 is not a perfect square.

If 7 gets a pair, then the number wil be a perfect square. So, we multiply 1008 by 7 to get

1008 × 7 = \(\underline{2 \times 2} \times \underline{2 \times 2} \times\underline{3 \times 3} \times \underline{3 \times 3} \times \underline{7 \times 7}\)

Now each prime factor has a pair, Therefore, 1008 x 7 = 7056 is a perfect square,  Thus the required smallest number is 7

Also, \(\sqrt{7056}\)

= 2 × 2 × 3× 7 = 84

4. The prime factorization of 2028

2028 = 2 × 2 × 3 × 13 × 13.

NCERT Solutions For Class 8 Maths Chapter 5 Squares And Square Roots The Prime Factors 2028

By pairing the prime factors, we get

2028 = \(\underline{2 \times 2}\) × 3 ×  \(\underline{13 \times 13}\)

As the prime factor 3 has no pair, 2028 is not a perfect square

If 3 gels a pair, then (.ho number will be a perfect square. So, we multiply 2028 by 3 to got

2028 × 3 = \(\underline{2 \times 2} \times \underline{3 \times 3} \times \underline{13 \times 13}\)

Now each prime factor has a pair. Therefore, 2028 x 3 = 6084 is a perfect square. Tims the required smallest number is 3.

Also,\(\sqrt{6084}\)

= 2 × 3 × 13 = 78.

5. The prime factorisation of 1458 is

1458 =2 × 3 × 3×  3  × 3 × 3 × 3

NCERT Solutions For Class 8 Maths Chapter 5 Squares And Square Roots The Prime Factorisation 1458

By pairing the prime 3 9 factors, we get

1458 = 2 ×\(\underline{3 \times 3} \times \underline{3 \times 3} \times \underline{3 \times 3} \times \underline{3 \times 3}\)

As the prime factor 2 has no pair,1458 is not a perfect square.

If 2 gets a pair, then the number will be a perfect square. So, we multiply 1458 by 2 to get

1458 × 2 = \(\underline{2 \times 2} \times\underline{3 \times 3} \times \underline{3 \times 3} \times \underline{3 \times 3} \times \underline{3 \times 3}\)

Now each prime factor has a pair. Therefore, 1458 x 2 = 2916 is a perfect square. Thus the required smallest number is 2

Also,\(\sqrt{2916}\)= 2 × 3 × 3 × 3

= 54

6. The prime 768 factorisation of  768 is

768 = 2 ×  2 ×  2 × 2 × 2 × 2 × 2 × 2 × 3.

NCERT Solutions For Class 8 Maths Chapter 5 Squares And Square Roots The Prime Factors 768

By pairing the prime factors, we get

786= \(\underline{2 \times 2} \times\underline{2 \times 2} \times \underline{2 \times 2} \times \underline{2 \times 2}\) x 3

As the prime factor 3 has no pair, 768 is not a perfect square.

If 3 gets a pair, then the number will be a perfect square. So, we multiply 768 by 3 to get

768 ×  3 = \(\underline{2 \times 2} \times\underline{2 \times 2} \times \underline{2 \times 2} \times \underline{2 \times 2} \times \underline{3 \times 3}\)

Now (the inch prime factor has a pair. Therefore, 768 x 3 = 2304 to a perfect square. Thus the required smallest number is 3.

Also,\(\sqrt{2304}\) = 2 × 2 × 2 × 2 × 3

= 48

Question 6. For each ofthe following numbers, find the smallest whole number by which it should be divided so as to get a perfect square. Also, find the square root ofthe square number so obtained

  1. 252
  2. 2925
  3. 396
  4. 2645
  5. 2800
  6. 1620

Solution:

1. The prime factorisation of 252 is

252 = 2 ×  2 × 3 × 3 × 7

NCERT Solutions For Class 8 Maths Chapter 5 Squares And Square Roots The Prime Factorisation Is 252.

By pairing the prime factors, we get

252 = \(\underline{2 \times 2} \times\underline{3 \times 3} \times\)7

We see that the prime factor 7 has no pair. So, if we divide 252 by 7, then we get

252 ÷ 7 = \(\underline{2 \times 2} \times\underline{3 \times 3}\)

Now each prime factor has a pair.

∴  252 ÷ 7 = 36 is a perfect square.

Thus, the required smallest number is 7

Also,\(\sqrt{36}\) = 2 ×  3

= 6

2. The prime factorisation of 2925 is

2925 = 3 × 3  ×5 × 5 × 13.

NCERT Solutions For Class 8 Maths Chapter 5 Squares And Square Roots The Prime Factorisation Is 2925

By pairing the prime factors, we get

2925 = \(\underline{3 \times 3} \times\underline{5 \times 5} \times\) 13

We see that the prime factor 13 has no pair.

So, if we divide 2925 by 13, then we get

2925 ÷ 13 =\(\underline{2 \times 2} \times\underline{5 \times 5}\)

Now each prime factor has a pair.

∴ 2925 ÷ 13 = 225 is a perfect square.

Thus, the required smallest number

Also,\(\sqrt{224}\)= 3 × 5

= 15

3. The prime factor ion of 306 is

306 = 2 × 2 × 3 × 3 × 11

NCERT Solutions For Class 8 Maths Chapter 5 Squares And Square Roots The Prime Factorisation Is 396

By pairing the prime 2 factors, we get

396 = \(\underline{3 \times 3} \times\underline{5 \times 5}\) ×11

We see that the prime factor 11 has no pair. So, if we divide 396 by 11, then we get

= \(\underline{2 \times 2} \times\underline{3 \times 3}\)

Now each prime factor has a pair.

∴ 396 ÷ 11 = 36 is a perfect square.

Thus, the required smallest number is 11.

Also , \(\sqrt{2}{3}\)

= 6

4. The prime factorisation of 2645 is

2645 = 5 × 23 × 23.

NCERT Solutions For Class 8 Maths Chapter 5 Squares And Square Roots The Prime Factorisation Is 2645

By pairing the prime factors, we get

2645 =  \(\underline{23 \times 23}\)

Now the only prime factor 23 has a pair.

Therefore, 2645 ÷ 5 = 529 is a perfect square.

Thus, the required smallest number is 5

Also,\(\sqrt{529}\) =23

5. The prime factorisation of 2800 is

2800 = 2  × 2 × 2 × 2 × 5 × 5 × 7.

NCERT Solutions For Class 8 Maths Chapter 5 Squares And Square Roots The Prime Factorisation Is 2800

By pairing the prime factors, we get

2800 = \(\underline{2 \times 2} \times\underline{2 \times 2} \times \times\underline{5\times 5} \times\)7

We see that the prime factor 7 has no pair. So, if we divide 2800 by 7, then we get

2800 ÷ 7  \(\underline{2 \times 2} \times\underline{2 \times 2} \times \times\underline{5\times 5}\)

Now each factor has a pair. Therefore, 2800 + 7 = 400 is a perfect square. Thus, the required smallest number is 7.

Also.\(\sqrt{400}\)

= 2 × 2 ×5 = 20

6. The prime factorisation of 1620 is

1620 = 2 × 2 × 3×3 × 3 × 3 × 5

NCERT Solutions For Class 8 Maths Chapter 5 Squares And Square Roots The Prime Factorisation Is 1620

By pairing the prime factors, we get

1620 ÷ 5 =  \(\underline{2 \times 2} \times\underline{3 \times 3} \times \times\underline{3\times 3}\)

Now each factor has a pair. Therefore, 1620 ÷ 5 = 324 is a perfect square. Thus, the required smallest number is 5.

Hence, \(\sqrt{324}\)= 2 × 3 × 3

= 18.

Question 7. The students of Class 8 of a school donated  ₹  2401 in all, for Prime Minister’s National ReliefFund, Each student donated as many rupees as the number of students in the class. Find the number of students in the class
Solution:

Let the number of students in the

Then rupees donated by each student = ₹ x

Rupees denoted by x students

= ₹ x  × x

= ₹ x²

∴ The students of class 8th of a school donated ₹ 2401 for the Prime Minister’s National ReliefFund

⇒ \(x^2= 2401 \)

= \(\sqrt{2401}\)

The prime factorisation of 2401 is

x = \(\sqrt{2401} \)

= \(\sqrt{7 \times 7} \times 7 \times 7\)

NCERT Solutions For Class 8 Maths Chapter 5 Squares And Square Roots The Prime Factorisation Is 2401

x = 7 × 7 = 49

Hence, the number of students in the class is 49

Question 8. 2025 plants arc to be planted in a Now prime factorization of 2 180 pardons in such a nay that each row contains as many plants as the number of rows. Find the number of rows and the number of plants in each row
Solution:

Let the number of rows 3 2025 be x. 3 675

Then, the number of plants in each row = x.

Number of plants to be planted

= x × x

=x²

But 2025 plants are to be planted in the garden.

⇒ \(x^2=2025 \Rightarrow x=\sqrt{2025}\)

The prime factorization of 2025 is

2025= \(\underline{3 \times 3} \times\underline{3 \times 3} \times \times\underline{5\times 5}\)

x = \(\sqrt{2025}\)

x = \(\sqrt{3 \times 3} \times 3 \times 3 \times 5 \times 5\)

NCERT Solutions For Class 8 Maths Chapter 5 Squares And Square Roots Factorisation Is 2025 Plants Are Obtained In A Garden

r = 3 × 3 × 5

x = 45

Hence, the number of rows is 45 and the number of plants in each row is 45

Question 9. Find the smallest square number that is divisible by each of the numbers 4, 9, and l0
Solution:

The least number divisible by each one of 4, 9, and  10 is their L.C.M

The L.C.M. of 4, 9 and 10 is

NCERT Solutions For Class 8 Maths Chapter 5 Squares And Square Roots The Least Number Divisible By Each One Of 4 And 9 And 10

2 × 2 × 3 × 3 × 5 = 180

Now prime factorisation of 180 is

180 = \(\underline{2 \times 2} \times \underline{3 \times 3} \times \underline{3 \times 3} \times \underline{5 \times 5}\)

NCERT Solutions For Class 8 Maths Chapter 5 Squares And Square Roots The Prime Factors 180

The prime factor 5 is not in pairs. Therefore 180 is not a perfect square.

To get a perfect square, each factor of 180 must be paired. So, we need to make a pair of 5

Therefore, 180 should be multiplied by 5.

Hence, the required smallest square

number is 180 ×5 = 900.

Question 10. Find the smallest square number that is divisible by each of the numbers 8, 15, and 20.
Solution:

The least number divisible by each one of 8, 15, and 20 is their L.C.M.

The L.C.M. of 8, 15 and

NCERT Solutions For Class 8 Maths Chapter 5 Squares And Square Roots The Least Number Divisible By Each One Of 8 And 15 And 20

2 × 2 × 2 × 3 × 5 = 120

Now prime factorisation of 120 is

120 = \(\underline{2 \times 2}\) × 2 × 3 × 5

NCERT Solutions For Class 8 Maths Chapter 5 Squares And Square Roots The Prime Factorisation Is 120

The prime factors 2, 3, and  5 are not in pairs.

∴ 120  is not a perfect square.

To get a perfect square, each factor of 120 must be paired. So, we need to make pairs of 2, 3 and 5. Therefore 120 should be multiplied by 2 × 3 × 5; ie. 30.

Hence, the required smallest square number is 120 × 30 = 3600

Finding Square Root By Division Method

Steps

  • Obtain the number whose square root is to be found. Place a bar for every pair of digits starting from the digit at one’s place. If the number of digits in it is odd, then the left-most single digit too will have a bar. Each pair and the left-most single digit (if any) is called a period.
  • Think of the largest, number whose square is less than or equal to the number under the extreme left bar. Take this number as the divisor and the quotient with the number under the extreme left bar as the dividend. Divide and get the remainder.
  • Bring down the number under the next bar to the right of the remainder. This becomes the new dividend.
  • Double the divisor and enter it with a blank on its right. Guess the largest possible digit to fill the blank which will also become the new digit in the quotient, such that when the new divisor is multiplied by the new quotient, the product is less than or equal to the new dividend obtained in step 3.
  • Continue this process till the remainder is 0 and no digits are left in the given number.
  • The quotient thus obtained is the required square root of the given number.

Question 1.  Can we say that if the perfect square is of n-digits, then its square root will have n/2 digits if n is even or \(\frac{(n+1)}{2}\)
Solution:

Yes. The following examples will establish the truth ofthe above fact.

1. Consider the perfect square 729. Here, n = 3 (which is odd)

∴ Number of digits in the square root

=\(\frac{n+1}{2}=\frac{3+1}{2}=2\)

= 2

Also =\(\sqrt{729}\)

= 27 (which has 2 digits)

2. Consider the perfect square 2025.

Here, n = 4 (which is even)

∴ Number of digits in the square root

= \(\frac{n}{2}=\frac{4}{2}\)

= 2

Also\(\frac{n}{2}=\frac{4}{2}\) =45(which has 2 digits)

Question 2. Without calculating square roots, find. the number of digits in the square root of the following numbers.

  1. 25600
  2. 100000000
  3. 36864

Solution:

1. By placing bars, we get  \(\overline{2}\) \(\overline{56}\) \(\overline{00}\) Since there are 3 bars, the square root will be of 3 digits

Aliter:

Here, n = 5 (which is odd)

Number of digits in the square root

= \(\frac{n+1}{2}\)

= \(\frac{5+1}{2}\)

= \(\frac{6}{2}\)

= 3

2. By placing bars, we get \(\overline{3}\) \(\overline{68}\)\(\overline{64}\)

⇒ \(\overline{1} \overline{00} \overline{00} \overline{00} \overline{00}\)

Since there are 5 bars, the square root will be 5 digits.

Aliter:

Here, n = 9 (which is odd)

Number of digits in the square root

=\(\frac{n+1}{2}=\frac{9+1}{2}\)

= \(\frac{10}{2}=5\)

3. By placing bars, we get \(\)

Since there are 3 bars, the square root will be of 3 digits.

Aliter:

Here, n = 5 (which is odd)

Number of digits in the square root

=\(\frac{n+1}{2}=\frac{5+1}{2}\)

= \(\frac{6}{2}\)

= 3

Square Roots Of Decimals

Put bars on the integral part of the number in the usual manner. Place bars in the decimal part on every pair of digits beginning with the first decimal place. Proceed as usual to find the square root.

NCERT Solutions For Class 8 Maths Chapter 5 Squares And Square Roots Exercise  5.4

Question 1. Find the square root of each of the following numbers by the Division method:

  1. 2304
  2. 4489
  3. 3481
  4. 529
  5. 3249
  6. 1369
  7. 5776
  8. 7921
  9. 576
  10. 1024
  11. 3136
  12. 900

Solution:

1. 2304

NCERT Solutions For Class 8 Maths Chapter 5 Squares And Square Roots 2304 Square Root Of Each The Numbers By Division Method

∴ \(\sqrt{2304}\)

= 48

2. 4489

NCERT Solutions For Class 8 Maths Chapter 5 Squares And Square Roots 4489 Square Root Of Each The Numbers By Division Method

∴ \(\sqrt{4489}\)

= 67

3. 3481

NCERT Solutions For Class 8 Maths Chapter 5 Squares And Square Roots 3481 Square Root Of Each The Numbers By Division Method

∴\(\sqrt{3481}\)

= 59

4. 529

NCERT Solutions For Class 8 Maths Chapter 5 Squares And Square Roots 529 Square Root Of Each The Numbers By Division Method

∴ \(\sqrt{529}\)

= 23

5.3249

NCERT Solutions For Class 8 Maths Chapter 5 Squares And Square Roots 3249 Square Root Of Each The Numbers By Division Method

∴ \(\sqrt{3249}\)

= 57

6. 1369

NCERT Solutions For Class 8 Maths Chapter 5 Squares And Square Roots 1369 Square Root Of Each The Numbers By Division Method

∴ \(\sqrt{1369}\)

= 37

7. 5776

NCERT Solutions For Class 8 Maths Chapter 5 Squares And Square Roots 5776 Square Root Of Each The Numbers By Division Method

∴ \(\sqrt{5776}\)

= 76

8. 7921

NCERT Solutions For Class 8 Maths Chapter 5 Squares And Square Roots 7921 Square Root Of Each The Numbers By Division Method

∴ \(\sqrt{7921}\)

= 89

9. 576

NCERT Solutions For Class 8 Maths Chapter 5 Squares And Square Roots 576 Square Root Of Each The Numbers By Division Method

∴ \(\sqrt{7921}\)

= 89

10. 1024

NCERT Solutions For Class 8 Maths Chapter 5 Squares And Square Roots 1024 Square Root Of Each The Numbers By Division Method

∴ \(\sqrt{1024}\)

= 32

11.  3136

NCERT Solutions For Class 8 Maths Chapter 5 Squares And Square Roots 3136 Square Root Of Each The Numbers By Division Method

∴ \(\sqrt{3136}\)

= 56

12. 900

NCERT Solutions For Class 8 Maths Chapter 5 Squares And Square Roots 900 Square Root Of Each The Numbers By Division Method

∴ \(\sqrt{900}\)

= 30

Question 2. Find the number of digits in the square root of each of the following numbers (without any calculation)

  1. 64
  2. 144
  3. 4489
  4. 27225
  5. 390625

Solution:

1. Number (n) of digits in 64

= 2 which is even.

∴ Number of digits in the square root of 64 = \(\frac{n}{2}=\frac{2}{2}\)

= 1

2. Number (n) of digits in 144

= 3 which is odd.

∴ Number of digits in the square root of

144 = \(\frac{n+1}{2}=\frac{3+1}{2}\)

= \(\frac{4}{2}\)

= 2

3. Number (n) of digits in 4489

= 4 which is even.

∴Number of digits in the square root of

4489 = \(\frac{n}{2}=\frac{4}{2}\)

= 2

4. Number (n) of digits in 27225

= 5 which is odd.

∴ Number of digits in the square root of

27225 = \(\frac{n}{2}=\frac{6}{2}\)

= 3

= \(\frac{6}{2}\)

5. Number (n) of digits in 390625

= 6 which is even.

∴ Number of digits in the square root of

=  \(\frac{n+1}{2}=\frac{5+1}{2}\) = \(\frac{6}{2}\)

= 3

Question 3. Find the square root of the following decimal number

  1. 2.56
  2. 7.29
  3. 51.84
  4. 42.25
  5. 31.36

Solution:

1. 2.56

NCERT Solutions For Class 8 Maths Chapter 5 Squares And Square Roots 2.56 Square Root Of The Decimal Number

Hence, \(\sqrt{2.56}=1.6\)

2. 7.29

NCERT Solutions For Class 8 Maths Chapter 5 Squares And Square Roots 7.29 Square Root Of The Decimal Number

Hence, \(\sqrt{7.29}=2.7\)

3. 51.84

NCERT Solutions For Class 8 Maths Chapter 5 Squares And Square Roots 51.84 Square Root Of The Decimal Number

Hence, \(\sqrt{51.84}=7.2\)

4.  42.25

NCERT Solutions For Class 8 Maths Chapter 5 Squares And Square Roots 42. 25 Square Root Of The Decimal Number

Hence, \(\sqrt{42.25}=6.5\)

5. 31.36

NCERT Solutions For Class 8 Maths Chapter 5 Squares And Square Roots 31.36 Square Root Of The Decimal Number

Hence, \(\sqrt{3136}=5.6\)

Question 4. Find the least number that must be subtracted from each of the following numbers to get a perfect square. Also, the square root of the perfect square  so obtained

  1. 402
  2. 1989
  3. 3250
  4. 825
  5. 4000

Solution:

1.  We have 402

NCERT Solutions For Class 8 Maths Chapter 5 Squares And Square Roots 402 Subtracted From Each The Number

This shows that 20² is less than 40² by 2. This means, that if we subtract the remainder of 2 from the number, we get a perfect square. So, the required least number is 2.

∴The required perfect square is 402-2 = 40

Hence,\(\sqrt{400}\)=20.

2. We have 1989

NCERT Solutions For Class 8 Maths Chapter 5 Squares And Square Roots 1989 Subtracted From Each The Number

This shows that 44² is less than 1989 by 53. This means that if we subtract the remainder of 53 from the number, we get a perfect square. So, the required least number is 53

∴The required perfect square is 1989 – 53 = 1936.

Hence, \(\sqrt{1936}\) = 44

3. We have,3250

NCERT Solutions For Class 8 Maths Chapter 5 Squares And Square Roots 3250 Subtracted From Each The Number

This shows that 57² is less than 3250 by

This means if we subtract the remainder (1) from the number, we get a perfect square. So, the required least number is 1.

∴ The required perfect square is

3250-1 = 3249

Hence,  \(\sqrt{43249}\)= 57

4. We have, 825

NCERT Solutions For Class 8 Maths Chapter 5 Squares And Square Roots 825 Subtracted From Each The Number

This shows that 28²  is less than 825 by 41. This means if we subtract the remainder (41) from the number, we get a perfect square. So, the required least number is 41.

∴ The required perfect square is 825-41 = 784

Hence,\(\sqrt{784}\) = 28

5.  We have, 4000

NCERT Solutions For Class 8 Maths Chapter 5 Squares And Square Roots 4000 Subtracted From Each The Number

This shows that 63² is less than 4000 by 31

This means if we subtract the remainder (31) from the number, we get a perfect square. So, the required least number is 31.

∴ The required perfect square is 4000 – 31 = 3969

Hence, \(\sqrt{3969}\) = 63

Question 5. Find the least number that must be added to each of the following numbers to get a perfect square. Also, find the square root of the perfect square to obtain

  1. 525
  2. 1750
  3. 252
  4. 1825
  5. 6412

Solution:

1. We have 525

NCERT Solutions For Class 8 Maths Chapter 5 Squares And Square Roots 525 The Least Number Which Must Added

This shows that 22² < 525.

The next perfect square is 23² = 529.

Hence, the least number to be added is

23² – 525 = 529 – 525 = 4

∴ The perfect square so obtained is 525 + 4 = 529

Hence,\(\sqrt{529}\) = 23

2. We have 1750

NCERT Solutions For Class 8 Maths Chapter 5 Squares And Square Roots 1750 The Least Number Which Must Added

This shows that 41² < 1750.

The next perfect square is 42² = 1764.

Hence, the least number to be added is

42² – 1750 = 1764 – 1750

= 14

∴ The perfect square so obtained is 1750 + 14 = 1764

Hence \(\sqrt{1764}\)

= 42

3. We have, 252

NCERT Solutions For Class 8 Maths Chapter 5 Squares And Square Roots 252 The Least Number Which Must Added

This shows that 15² < 252.

The next perfect square is 16² = 256.

Hence, the least number to be added is

16² – 252 = 256 – 252 = 4.

∴ The perfect square so obtained is 252 + 4 = 256

Hence \(\sqrt{256}\)

= 16

4. We have, 6412

NCERT Solutions For Class 8 Maths Chapter 5 Squares And Square Roots 1825 The Least Number Which Must Added

This shows that 42² < 1825

The next perfect square is 43² = 1849.

Hence, the least number to be added is

43² – 1825 = 1849 – 1825

= 24.

Therefore, the perfect square so obtained is 1825 + 24 = 1849

Hence, \(\sqrt{1849}\)

= 43

5. We have 1825

NCERT Solutions For Class 8 Maths Chapter 5 Squares And Square Roots 6412 The Least Number Which Must Added

This shows that 80² < 6412.

The next perfect square is 81² = 6561.

Hence, the least number to be added is

81² – 6412 = 6561 – 6412 = 149.

Therefore, the perfect square so obtained is 6412+ 149 = 6561

Hence,\(\sqrt{6561}\)

= 81

Question 6. Find the length of the side of a square whose area is 441 m.2.
Solution:

Area of the square = 441 m

Length ofthe side of the square

⇒  \(\sqrt{441}\)m

NCERT Solutions For Class 8 Maths Chapter 5 Squares And Square Roots 441 Length Of The Side Of A Square

∴  \(\sqrt{441}\) = 21

Hence, the length ofthe side ofthe square is 21 m

Question 7. In a right triangle ABC, ∠B – 30°.

  1. If AB = 6 cm, BC = 8 cm, find AC
  2.  If AC =13 cm, BC = 5 cm, find AB.

Solution:

1. In the right triangle ABC,

∠B = 90°

NCERT Solutions For Class 8 Maths Chapter 5 Squares And Square Roots Right AngleTriangle.

By Pythagoras Theorem

AC² = AB² + BC²

AC² = 6² + 8²

AC² = 36 + 64

AC² = 100

AC = \(\sqrt{100}\)

⇒ \(\sqrt{100}\) = 10

Hence, AC is equal to 10 cm.

In the right triangle ABC,

∠B = 90°

NCERT Solutions For Class 8 Maths Chapter 5 Squares And Square Roots Right AngleTriangle

By Pythagoras Theorem,

AC² = AB² + BC²

13² = AB² + 5²

169 = AB² + 25

AB² = 169-25

AB² = 144

AB = \(\sqrt{144}\)

NCERT Solutions For Class 8 Maths Chapter 5 Squares And Square Roots Pythagoras Theorem Of 441

Therefore,= \(\sqrt{144}\) 12.

Hence, AB is equal to 12 cm

Question 8. A gardener has 1000 plants. He wants to plant these in such a way that the number of rows and the number of columns remain the same. Find the minimum number of plants he needs for this
Solution:

Let the number of rows be x. Then the number of columns is x. Let us find out the square root of1000 by the division method

NCERT Solutions For Class 8 Maths Chapter 5 Squares And Square Roots Square Root Of 1000

This shows that 31² < 1000

32² = 1024.

Hence, the minimum number of plants needs more for this  = 1024 – 1000 = 24

So, number of rows

= number of columns = 32

Question 9. There are 500 children in a school. For a. P. T. drill they have to stand so that the number of rows is equal to several columns. How many children would be left out in this arrangement
Solution:

Let us find out the square root of the 500 division method.

NCERT Solutions For Class 8 Maths Chapter 5 Squares And Square Roots Square Root Of 500 Division Method

We get the remainder of 16. It shows that 222 is less than 500 by 16.

This means that 16 children would be left out of this arrangement

NCERT Solutions For Class 8 Maths Chapter 5 Squares And Square Roots Multiple-Choice Questions

Question 1. The perfect, square number out of 2, 3, 4, and 5 is

  1. 2
  2. 3
  3. 4
  4. 5

Solution: 3. 4

4 = 2 × 2 = 2²

Question 2. A perfect square number between 30 and 40 is

  1. 36
  2. 32
  3. 33
  4. 39

Solution: 1. 36

36 = 6 × 6 = 6²

Question 3. Between 50 and 60, the perfect square number is

  1. 56
  2. 55
  3. 54
  4. None

Solution: 4. None

None of 51,52,……59 is a perfect square

Question 4. Which ofthe following is a perfect square number?

  1. 1067
  2. 7828
  3. 4333
  4. 625

Solution: 4. 625

Perfect square numbers end with 0, 1, 4, 5, 6, or 9 only at the unit’s place.

Question 5. Which ofthe following is a perfect square number?

  1. 2222
  2. 32543
  3. 888
  4. 10000

Solution: 4. 10000

Perfect square numbers end with 0, 1, 4, 5, 6, or 9 only at the unit’s place.

6. Which of 132², 87², 72² and 209² would end with digit 1?

  1. 132²
  2. 87²
  3. 72²
  4. 209²

Solution: 4. 209²

If a number has 1 or 9 in the unit’s place, then its square ends in 1

Question 7. Which of 105², 216², 333² and 111² would end with the digit 1

  1. 105²
  2. 216²
  3. 333²
  4. 111²

Solution: 4. 111²

If a number has 1 or 9 in the unit’s place, then its square ends in 1

Question 8. Which of 17², 34²,  25², and 49² would have 6 at unit place?

  1. 17²
  2. 34²
  3. 25²
  4. 49²

Solution: 2. 34²

If a number has 4 or 6 in the unit’s place, then its square ends in 6.

Question 9. Which of 21², 33², 47², and 36² would have 6 at unit place?

  1. 21²
  2. 33²
  3. 47²
  4. 36²

Solution: 4. 36²

If a number has 4 or 6 in the unit’s place, then its square ends in 6.

Question 10. What will be the number of zeros in the square of the number 100

  1. 2
  2. 4
  3. 6
  4. 8

Solution: 2. 4

Number of zeros at the end of the number 100 = 2

Number of zeros at the end of the square ofthe number 100 = 2 ×2 = 4

Question 11. What will be the number of zeros in the square of the number 50?

  1. 1
  2. 2
  3. 3
  4. 4

Solution: 2. 2

Number of zeros at the end ofthe number 50 = 1

Number of zeros at the end of the square of the number 50 = 2×1 = 2

Question 12. What will be the number of zeros in the square ofthe number 9000?

  1. 2
  2. 3
  3. 4
  4. 6

Solution: 4. 6

Number of zeros at the end ofthe number 9000 = 3

Number of zeros at the end of the square ofthe number 9000 = 2 × 3 = 6

Question 13. The square of which of the following numbers will be even?

  1. 11
  2. 111
  3. 1111
  4. 112

Solution: 4. 112

∴ 112 is even 3.

∴ Its square will be even

Question 14. The square of which of the following numbers will be odd?

  1. 10
  2. 100
  3. 1000
  4. 99

Solution: 4. 99

∴ 99 is odd

∴  Its square will be odd.

Question 15. The square of which of the following numbers will be even?

  1. 21
  2. 27
  3. 35
  4. 50

Solution: 4. 50

∴ 50 is even.

∴  Its square will be even.

Question 16. The square of which of the following numbers will be odd?

  1. 42
  2. 54
  3. 66
  4. 81

Solution: 4. 81

∴ 81 is odd

∴  Its square will be odd

Question 17. How many natural numbers is he between 8² and 9²

  1. 16
  2. 17
  3. 18
  4. 19

Solution: 1. 16

2 × 8 = 16

Question 18. How many natural authors lie between 12² and 13²?

  1. 20
  2. 22
  3. 24
  4. 6

Solution: 3. 24

2× 12 = 24

Question 19. How many nonsquare numbers lie between the pair of numbers 80² and

  1. 162
  2. 160
  3. 161
  4. 154

Solution: 2. 160

2 × 80 = 160

Question 20. How many nonsquare numbers lie between the pair of numbers 36² and 37²?

  1. 36
  2. 37
  3. 74
  4. 72

Solution: 4. 72

2 × 36 = 72

Question 21. How many nonsquare numbers lie between the pair of numbers 500² and 501²

  1. 1000
  2. 999
  3. 1001
  4. 1002

Solution: 1. 1000

2 × 500=1000

Question 22. Express the square number 5² as the sum of two consecutive integers.

  1. 12+13
  2. 10+ 15
  3. 9+16
  4. 20 + 5

Solution: 1. 12+13

⇒  \(\frac{5^2-1}{2}=12, \quad \frac{5^2+1}{2}=13\)

Question 23. Express 9² as the sum of two consecutive integers.

  1. 40 + 41
  2. 50+31
  3. 36 + 45
  4. 72 + 9

Solution: 1. 40 + 41

⇒ \(\frac{9^2-1}{2}=40, \quad \frac{9^2+1}{2}=41\)

Question 24. Express 7² as the sum of two consecutive integers.

  1. 40 + 9
  2. 24+25
  3. 36+13
  4. 32+17

Solution: 2. 24+25

⇒ \(\frac{7^2-1}{2}=24, \quad \frac{7^2+1}{2}=25 .\)

Question 25. The unit digit in the square ofthe number 132 is

  1. 1
  2. 2
  3. 3
  4. 4

Solution: 4. 4

2 × 2 = 4

Question 26. The unit digit in the square ofthe number 1000 is

  1. 1
  2. 0
  3. 2
  4. None of these

Solution: 2. 0

0 × 0 = 0

Question 27. The unit digit in the* square of the number 1111 is

  1. 1
  2. 2
  3. 3
  4. 4.

Solution: 1. 1

1 × 1 = 1

Question 28. The unit digit in the square of the number  1333 is

  1. 3
  2. 6
  3. 9
  4. 1

Solution: 2. 6

3 × 3 = 9

Question 29. The unit digit in the square of the number 2644 is

  1. 4
  2. 6
  3. 8
  4. 2

Solution: 2. 6

4 × 4 = 1\(\underline{6}\)

Question 30. The unit digit in the square of the number 125 is

  1. 1
  2. 2
  3. 5
  4. 6

Solution: 3. 5

5 × 5 = 2 \(\underline{5}\)

Question 31. The unit digit in the square of the number 166 is

  1. 2
  2. 4
  3. 6
  4. 8.

Solution: 3. 6

6 × 6 = 3\(\underline{6}\)

Question 32. The unit digit in the square of the number 27 is

  1. 7
  2. 6
  3. 5
  4. 9.

Solution: 4. 9

7 × 7 =4 \(\underline{9}\)

Question 33. The unit digit in the square of number 78 is

  1. 1
  2. 2
  3. 3
  4. 4

Solution: 3. 3

8 × 8= 6\(\underline{4}\)

Question 34. The unit digit in the square of number 209 is

  1. 1
  2. 2
  3. 0
  4. 3

Solution: 1. 1

9× 9 = 8\(\underline{1}\)

Question 35. If 10² = 100, then the square root of 100 is

  1. 1
  2. 10
  3. 100
  4. 1000.

Solution: 2. 10

⇒ \(\sqrt{100}\) = 10

Question 36. If 25² = 625, (hen the square root of 625 is

  1. 5
  2. 26
  3. 125
  4. 625.

Solution: 2. 26

⇒  \(\sqrt{625}\) = 25

Question 37. What could be the possible one digit of the square root of 625?

  1. 2
  2. 3
  3. 4
  4. 5

Solution: 4. 5

5 × 5 = 25

Question 38. What could be the possible one digit of the square root of 121?

  1. 1, 9
  2. 3, 4
  3. 6, 7
  4. 7, 8.

Solution: 1. 1, 9

1 × 1 = 1

9 × 9 = \(1 \underline{81}\)

Question 39. What could be the possible one digit of the square root of 361?

  1. 1, 9
  2. 3, 4
  3. 6, 7
  4. 7, 8.

Solution: 1. 1, 9

1 × 1 = 1

9 × 9 = \(8 \underline{1}\)

Question 40. What could be the possible one digit of the square root of 576?

  1. 1, 9
  2. 5, 7
  3. 1, 8
  4. 2, 9.

Solution: 1. 1, 9

4 × 4 = \(1 \underline{6}\)

6 × 6 = \(1 \underline{36}\)

Question 41. What could be the possible one digit of the square root of 676?

  1. 4, 6
  2. 5, 7
  3. 1, 8
  4. 2, 9.

Solution: 1. 4, 6

4 × 4 = \(1 \underline{6}\)

6 × 6 = \(1 \underline{36}\)

Question 42. The smallest number by which 32 should be multiplied to get a perfect square is

  1. 2
  2. 3
  3. 4
  4. 8

Solution: 1. 2

32 × 2 = 64 = 8²

Question 43. The smallest number by which 48 should be multiplied to get a perfect square is

  1. 2
  2. 3
  3. 4
  4. 5

Solution: 2. 3

48 ×  3 = 144 = 12²

Question 44. The smallest number by which 45 should be multiplied to get a perfect square is

  1. 2
  2. 3
  3. 5
  4. 7

Solution: 3. 5

45 ×  5 = 225 = 15²

Question 45. The smallest number by which 54 should be multiplied so that we get a perfect square is

  1. 2
  2. 3
  3. 4
  4. 6

Solution: 1. 2

54 × 6 = 324 = 18²

Question 46. The smallest number by which 28 should be multiplied to get a perfect square is

  1. 2
  2. 4
  3. 3
  4. 7

Solution: 4. 7

28 × 7 = 196 = 14²

Question 47. The smallest number by which 1000 should be multiplied to get a perfect square is

  1. 5
  2. 10
  3. 4
  4. 8.

Solution: 2. 10

1000 × 10 = 10000 = 100²

Question 48. The smallest number by which 128 should be divided to get a perfect square is

  1. 2
  2. 3
  3. 4
  4. 8.

Solution: 1. 2

128 ÷ 2 = 64 = 8²

Question 49. The smallest number by which 48 should be divided to get a perfect square is

  1. 2
  2. 3
  3. 4
  4. 6.

Solution: 2. 3

48 ÷ 3 = 16 = 4²

Question 50. The smallest number by which 125 should be divided so as to get a perfect square is

  1. 3
  2. 5
  3. 25
  4. 125

Solution: 2. 5

125 ÷ 5 = 25 = 5²

Question 51. The smallest number by which 150 should be divided so as to get a perfect square is

  1. 4
  2. 2
  3. 5
  4. 6

Solution: 4. 6

150 ÷ 6 = 25 = 5²

Question 52. The smallest number by which 112 should be divided so as to get a perfect square is

  1. 6
  2. 4
  3. 3
  4. 7

Solution: 4. 7

112 ÷ 7 = 16 = 4²

Question 53. The smallest number by which 1000 should be divided so as to get a perfect square is

  1. 5
  2. 10
  3. 100
  4. 1000.

Solution: 2. 10

1000 ÷ 10 = 100 = 10²

Question 54. The smallest 3-digit, perfect square is

  1. 999
  2. 100
  3. 961
  4. 125

Solution: 2. 100

100 = 10²

Question 55. The number of digits in the square root of 62500 is

  1. 1
  2. 2
  3. 3
  4. 4

Solution: 3. 3

n = 5, \(\frac{n+1}{2}=3\)

Question 56. The number of digits in the square root of 441 is

  1. 1
  2. 2
  3. 3
  4. 4

Solution: 2. 2

n = 3, \(\frac{n+1}{2}=2\)

Question 57. The number of digits in the square root of 100 is

  1. 1
  2. 2
  3. 3
  4. 4

Solution: 2. 2

n = 3,  \(\frac{n+1}{2}=\) = 2

Question 58. Find the length of the side of a square whose area is 100 cm2.

  1. 5 cm
  2. 10 cm
  3. 100 cm
  4. 4 cm

Solution: 2. 10 cm

⇒ \(\sqrt{100}=10\)

Question 59. The students of class VIII of a school donated? 10000 in all, for Prime Minister’s National Relief Fund. Each student donated as many rupees as the number of students in the class. The number of students in the class is

  1. 10
  2. 100
  3. 1000
  4. 10000

Solution:  2. 100

⇒ \(\sqrt{10000}=100\)

NCERT Solutions For Class 8 Maths Chapter 5 Squares And Square Roots True or False

1. That sum of two perfect squares is a perfect square – False

2. The product of two perfect squares is a perfect square – True

3. 1000 is a perfect square – False

4. All numbers of a Pythagorean triplet are even – False

5. There is only one square number between 20 and 30 – True

NCERT Solutions For Class 8 Maths Chapter 5 Squares And Square Roots Fill In The Blanks

1. 11² = 60 +61

2. 112 + 113 =→ (15)²

3. There are non-square numbers between 42 and 52.  8

4. The square of 1.1 is  1.21

5. For every natural number m (> 1), 2m, m² – 1 and m² + 1 form a -Pythagorea

1. Write the greatest two-digit square number → 81

2. The smallest number of a Pythagorean triplet is 3. Find its other two numbers 4,5

3. Which letter best represents the location of 716 on a number line  E

NCERT Solutions For Class 8 Maths Chapter 5 Squares And Square Roots Number Line

4. The area of a square board is 144 square units. Find the length ofthe side ofthe square board  12 units.

5. Find the value of \(\sqrt{17+\sqrt{64}}\)  5

NCERT Solutions For Class 8 Maths Chapter 10 Exponents And Powers

Exponents And Powers

Exponents And Powers Introduction

If a is any non-zero integer and n is a positive integer, then

x a (n times) is written as a”,

i.e., a” is the continued product of a multiplied by itself n times.

Here, ‘o’ is called the base, and V is called the ‘exponent’ or ‘index’.

The number a” is read as a raised to the power of or simply as ‘nth power of o’.

The notation a” is called the exponential or power notation.

We can write large numbers more conveniently using exponents.

For example :

10000 = 104; 243 = 35; 128 = 27, etc.

Now, we shall learn about negative exponents

Read and Learn More NCERT Solutions For Class 8 Maths

Powers With Negative Exponents

If a is any non-zero integer and m is a positive integer, then

⇒ \(a^{-m}=\frac{1}{a^m}\)

Note: a m is called the multiplicative inverse of am as a-m x am = 1

Am and a ~m are multiplicative inverses of each other.

Note 2: \(a^m=\frac{1}{a^{-m}}\)

Question: What is 10 ~10 equal to?

Solution: \(10^{-10}=\frac{1}{10^{10}}\)

Question 1. Find the multiplicative inverse of the following:

  1. 2-4
  2. 10-5
  3. 7-2
  4. 5-3
  5. 10-100

Solution:

The multiplicative inverse is \(2^{-4}=\left(\frac{1}{2^{-4}}\right) \text { is } 2^4\)

⇒ \(a^{-m}=\frac{1}{a^m}\)

The multiplicative inverse of \(10^{-5}=\left(\frac{1}{10^{-5}}\right) \text { is } 10^5\)

⇒ \(a^{-m}=\frac{1}{a^m}\)

The multiplicative inverse of \(7^{-2}=\left(\frac{1}{7^{-2}}\right) \text { is } 7^2\)

⇒ \(a^{-m}=\frac{1}{a^m}\)

The multiplicative inverse \(5^{-3}=\left(\frac{1}{5^{-3}}\right) \text { is } 5^3\)

⇒ \(a^{-m}=\frac{1}{a^m}\)

The multiplicative inverse\(10^{-100}=\left(\frac{1}{10^{-100}}\right) \text { is } 10^{100}\)

⇒ \(a^{-m}=\frac{1}{a^m}\)

Question 2. Expand the following numbers using exponents :

  1. 1025.63
  2. 1256.249

Solution:

1025.63

1 x 1000 + 0 x 100 + 2 x 10 + 5 x 1 + 6 x\(\frac{1}{10}+3 \times \frac{1}{100}\)

= 1 x 103 + 0 x 102 + 2 x 101 + 5 x 10° + 6 x 10-1 + 3 x 10-2

1256.249

= 1 x 1000 + 2 x 100 + 5 x 10

+ 6x 1+ 2 x \( \frac{1}{10}+4 \times \frac{1}{100}+9 \times \frac{1}{1000}\)

= 1 X 103 + 2 X 102 + 5 X 101 + 6 x 10° + 2 x 10-1 + 4 x 10-2 + 9 x lO-3

Laws Of Exponents

If a, b are non-zero integers and m, n are any integers, then

  1. am x an = am+ n
  2. \(\frac{a^m}{a^n}=a^{m-n}\)
  3. (am)n = amn
  4. am x bm = (ab)m
  5. \(\frac{a^m}{b^m}=\left(\frac{a}{b}\right)^m\)
  6. a° = 1
  7. \(\left(\frac{a^{-m}}{b^{-n}}\right)=\frac{b^n}{a^m}\)
  8. \(\left(\frac{a}{b}\right)^{-m}=\left(\frac{b}{a}\right)^m\)

Remember

an” = 1 = n = 0

1n = 1 where n is any integer.

(- 1)n = 1 where n is any even integer.

(-1)n =-1 where n is any odd integer.

Q. Simplify and write in exponential form:

(-2)-3X (-2)-4

p3 x p -I0

32 x 3-5 x 36

Solution:

(-2)~3 x (-2)-4 = (- 2)(“3) + (-4>

am x an = am +n

= (-2)-7 = {(-1) x 2} -7

⇒ \(\frac{1}{\{(-1) \times 2\}^7}= \frac{1}{(-1)^7 \times(2)^7}\)

(ab)m = am bm

⇒ \(\frac{1}{(-1) \times 2^7}=-\frac{1}{2^7}\)

⇒ \(\mid(-1)^{\text {odd integer }}=-1\)

⇒ \(p^3 \times p^{-10}=p^{3+(-10)}=p^{-7}=\frac{1}{p^7}\)

⇒ \(a^{-m}=\frac{1}{a^m}\)

32 x 3-5 x 36 = 32 + (-5) + 6 = 33

Exponents And Powers Exercise 10.1

Question 1. Evaluate:

  1. 3-2
  2. (-4)-2
  3. \(\left(\frac{1}{2}\right)^{-5}\)

Solution:

⇒ \(3^{-2}=\frac{1}{3^2}=\frac{1}{9}\)

⇒ \(a^{-m}=\frac{1}{a^m}\)

⇒ \((-4)^{-2}=\frac{1}{(-4)^2}=\frac{1}{16}\)

⇒ \(a^{-m}=\frac{1}{a^m}\)

⇒ \(\left(\frac{1}{2}\right)^{-5}=\left(\frac{2}{1}\right)^5\)

⇒ \( a^{-m}=\frac{1}{a^m}\)

⇒ \(\frac{2^5}{1^5}\)

⇒ \(\left(\frac{a}{b}\right)^m=\frac{a^m}{b^m}\)

⇒ \(\frac{32}{1}=32\)

1n = 1 where n is an integer

Question 2. Simplify and express the result in power notation with a positive exponent

  1. \((-4)^5 \div(-4)^8\)
  2. \(\left(\frac{1}{2^3}\right)^2\)
  3. \((-3)^4 \times\left(\frac{5}{3}\right)^4\)
  4. (3-7 – 3-10) x 3-5
  5. 2-3 x (-7) -3

Solution:

1. (-4)6 ÷ (-4)8

⇒ \(\frac{(-4)^5}{(-4)^8}\)

⇒ \((-4)^{5-8}\)

⇒ \((-4)^{-3}\frac{a^m}{a^n}=a^{m-n}\)

⇒ \(=\frac{1}{(-4)^3}  a^{-m}=\frac{1}{a^m}\)

which is the required form.

2. \(\left(\frac{1}{2^3}\right)^2 =\frac{1^2}{\left(2^3\right)^2}\)

⇒ \(\left(\frac{a}{b}\right)^m=\frac{a^m}{b^m}\)

⇒ \(\frac{1}{2^{3 \times 2}} \)

⇒ \( \mid\left(a^m\right)^n=a^m\)

⇒ \(\frac{1}{2^6}\)

which is the required form

3. \((-3)^4 \times\left(\frac{5}{3}\right)^4=\{(-1) \times 3\}^4 \times \frac{5^4}{3^4} \)

⇒ \(\left(\frac{a}{b}\right)^m=\frac{a^m}{b^m}\)

⇒ \((-1)^4(3)^4 \times \frac{5^4}{3^4}\)

I (ab)m = am bm

= (- 1)4 x 54

= 1 x 54

(-1) even integer = 1

= 54

which is the required form.

4. (3-7 / 3-10) x 3-5

⇒ \(=\frac{3^{-7}}{3^{-10}} \times \frac{1}{3^5}\)

⇒ \( a^{-m}=\frac{1}{a^m}\)

⇒ \(3^{-7-(-10)} \times \frac{1}{3^5}\)

⇒ \(\frac{a^m}{a^n}=a^{m-n}\)

⇒ \(3^3 \times \frac{1}{3^5} \)

⇒ \(\frac{a^m}{a^n}=a^{m-n}\)

⇒ \(3^{3-5} \)

⇒ \(3^{-2} \)

⇒ \(a^{-m}=\frac{1}{a^m}\)

which is the required form.

5. \(2^{-3} \times(-7)^{-3}\)

⇒ \(\frac{1}{2^3} \times \frac{1}{(-7)^3}\)

⇒ \(\frac{1}{[2 \times(-7)]^3}=\frac{1}{a^m}\)

(ab)m = amb

⇒ \(\frac{1}{(-14)^3}\)

which is the required form.

Question 3. Find the value of:

(3° + 4 -1) x 22

(2-1 x 4-1)/2 -2

⇒ \(\left(\frac{1}{2}\right)^{-2}+\left(\frac{1}{3}\right)^{-2}+\left(\frac{1}{4}\right)^{-2}\)

⇒ \(\left(3^{-1}+4^{-1}+5^{-1}\right)^0\)

⇒ \(\left\{\left(\frac{-2}{3}\right)^{-2}\right\}^2.\)

Solution:

1. (3° + 4-1) x 22

⇒ \(\left(1+\frac{1}{4}\right) \times 4\)

⇒ \(a^{-m}=\frac{1}{a^m}, \quad a^0=1\)

⇒ \(\frac{5}{4} \times 4=5\)

(2-1 x 4-1) + 2″2

= {2 -l x (22)-1} + 2 -2

= {2-1 x 22x(-1>} -f2-2

(am)n = amn

= (2-’X2-2)T2-2 = 2(-1)+(-2) 2-2

am x a” = am+”

= 2-3 4- 2″2

⇒ \(\frac{2^{-3}}{2^{-2}}=2^{-3-(-2)} \)

⇒ \(\frac{a^m}{a^n}=a^{m-n}\)

⇒ \(2^{-1}\)

⇒ \(a^{-m}=\frac{1}{a^m}\)

⇒ \(\frac{1}{2}\)

3.\(\left(\frac{1}{2}\right)^{-2}+\left(\frac{1}{3}\right)^{-2}+\left(\frac{1}{4}\right)^{-2}\)

⇒ \(\frac{1^{-2}}{2^{-2}}+\frac{1^{-2}}{3^{-2}}+\frac{1^{-2}}{4^{-2}}\)

⇒ \(\left(\frac{a}{b}\right)^m=\frac{a^m}{b^m}\)

⇒ \(\frac{2^2}{1^2}+\frac{3^2}{1^2}+\frac{4^2}{1^2}\)

⇒ \(\frac{4}{1}+\frac{9}{1}+\frac{16}{1}\)

= 4 + 9 + 16 = 29

4. [3-1 + 4-1 + 5-1]0

⇒ \({\left[\frac{1}{3}+\frac{1}{4}+\frac{1}{5}\right]^0 }\)

⇒ \(\left(\frac{20+15+12}{60}\right)^0\)

⇒ \(\quad \text { | LCM }(3,4,5)=60\)

⇒ \(\left(\frac{47}{60}\right)^0=1\)

\(a^0=1\)

Aliter

(3-1 + 4-1 +5-1)° = 1

a°= 1

5. \(\left\{\left(\frac{-2}{3}\right)^{-2}\right\}^2\)

⇒ \(\left(\frac{-2}{3}\right)^{(-2) \times 2}\)

⇒ \(\mid\left(a^m\right)^n=a^{m n}\)

⇒ \(\left(\frac{-2}{3}\right)^{-4}\)

⇒ \(\left(\frac{3}{-2}\right)^4 \)

⇒ \( \left(\frac{a}{b}\right)^{-m}=\left(\frac{b}{a}\right)^m\)

⇒ \(\frac{3^4}{(-2)^4}=\frac{3^4}{(-1 \times 2)^4}\)

⇒ \(\left(\frac{a}{b}\right)^m=\frac{a^m}{b^m}\)

⇒ \(\frac{3^4}{(-1)^4(2)^4}\)

⇒ \((a b)^m=a^m b^m\)

⇒ \(\frac{3^4}{1 \times 2^4} \)

⇒ \((-1)^{\text {even integer }}=1\)

⇒ \(\frac{3^4}{2^4}\)

⇒ \(\frac{3 \times 3 \times 3 \times 3}{2 \times 2 \times 2 \times 2}=\frac{81}{16}\)

Question 4. Evaluate:

  1.  \(\frac{8^{-1} \times 5^3}{2^{-4}}\)
  2.  \(\left(5^{-1} \times 2^{-I}\right) \times 6^{-1}\)

Solution:

1. \(\frac{8^{-1} \times 5^3}{2^{-4}}\)

⇒ \(\frac{2^4 \times 5^3}{8^1}\)

⇒ \(a^{-m}=\frac{1}{a^m}\)

⇒ \(\frac{16 \times 125}{8}=250\)

2. \(\left(5^{-1} \times 2^{-1}\right) \times 6^{-1}=\left(\frac{1}{5} \times \frac{1}{2}\right) \times \frac{1}{6}\)

⇒ \(a^{-m}=\frac{1}{a^m}\)

⇒ \(\frac{1}{10} \times \frac{1}{6}=\frac{1}{60}\)

Question 5. Find the value of m for which 5m + 5-3 = 55
Solution:

5m + 5-3 = 55

⇒ \(\frac{5^m}{5^{-3}} =5^5\)

⇒ \(5^{m-(-3)} =5^5\)

⇒ \(\frac{a^m}{a^n}=a^{m-n}\)

⇒ \(5^{m+3} =5^5\)

bases are equal exponents are equal

m + 3 = 5

m = 5-3

m = 2

Question 6. Evaluate:

  1. \(\left\{\left(\frac{1}{3}\right)^{-1}-\left(\frac{1}{4}\right)^{-1}\right\}^{-1}\)
  2. \(\left(\frac{5}{8}\right)^{-7} \times\left(\frac{8}{5}\right)^{-4}.\)

Solution:

⇒ \(\left\{\left(\frac{1}{3}\right)^{-1}-\left(\frac{1}{4}\right)^{-1}\right\}^{-1}\)

⇒ \(\left(\frac{1^{-1}}{3^{-1}}-\frac{1^{-1}}{4^{-1}}\right)^{-1}\)

⇒ \(\left(\frac{a}{b}\right)^m=\frac{a^m}{b^m}\)

⇒ \(\left\{\frac{\frac{1}{1^1}}{\frac{1}{3^1}}-\frac{\frac{1}{1^1}}{\frac{1}{4^1}}\right\}^{-1}\)

⇒ \(a^{-m}=\frac{1}{a^m}\)

⇒ \(\left(\frac{3^1}{1^1}-\frac{4^1}{1^1}\right)^{-1}\)

⇒ \(a^1=a\)

⇒ \(\left(\frac{3}{1}-\frac{4}{1}\right)^{-1}\)

⇒ \( (3-4)^{-1}\)

⇒ \((-1)^{-1}=\frac{1}{(-1)^1}\)

⇒ \(| a^{-m}=\frac{1}{a^m}\)

⇒ \(\frac{1}{(-1)}\)

\((-1)^{\text {odd integer }}=-1\)= -1

⇒ \(\left(\frac{5}{8}\right)^{-7} \times\left(\frac{8}{5}\right)^{-4}=\frac{5^{-7}}{8^{-7}} \times \frac{8^{-4}}{5^{-4}}\)

⇒ \(\frac{5^{-7}}{5^{-4}} \times \frac{8^{-4}}{8^{-7}}\)

⇒ \(=5^{(-7)-(-4)} \times 8^m=\frac{a^m}{b^m}\)

⇒ \(\frac{a^m}{a^n}=a^{m-n}\)

⇒ \(5^{-7+4} \times 8^{-4}+7 \)

⇒ \(\frac{1}{5^3} \times 8^3=\frac{8^3}{5^3}\)

⇒ \(\frac{8 \times 8 \times 8}{5 \times 5 \times 5}=\frac{512}{125}\)

Question 7. Simplify:

  1. \(\frac{25 \times t^{-4}}{5^{-3} \times 10 \times t^{-8}} \quad(t \neq 0)\)
  2. \(\frac{3^{-5} \times 10^{-5} \times 125}{5^{-7} \times 6^{-5}}\)

Solution:

1. \(\frac{25 \times t^{-4}}{5^{-3} \times 10 \times t^{-8}}\)

⇒ \(\frac{25 \times 5^3}{10} \times \frac{t^8}{t^4}\)

⇒ \(\frac{625}{2} \times \frac{t^8}{t^4}\)

⇒ \(\frac{625 t^4}{2}\)

⇒ \(\frac{3^{-5} \times 10^{-5} \times 125}{5^{-7} \times 6^{-5}}\)

⇒ \(\frac{3^{-5} \times(2 \times 5)^{-5} \times(5 \times 5 \times 5)}{5^{-7} \times(2 \times 3)^{-5}}\)

⇒ \(\frac{3^{-5} \times 2^{-5} \times 5^{-5} \times 5^3}{5^{-7} \times 2^{-5} \times 3^{-5}} \)

⇒ \(\quad(a b)^m=a^m b^m\)

⇒ \(\frac{5^{-5} \times 5^3}{5^{-7}}=\frac{5^{(-5)+3}}{5^{-7}}\)

⇒ \(\mid a^m \times a^n=a^{m+n}\)

⇒ \(\frac{5^{-2}}{5^{-7}}=5^{(-2)-(-7)}\)

⇒ \(5^{-2+7}=5^5\)

Use Of Exponents To Express Small Numbers In Standard Form

A number is said to be in standard form if expressed in the form K x 10″ where 1 < K < 10 and n is an integer. A number written in standard form is said to be expressed in scientific notation.

Tiny numbers can be expressed in standard form using negative exponents.

1. To express a large number, we move the decimal point to the left such that only one digit is left to the left side of the decimal point and multiply the resulting number by 10n where n is the number of places to which the decimal point has been moved to the left.

For example : 270,000,000,000 = 2.7 x 1011

(Decimalpointhas have been moved to the left for11 places)

2. To express a number (< 1), we move the decimal point to the right such that only one digit is left to the left side of the decimal point and multiply the resulting number by 10~’1 where n is the number of places to which the decimal point has been shifted to the right.

For example : 0.000 0009 = 9 x 10-7

(Decimalpointhas have been shifted to the right for 7 places.)

Question 1. Identify huge and very small numbers from the above facts and write them in the following table:

NCERT Solutions For Class 8 Maths Chapter 10 Large And very Small

Solution:

NCERT Solutions For Class 8 Maths Chapter 10 Very Large Numbers

Question 2. Write the following numbers in standard form:

  1. 0.000000564
  2. 0.0000021
  3. 21600000
  4. 15240000

Solution:

1.  0.000000564

= 5.64 x 10-7

Moving decimal 7 places to the right

2.  0.0000021

0.0000021 = 2.1 x 10-6

Moving decimal 6 places to the right

3.  21600000

21600000 = 2.16 x 107

Moving decimal 7 places to the left

4.  15240000

15240000 = 1.524 x 107

Moving decimal 7 places to the left

Question 2. Write all the facts given in the standard form.
Solution:

(1) The distance from the Earth to the Sun is 1.496 x 1011 m

149, 600,000,000 = 1.496 x 1011.

Moving the decimal 11 places to the left

(2) The speed of light is 3 x 108 m/sec.

300, 000, 000 = 3 x108.

Moving decimal 8 places to the left

The thickness of the Class VII Mathematics book is 2 x 101 mm.

20 = 2 x 10 = 2 x 101

The average diameter of a Red Blood Cell is 7 x 10 6 mm.

0.00 0007 = 7 x 10-6

Moving decimal 6 places to the right

The thickness of human hair is in the range of 5 x 10 -3 cm to 1 x I0″2cm

⇒ \(0.005=\frac{5}{1000}=\frac{5}{10^3}=5 \times 10^{-3}\)

⇒ \(a^{-m}=\frac{1}{a^m}\)

⇒ \(0.01=\frac{1}{100}=\frac{1}{10^2}=1 \times 10^{-2}\)

The distance of the moon from the Earth is 3.84467 x 108m

384,467,000 = 3.84467 x 108

I Moved the decimal 8 places to the left

(7) The size of a plant cell is 1.275 x 10-5m

0.00001275 = 1.275 x 105

Moving decimal 5 places to the right

The average radius of the Sun is 6.95 x105km

695000 = 695 x 1000 = 695 x 103 = 6.95 x 105

(9) Mass of fuel in a space shuttle

solid rocket booster is 5.036 x 105 kg

503600

= 5036 x 100 = 5036 x 102

= 5.036 x 103 x 102

= 5.036 x 103+2

am x an = am+n

= 5.036 x 105

(10) Thickness of a piece of paper is 1.6 x10-3 cm

0.0016 = 1.6 x 10-3

Moving decimal 3 places to the right

The diameter of wire on a computer chip is 3 x 10-6 m

is 3 x 10-6

0.000003 = 3 x 013

Moving decimal 6 places to the right

(12) The height of Mount Everest is 8.848 x 103  m.

8848 = 8.848 x 1000 = 8.848 x 103

Exponents And Powers Exercise 10.2

Question 1. Express the following numbers in standard form:

  1. 0.0000000000085
  2. 0.00000000000942
  3. 6020000000000000
  4. 0.00000000837
  5. 31860000000.

Solution:

1. 0.0000000000085

0.0000000000085 = 8.5 x 10-12

Moving the decimal 12 places to the right

2. 0.00000000000942

0.00000000000942 = 9.42 x 10-15

Moving the decimal 12 places to the right

3. 6020000000000000

6020000000000000 = 6.02 x 1015

Moving the decimal 15 places to the left

4. 0.00000000837

0.00000000837 = 8.37 x 10-9

Moving decimal 9 places to the right

5. 31860000000

31860000000 = 3.186 x 1010

Moving decimal 10 places to the left

Question 2. Express the following numbers in the usual form:

  1. 3.02 x 10-6
  2. 4.5 x 104
  3. 3 x 10-8
  4. 1.0001 x 109
  5. 5.8 x 1012
  6. 3.61492 x 106

Solution:

⇒ \(3 \times 10^{-8}=\frac{3}{10^8}\)

⇒ \(a^{-m}=\frac{1}{a^m}\)

⇒ \(=\frac{3.02}{1000000}\)

⇒ 0.00000302

2. 4.5 x 104 = 4.5 x 10000 = 45000

3.  \(3.02 \times 10^{-6}=\frac{3.02}{10^6}\)

⇒ \(a^{-m}=\frac{1}{a^m}\)

⇒ \(\frac{3}{100000000}\)

=0.00000003

4. 1.0001 X 109

= 1.0001 X 1000,000,000

= 1,000,100,000

5. 5.8 X 1012

= 5.8 X 1,000,000,000,000

= 5,800,000,000,000

6. 3.61492 x 106

= 3.61492 x 1,000,000

= 3,614,920

Question 3. Express the number appearing in the following statements in standard form:

  1. 1 micron is equal to \(\frac{1}{1000000} m\)
  2. The charge of an electron is 0. 000, 000,000,000,000,000,1 6 coulomb.
  3. The size of bacteria is 0. 0000005 m
  4. The size of a plant cell is 0. 00001 275 m
  5. The thickness of thick paper is 0.07 mm.

Solution:

1.  \(\frac{1}{1000000} \mathrm{~m}= \frac{1}{10^6}\)

⇒ \(1 \times 10^{-6} \mathrm{~m}\)

⇒ \(a^{-m}=\frac{1}{a^m}\)

which is the required standard form.

2.  0.000,000,000,000,000,000,16 coulomb

⇒ \(\frac{16}{100,000,000,000,000,000,000} \text { coulomb }\)

⇒ \(frac{16}{10^{20}} \text { coulomb }\)

⇒ \(\frac{1.6 \times 10}{10^{20}} \text { coulomb }\)

⇒ \(\frac{1.6 \times 10^1}{10^{20}} \text { coulomb } \quad \mid a^1=a\)

⇒ \(1.6 \times 10^{1-20} \text { coulomb } \frac{a^m}{a^n}=a^{m-n}\)

⇒ \(1.6 \times 10^{-19} \text { coulomb }\)

which is the required standard form.

3. 0.0000005 m

⇒ \(\frac{5}{10000000} \mathrm{~m} \)

⇒ \(\frac{5}{10^7} \mathrm{~m}\)

⇒ \(5 \times 10^{-7} \mathrm{~m}\)

which is the required standard form.

4.  \(0.00001275 \mathrm{~m} =\frac{1275}{100,000,000} \mathrm{~m}\)

⇒ \(\frac{1275}{10^8} \mathrm{~m}\)

⇒ \(\frac{1275}{10^3 \times 10^5} \mathrm{~m}\)

| am x a” = am + n

⇒ \(\frac{1275}{10^3} \times 10^{-5} \mathrm{~m}\)

⇒ \(a^{-m}=\frac{1}{a^m}\)

⇒ \(1.275 \times 10^{-5} \mathrm{~m}\)

which is the required standard form.

5. \( 0.07 \mathrm{~mm} =\frac{7}{100}\)

⇒ \(\frac{7}{10^2}=7 \times 10^{-2} \mathrm{~mm}\)

⇒ \(a^{-m}=\frac{1}{a^m}\)

which is the required standard form

Question 4. In a stack, there are 5 books each of thickness 20 mm, and 5 paper sheets each of thickness 0. 016mm. What is the total thickness of the stack?
Solution:

Total thickness of books = 5 x 20 mm = 100 mm

Total thickness of paper sheets = 5 x 0.016 mm = 0.080 mm

Total thickness of the stack

= Total thickness of books + Total thickness of paper sheets

= 100 mm + 0.080 mm

= (100 + 0.080) mm

= 100.080 mm

= 1.0008 x 102 mm.

Moving decimal 2 places to the left

Hence, the total thickness of the stack is 1.0008 x IQ2 mm.

Exponents And Powers Multiple-Choice Question And Solutions

Question 1. am x an is equal to

  1. am+n
  2. am-n
  3. amn
  4. an-m

Solution: 1. am+n

Question 2. am ÷ am is equal to

  1. am+n
  2. am-n
  3. amn
  4. an-m

Solution: 1. am+n

Question 3. (am)is equal to

  1. am+n
  2. am-n
  3. amn
  4. an-m

Solution: 3. amn

Question 4. Am x is equal to

  1. (ab)m
  2. (ab)-m
  3. amb
  4. abm

Solution: 1. (ab)m

Question 5. a0 is equal to

  1. 0
  2. 1
  3. -1
  4. a

Solution: 2. 1

Question 6. \(\frac{a^m}{b^m}\) is equal to

  1. \(\left(\frac{a}{b}\right)^m\)
  2. \(\left(\frac{b}{a}\right)^m\)
  3. \(\frac{a^m}{b}\)
  4. \(\frac{a}{b^m}\)

Solution: 1. \(\left(\frac{a}{b}\right)^m\)

Question 7. 2 x 2 x 2 x 2 x 2 is equal to

  1. 24
  2. 23
  3. 22
  4. 25

Solution: 4. 25

Question 8. In 102, the exponents

  1. l
  2. 2
  3. 10
  4. 1

Solution: 2. 2

Question 9. In 102, the base is

  1. 1
  2. 0
  3. 10
  4. 100

Solution: 3. \(\frac{1}{10}\)

Question 10. 10-1 is equal to

  1. 10
  2. -1
  3. \(\frac{1}{10}\)
  4. \(-\frac{1}{10} \text {. }\)

Solution: 3. \(\frac{1}{10}\)

Question 11. The multiplicative inverse of 2-3 is

  1. 2
  2. 3
  3. -3
  4. 23

Solution: 4. 23

Question 12. The multiplicative inverse of 105 is

  1. 5
  2. 10
  3. 10-5
  4. 105

Solution: 3. 10-5

Question 13. The multiplicative inverse of \( \frac{1}{2^2}\)

  1. 2-2
  2. 22
  3. 2
  4. 1

Solution: 2. 22

Question 14. The multiplicative inverse of 10 “10 is

  1. 10
  2. \(\frac{1}{10}\)
  3. 10-10
  4. 1010

Solution: 4. 1010

Question 15. The multiplicative inverse of am is

  1. a
  2. m
  3. am
  4. a-m

Solution: 5. a-m

Question 16. 53 x 5-1 is equal to

  1. 5
  2. 53
  3. 5-1
  4. 52

Solution: 4. 52

Question 17. (-2)5 x (- 2)6 is equal to

  1. 2
  2. -2
  3. -5
  4. 6

Solution: 2. -2

Question 18. 32 x 3-4 x 35 is equal to

  1. 3
  2. 32
  3. 33
  4. 35

Solution: 3. 33

19. (- 2) 2 is equal to

  1. \(\frac{1}{4}\)
  2. \(\frac{1}{2}\)
  3. \(-\frac{1}{2}\)
  4. \(-\frac{1}{4}\)

Solution: 1. \(\frac{1}{4}\)

Question 20. \(\left(\frac{1}{2}\right)^{-4}\) is equal to

  1. 2
  2. 24
  3. 1
  4. 2-4

Solution: 2. 24

Question 21. (20 + 4-1) x 22 is equal to

  1. 2
  2. 3
  3. 4
  4. 5

Solution: 4. 5

Question 22. (2-1 + 3-1 + 5-1)0 is equal to

  1. 2
  2. 3
  3. 5
  4. 1

Solution: 4. 1

Question 23. 3m÷ 3-3 = 35 ⇒ m is equal to

  1. 1
  2. 2
  3. 3
  4. 4

Solution: 2. 2

Question 24. (-2)m+1 x (-2)4 = (- 2)6 ⇒ m =

  1. 0
  2. 1
  3. -1
  4. none of these

Solution: 2. 1

Question 25. (-1)60 is equal to

  1. -1
  2. 1
  3. 50
  4. -50

Solution: 2. 1

Question 26. (-1)51 is equal to

  1. -1
  2. 1
  3. 51
  4. -51

Solution: 1. -1

Question 27. 149600000000 is equal to

  1. 1.496 x 1011
  2. 1.496 x lO10
  3. 1.496 x 1012
  4. 1.496 x 105

Solution: 1. 1.496 x 1011

Question 28. 300000000 is equal to

  1. 3 x 108
  2. 3 x 107
  3. 3 x 106
  4. 3 x 109

Solution: 1. 3 x 108

Question 29. 0.000007 is equal to

  1. 7 x 10-6
  2. 7 x 10-6
  3. 7 x 10-4
  4. 7 x 10-3

Solution: 1. 7 x 10-6

Question 30. 384467000 is equal to

  1. 3.84467 x 1o8
  2. 3.84467 x 103
  3. 3.84467 x 107
  4. 3.84467 x 106

Solution: 1. 3.84467 x 108

Question 31. 0.00001275 is equal to

  1. 1.275 x 10-6
  2. 1.275 x 10-3
  3. 1.275 x 104
  4. 1.275 x 103

Solution: 1. 1.275 x 10-6

Question 32. 695000 is equal to

  1. 6.95 x 105
  2. 6.95 x 103
  3. 6.95 x 106
  4. 6.95 x 104

Solution: 1. 6.95 x 105

Question 33. 503600 is equal to

  1. 5.036 x 105
  2. 5.036 x 106
  3. 5.036 x 104
  4. 5.036 x 107

Solution: 1. 5.036 x 105

Question 34. 0.0016is equal to

  1. 1.6 x 10 -3
  2. 1.6 x 10-2
  3. 1.6 x 10 -4
  4. 1.6 x lO-5

Solution: 1. 1.6 x 10-3

Question 35. 0.000003 is equal to

  1. 3 x 10-6
  2. 3 x 106
  3. 3 x 105
  4. 3 x 10-5

Solution: 1. 3 x 10-6

Question 36. 8848 is equal to

  1. 8.848 x 103
  2. 8.848 x 102
  3. 8.848 x 10
  4. 8.848 x 104

Solution: 1. 8.848 x 103

Question 37. 1.5 x 1011 is equal to

  1. 150000000000
  2. 15000000000
  3. 1500000000
  4. 1500000000000

Solution: 1. 150000000000

Question 38. 2.1 x 10-6 is equal to

  1. 0.0000021
  2. 0.000021
  3. 0.00021
  4. 0.0021.

Solution: 1. 0.0000021

Question 39. 2.5 x 104 is equal to

  1. 25
  2. 250
  3. 2500
  4. 25000

Solution: 4. 25000

Question 40. 0.07 x 1O10is equal to

  1. 700000000
  2. 7000000
  3. 7000
  4. 7

Solution: 1. 700000000

Exponents And Powers True-False

Write whether the following statements are True or False:

1. The value of \(\left\{(-1)^{-1}\right\}^{-1}\) is 1: False

2. The reciprocal of \(\left(\frac{4}{3}\right)^0\) is 1: True

3. The standard form of \(\frac{1}{1000000}\) is 1.0 x 10 -6: True

4. If 6m + 6 “3 = 66, then the value of m is 3: True

5. 2345.6 = 2 x 1000 + 3 x 100 + 4 x 10 + 5 x 1 + 6 x 10 – 1: True

Exponents And Powers Fill in the Blanks

1. (1000)° = 1

2. The standard form of 1,234,500,000,000 is: 1.2345 x 1012

3. The multiplicative inverse of(-3) ~2 is: (-3)2

4. (- 9)4 -5- (- 9)10 is equal to (-9)-6

5. The value of (2 -1 + 3 -1 + 4 -1)° is : 1

6. Write 1.0002 x 109 in the usual form: 1000200000

7. Write the reciprocal of 10 _1: 10

8. Find the value of*if* “3 = (100)1-4 + (100)°: 100

9. By what number should (-8) -1 be divided, so that the quotient may be equal to (-8) -1: 1

10. If = \(\frac{5^m \times 5^2 \times 5^{-3}}{5^{-5}}=5^4\) then find the value of m: 0

NCERT Solutions For Class 8 Maths Chapter 4 Data Handling

NCERT Solutions For Class 8 Maths Chapter 4 Data Handling Introduction

The information collected in the form of numbers in the context of a situation under study is called data.

For example :

Marks obtained by the students of your class in the monthly test in English, weights of students of your class, etc.

We systematically organise the data and then interpret it.

Sometimes, we represent the data graphically to get a clear idea of what it represents.

Question 1. A pictograph

It is a pictorial representation of data using symbols

Read and Learn More NCERT Solutions For Class 8 Maths

NCERT Solutions For Class 8 Maths Chapter 4 Data Handling It Is A Pictorial Representation Of Data Using Symbols

1. How many cars were produced in July?
Solution:

250 cars were produced in July

2. In which month was the maximum number of cars produced?
Solution:

The maximum number (= 400) was produced in September’

Question 2. A bar graph

It is a display of information using bars of uniform width, with equal gaps in between. Their heights are proportional to the respective values.

NCERT Solutions For Class 8 Maths Chapter 4 Data Handling A Bar Graph

  1. What is the information given by the bar graph?
  2. In which year is the increase in the number of students maximum?
  3. In which year is the number of students maximum?
  4. Slate whether true or false :

‘The number of students during 2005-2006 is twice that of 2003-04’.

Solution:

1. The bar graph gives information about the number of students in class VIII in various academic years in the school.

2. Increase in the number of students in

2004 – 05 = 200 – 100 = 100

Increase in the number of students in

2005- 06 = 250 – 200 = 50

Increase in the number of students in

2006-07 = 300 – 250 = 50

Increase in the number of students in

2007-08 = 350 – 300 = 50

Hence, the increase in the number of X students is maximum in the year 2004-2005.

3. The number of students is maximum in the year 2007-08.

4. The number of students during

2005-06 = 250

The number of students during

2003-04 = 100

Twice the number of students during

2003-04 = 2 xl00 = 200* 250

( = The number of students during 2005- 06)

Since the number of students during 2005-06 is not twice that of 2003-04, therefore the given statement is false.

Question 3. Double bar graph

Bar graph allowing Iwo sola of data simultaneously. It is useful for the comparison of the data.

NCERT Solutions For Class 8 Maths Chapter 4 Data Handling Double Bar Graph

  1. What is the information given by the double bar graph?
  2. In which subject has the performance improved the most?
  3. In which subject has the performance deteriorated?
  4. In which subject is the performance at par?

Solution:

1. The double bar graph gives information about the marks obtained by a student in different subjects in the academic years 2005-06 and 2006-07.

2. The performance has improved the most in the subject of Mathematics.

3. The performance has deteriorated in the subject of English.

4. The performance is at par in the subject of Hindi

Question 4. If we change the position of any of the bars of a bar graph, would it change the information being conveyed? Why
Solution:

No, because bar heights give the quantity for each category. Since by changing the position of any of the bars of a bar graph, the height of the bar remains unchanged, the information conveyed regarding the quantity remains unchanged

Question 5. Draw an appropriate graph to represent the given information.

NCERT Solutions For Class 8 Maths Chapter 4 Data Handling Represent The Given Information

Solution:

NCERT Solutions For Class 8 Maths Chapter 4 Data Handling Number of Watches Sold

Question 6.

NCERT Solutions For Class 8 Maths Chapter 4 Data Handling Children Who Prefer In School

Solution:

NCERT Solutions For Class 8 Maths Chapter 4 Data Handling Number Of Childrens And Schools

Question 7. Percentage wins in ODI by 8 top cricket teams

NCERT Solutions For Class 8 Maths Chapter 4 Data Handling Percentage Wins In ODI By 8 Top Cricket Teams

Solution:

NCERT Solutions For Class 8 Maths Chapter 4 Data Handling Cricket Teams

Circle Graph Or Pie Chart

A circle graph shows the relationship between a whole and its parts. We divide the whole circle into various sectors. The size of each sector is proportional to the information or activity it represents.

Question 8. Each of the following pie charts gives you a different piece of information about your class. Find the fraction ofthe circle representing each of these information

NCERT Solutions For Class 8 Maths Chapter 4 Data Handling Different Pieces Of Uniform About Class

Solution:

1. Fraction ofthe circle representing the ‘girls’

= \(\frac{\text { Percentage of girls }}{\text { Percentage of all students }}\)

= \(\frac{50}{100}=\frac{1}{2}\)

Dividing the numerator and denominator by 50 [= HCF (50, 100)]

Fraction of the circle representing the boys

= \(\frac{\text { Percentage of boys }}{\text { Percentage of all students }}\)

= \(\frac{50}{100}=\frac{1}{2}\)

2. Fraction of the circle representing walk

= \(\frac{\text { Percentage of walk }}{\text { Total percentage of transport }} \)

= \(\frac{40}{100}=\frac{2}{5}\)

Dividing the numerator and denominator by 20 [= HCF (40, 100)]

Fraction ofthe circle representing‘cycle’

= \(\frac{\text { Percentage of cycle }}{\text { Total percentage of transport }} \)

= \(\frac{20}{100}=\frac{1}{5}\)

Fraction of the circle representing (Bus or car)

= \(\frac{\text { Percentage of Bus or } \mathrm{Car}}{\text { Total percentage of transport }}\)

= \(\frac{40}{100}=\frac{2}{5}\)

Dividing the numerator and denominator by 20 [= HCF (40, 100)]

3. The fraction of the circle represents those who hate Mathematic

= \(\frac{\text { Percentage of students who hate Mathematics }}{\text { Percentage of all students }}\)

= \(\frac{15}{100}=\frac{3}{20}\)

Dividing the numerator and denominator by 20

[= HCF (40, 100)]

Dividing the numerator and denominator by 5

[= HCF (15, 100)

The fraction ofthe circle representing those who whose Mathematic

= \(\frac{\begin{array}{c}
\text { Percentage of those who love Mathematics } \\
\end{array}}{\text { Percentage of all students }}\)

= \(\frac{100-15}{100}\)

= \(\frac{85}{100}=\frac{17}{20}\)

Question 9. Answer the following questions based on the pie chart given

  1. Which type of programmes are viewed the most?
  2. Which two types of programmes have several viewers equal to those watching sports channels

NCERT Solutions For Class 8 Maths Chapter 4 Data Handling Viewers Watching Different Types Of Channels

Solution:

From the given pie chart, we prepare the following table

NCERT Solutions For Class 8 Maths Chapter 4 Data Handling Types Of Viewers

1. Since the percentage of entertainment viewers is the highest, therefore, entertainment programmes are viewed the most.

2. Percentage of viewers watching the news

= 15% r

Percentage of viewers watching X informative = 10%

∴ Some of the percentages of viewers watching news and informative

= (15 + 10)% = 25%

= Percentage of viewers watching spo ts

Hence, news and information programmes have many viewers equal m those watching sports channels.

Drawing Pie Charts

The total angle at the centre of a circle is 360°. The central angles of sectors are fractions of 360°.

The central angle of a sector

= \(\left(\frac{\text { Value of the component represented by the sector }}{\text { Sum of the values of all the components }} \times 360^{\circ}\right)\)

Question 1. Draw a pie chart of the data given below. The time spent, by a child during a day.

  1. Sleep – 8 hours
  2. School – 6 hours
  3. Homework – 4 hours
  4. Play – 4 hours
  5. Others – 2 hours

Solution:

NCERT Solutions For Class 8 Maths Chapter 4 Data Handling The Time Spent By A Child During A Day

Now, we make the pie chart pie chart

NCERT Solutions For Class 8 Maths Chapter 4 Data Handling The Time Spent By A Child During A Day Pie Chart

NCERT Solutions For Class 8 Maths Chapter 4 Data Handling  Exercise 4.1

Question 1. A survey was made to find the type of music Il\al a certain group of young people liked in a city. The adjoining pie chart shows the findings of this survey

NCERT Solutions For Class 8 Maths Chapter 4 Data Handling A Survey Was Made To Find The Type Of Music

  1. From this pie chart answer the following
  2. If 20 people liked classical music, how many young people were surveyed?
  3. Which type of music is liked by the maximum number of people?
  4. If a cassette company were to make 1000 CD’s, how many of each type would they make?

Solution:

1. Suppose that x young people were surveyed. Then, the number of young people who liked classical music = 10% of x

= \(\frac{10}{100} \times x=\frac{x}{10}\)

According to the question

⇒ \(\frac{x}{10}=20\)

x = 20 × 10

Multiplying both sides by 10

X = 200

Hence 200 young people were surveyed

2. Since the percentage of young people who liked light music is the highest, therefore, light music is liked by the maximum number of people

3. Total number of CD’s = 1000

Number of CD’s of semi-classical music

= 20% of 1000

= \(\frac{20}{100} \times 1000\)

= 200

Number of CDs of classical music 10% of 1000

= \(\frac{10}{100} \times 1000\)

=100

Number of CD’s of folk music

= 30% of 1000

= \(\frac{30}{100} \times 1000\)

= 300

Number of CDs of light music

= 40% of 1000

= \(\frac{40}{100} \times 1000\)

= 400

Question 2. A group of 360 people were asked to vote for their favourite season from the h “ee Semi seasons rainy, winter and summer.

NCERT Solutions For Class 8 Maths Chapter 4 Data Handling Favourite Season

  1. Which season got the most votes?
  2. Find the central angle ofeach sector.
  3. Draw a pie chart to show this information.

Solution:

1. Since the no. of votes corresponding to the winter season is the maximum, therefore, winter season got the most votes.

2. Total votes = 90+ 120+ 150 = 360.

The central angle of the sector corresponds to the summer season

= \(\frac{\begin{array}{c}\text { Number of people who vote for summer season } \\\end{array}}{\text { Total number of people }} \times 360^{\circ} \)

= \(\frac{90}{360} \times 360^{\circ}=90^{\circ}\)

The central angle of the sector corresponds to the rainy season

= \(\frac{\begin{array}{c}\text { Number of people who vote for rainy season } \\\end{array}}{\text { Total number of people }} \times 360^{\circ}\)

= \(\frac{120}{360} \times 360^{\circ}=120^{\circ}\)

The central angle of the sector corresponding to the winter season

= \(\frac{\begin{array}{c}\text { Number of people who vote for winter season } \\\end{array}}{\text { Total number of people }} \times 360^{\circ}\)

= \(\frac{150}{360} \times 360^{\circ}=150^{\circ}\)

NCERT Solutions For Class 8 Maths Chapter 4 Data Handling Favourite Season Voted

Pie Chart

NCERT Solutions For Class 8 Maths Chapter 4 Data Handling Favourite Season Pie Chart

Question 3. Draw a pie chart showing the following information. The table shows the colours Preferred  by a group of people

NCERT Solutions For Class 8 Maths Chapter 4 Data Handling Colours And Number Of Peoples

Find the proportion of each sector. For example,

Blue is – \(\frac{18}{36}=\frac{1}{2}\)

Green – \(\frac{9}{36}=\frac{1}{4}\)  and so on

Use this to find the corresponding and

Solution:

NCERT Solutions For Class 8 Maths Chapter 4 Data Handling Information Of The Colours

Pie chart:

NCERT Solutions For Class 8 Maths Chapter 4 Data Handling Information Of The Colours Pie Chart

Question 4. The adjoining pie chart gives the marks scored in an examination by students in Hindi, English, Mathematics, SocialScienceand Science. If the total marks obtained by the students were 540, answer the following questions:

NCERT Solutions For Class 8 Maths Chapter 4 Data Handling Marks Scored Of The Information

1. In which subject did the student score 105 marks?

Hint: For 540 marks, the central angle

= 360°. So, for 105 marks, what is the central angle?

2. How many more malts were obtained by the student in Mathematics than in Hindi?

3. Examine whether the sum of the marks obtained in Social Science and Mathematics is more than that in Science and Hindi

Solution:

1. Total marks = 540

∴ Central angle corresponding to 540 marks

= 360°

∴  Central angle corresponding to 105 marks

= \(\frac{360^{\circ}}{540} \times 105=70^{\circ}\)

Since the sector having a central angle of 70° corresponds to Hindi, therefore, the student scored 105 marks in Hindi.

2. Central angle corresponding to the sector of mathematics = 90°

∴ Marks obtained by the student in Mathematics

= \(\frac{90^{\circ}}{360^{\circ}} \times 540\)

= 135

Marks obtained by the student in Hind =105

Hence, the student obtained 135 – 105 = 30 marks more in Mathematics than in Hindi.

3. Sum ofthe Central Angles for Social

Science and Mathematics

= 65° + 90° = 155°

Some of the central angles for Science and Hindi

= 80° + 70° = 150°

Since the sum of the central angles for Social Science and Mathematics is greater than the sum of the central angles for Science and Hindi, therefore the sum of the marks obtained in social science and mathematics is more than that in Science and Hindi.

Question 5.The number of students in a hostel, speaking different languages is given below. Display the data in a pie chart

NCERT Solutions For Class 8 Maths Chapter 4 Data Handling Languages

Solution:

NCERT Solutions For Class 8 Maths Chapter 4 Data Handling The Number Of Students In A hostel Speaking

Now, we make the pie chart.

NCERT Solutions For Class 8 Maths Chapter 4 Data Handling Languages Pie Chart

Chance And Probability

We face many situations where we take a chance and it does not go as per our wish. For example, on tossing a coin, we may get a head or we may get a tail; but simply by chance. There exist many situations when the chances of a certain thing happening or not happening are not equal.

Getting A Result

A random experiment is one whose outcome cannot be predicted exactly in advance.

Question 1. If you try to start a scooter, what arc the possible outcomes?
Solution:

If we try to start, a scooter, then the possible outcomes are

  1. The scooter may start ;
  2. The scooter may not start.

Question 2. When a die is thrown, what are the six possible outcomes?
Solution:

When a die is thrown, the six possible outcomes are 1, 2, 3, 4, 5 and 6.

Question 3. When you spin the wheel shown, what are the possible outcomes?  List them. (Outcome here means the sector at which the pointer stops).

NCERT Solutions For Class 8 Maths Chapter 4 Data Handling Spin Wheel And Bag With 5 Identical Balls

Solution:

Since in the given wheel, there are three sectors A, B, and C, so, when we spin the wheel, the possible outcomes are A, B and C.

Question 4. You have a bag with five identical balls of different colours and you are to pull out (draw) a ball without looking at it; list the outcomes you would
Solution:

Since the given bag contains five identical balls of different colours, W, R, B, G and Y, therefore, when we pull out (draw) a ball without looking at it, the possible outcomes are W, R, B, G and Y.

Question 5. In throwing a die

1. Does the first player have a greater chance of getting a six?
Solution: No

2. Would the player who played after him have a lesser chance of getting a six?
Solution: No

3. Suppose the second player got a six. Does it mean that the third player would not have a chance of getting a six?
Solution: No

Equally Likely Outcomes

Outcomes of an experiment are equally likely if each has the same chance of occurring.

Linking Chances To Probability

Chances of the happening of an event lead us to find the probability of that event. Of course, we have to take into account the total number of chances related to a particular event under consideration

Coin And Die

Question 1. What is the probability of getting a tail?
Solution: The probability of getting a tail is \(\frac{1}{2}\)

Question 2. What is the probability of getting Ike number 5?
Solution: The probability of getting the number 5 is \(\frac{1}{6}\)

Question 3. What is the probability of getting the number 7?
Solution: The probability of getting the number 7 is 0.

Question 4. What is the probability of getting a number 1 through 6?
Solution: The probability of getting a number 1 through 6 is \(\frac{1}{6}\)

Probability of an event =\(\frac{\text { Number of outcomes that make an event }}{\text { Total number of outcomes of the experiment }} \text {, }\) when the outcomes are equally likely.

Outcomes As Events

One or more outcomes of an experiment make an event

Question 1. List the number of outcomes of getting a green sector and not getting a green sector on this wheel

NCERT Solutions For Class 8 Maths Chapter 4 Data Handling Green Sector On This Wheel

  1. Find the probability of getting a green sector.
  2. Find the probability of not getting a green sector.

Solution:

1. Number of outcomes getting a green sector on this wheel = 5

The number of outcomes of not getting a green sector on this wheel = 3

2. Total number of equally likely outcomes = 8

Number of outcomes getting a green sector = 5

∴ Probability of getting a green sector

= \(=\frac{\begin{array}{l}
\text { Number of outcomes of getting a green sector } \\
\end{array}}{\text { Total number of outcomes }}\)

= \(\frac{5}{8}\)

Total number of equally likely outcomes = 8

Number of outcomes of not getting a green sector = 3

The probability of not getting a green sector

= \(\frac{\begin{array}{l}\text { Number of outcomes of getting a green sector }\end{array}}{\text { Total number of outcomes }}\)

= \(\frac{3}{8}\)

Chance And Probability Related To Real Life

Chances and probability are related to real life. The UHO of probability i.s ma(te in various cases in real life. For example, during elections ‘an exit poll’ is taken which gives a rough idea of the chance of winning of each candidate and predictions regarding the poll are made based on it. Here, we have used a small part of the group to find the characteristics of a large group.

NCERT Solutions For Class 8 Maths Chapter 4 Data Handling Exercise 4.2

Question 1. List the outcomes you can see in these experiments.

1. Spinning wheel

NCERT Solutions For Class 8 Maths Chapter 4 Data Handling Spinnig A Wheel

2. Tossing two coins together

Solution:

  1. The outcomes we can see in spinning the given wheel are A, B, C and D.
  2. The outcomes we can see in tossing two coins together are HT, HH, TH, and TT (HereIT means Head on first coin and Tail on the end coin and so on).

Question 2. When a die is thrown, list the outcomes of an event of getting

1.

  1. A prime number
  2. Not a prime number.

2.

  1. A number greater than 5
  2. A number not greater than 5

Solution:

Possible outcomes are :

1, 2, 3, 4, 5, and 6.

Out of these, prime numbers are 2, 3 and 5.

The outcomes of an event of getting a prime number are :

2, 3 and 5

The outcomes of an event of not getting a prime number are 1, 4 and 6.

The outcome of an event of getting a number greater than 5 is 6.

Outcomes of an event of getting a number not greater than 5 are1, 2, 3, 4 and 5

Question 3. Find the

  1. The probability of the pointer stopping on D in
  2. Probability of getting an AE from a well-shuffled deck of 52 playing cards?
  3.  Probability of getting a red apple.

NCERT Solutions For Class 8 Maths Chapter 4 Data Handling Probability Of Getting A Red Apple

Solution:

1. There are 5 sectors on the spinning wheel represented by A, A, B, C and D. The pointer stopping on D has m only 1 outcome, i.e., D

∴ Probability of the pointer

Stopping on D =\(\frac{1}{5}\)

2. Total number of playing cards = 52

Number of possible outcomes = 52

Number of aces in a deck of playing cards = 4

∴Probability y of getting an ace from a well-shuffled deck of 52 playing cards =\(\frac{4}{52}\)

= \(\frac{1}{3}\)

3. Total number of apples = 7

Number of red apples = 4

The probability of getting a red apple

= \(\frac{4}{7}\)

Question 4. Numbers 1 to 10 are written on ten separate slips (one number on one slip), kept in a box and mixed well One slip is chosen from the box without looking into it. What is the probability of this?

  1. Getting a number 6?
  2. Getting a number less than 6?
  3. Getting a number greater than 6?
  4. Getting a 1-digit number?

Solution:

Total number of outcomes of the event (1, 2. 3, 4, 5, 6, 7, 8, 9 and 10) = 10

1. Number of outcomes of getting a number 6 = 1

∴ The probability of getting the number 6

= \(\frac{1}{10}\)

2. There are 5 numbers (1, 2, 3, 4 and 5) less than 6.

∴  Number of outcomes getting a number less than 6 = 5

∴  Probability of getting a number Less than

6 = \(\frac{5}{10}\)

= \(\frac{1}{2}\)

3. There are ‘I number (7, 8, 9nnd 10) greater than

Number of outcomes of ting a number greater than G = 4

The probability of getting a number greater than 6 = 4 is greater than

6 = \(\frac{4}{10}\)

= \(\frac{2}{5}\)

There are 9 1-digit numbers(1, 2, 3, 4, 5, 6, 7, 8 and 9)

∴  Number of outcomes of getting a1-digit number = 9

∴  Probability of getting a 1-digit number = \(\frac{9}{10}\)

Question 5. If you have a spinning wheel with 3 green sectors, 1 blue sector and 1 red sector, what is the probability of getting a green sector? What is the probability of getting a non-blue sector?
Solution:

Number of green sectors = 3

Number of blue sectors = 1

Number of red sectors = 1

∴  Total number of sectors

= 3+1+1 = 5

∴ Total number of outcomes of the event = 5

Number of outcomes of getting green sector = 3

Probability of getting a green sector

= \(\frac{3}{5}\)

Number of outcomes of getting non-blue sector

= Number of green sectors + Number of red sectors

= 3+1 = 4

∴ The probability of getting a non-blue

= \(\frac{4}{5}\)

Question 6 Find the probabilities ofthe (rents I have given in
Solution:

Total number of outcomes of the event (1. 2. 3, 4, 5 and 6) = 6

1.

1. Number of prime numbers

(2. 3 and 5) = 3

Number of outcomes of getting a prime number = 3

∴ Probability of getting a prime number

= \(\frac{3}{6}\)

= \(\frac{1}{2}\)

2. Number of non-prime numbers (1, 4 and 6) = 3

Number of outcomes of getting a prime number = 3

∴ Probability of gelling a non-prime

= \(\frac{3}{6}\)

= \(\frac{1}{2}\)

2.

1. Number greater than 5 = 6, i.e., only one.

∴ The probability of getting a number greater than 5 =\(\frac{1}{6}\)

2. Number of numbers not greater than 5 (1, 2, 3, 4 and 5) = 5

The number of outcomes of getting a number not greater than 5 = 5

Probability of getting a number not greater than 5 = \(\frac{5}{6}\)

NCERT Solutions For Class 8 Maths Chapter 4 Data Handling Multiple Choice Questions

Observe the following bar graph and answer the following questions :

NCERT Solutions For Class 8 Maths Chapter 4 Data Handling Expenditure In Thousands Rupees

Question 1. On which item has the maximum expenditure been done?

  1. Conveyance
  2. Rent
  3. Fee
  4. Servant’s salary.

Solution:  1. Conveyance

The height of the bar corresponding to conveyance is the maximum.

Question 2. On which item has the minimum expenditure been done?

  1. Servant’s salary
  2. Food
  3. Rent
  4. Conveyance.

Solution: 1. Servant’s salary

The height of the bar corresponding to the servant’s salary is the minimum.

Question 3. What is the expenditure on food?

  1. ₹ 1000
  2. ₹ 2000
  3.  ₹3000
  4.  ₹5000

Solution: 4. ₹5000

Expenditure done on food = 5 × 1000 = 5000.

Question 4. What is the difference between expenditures done on conveyance and rent?

  1.  ₹ 1000
  2.  ₹ 2000
  3.  ₹ 3000
  4.  ₹ 4000

Solution: 2. ₹ 2000

Expenditure done on conveyance

= 6 × 1000 = ₹ 6000

Expenditure done on rent = 4 × 1000

= ₹ 4000

.-. Difference = ₹ 6000 – ₹ 4000

= ₹ 2000

Question 5. ₹ 5000 is the expenditure done on

  1. Rent
  2. Food
  3. Fee
  4. Recreation

Solution: 2. Food

⇒ \(\frac{5000}{1000}\) = 5 cm is the height of the bar corresponding to food

Question 6. ₹60000 is the expenditure done on

  1. Fee
  2. Rent
  3. Conveyance
  4. Food

Solution:  3. Conveyance

⇒ \(\frac{6000}{1000}\) = 6 cm is the height of the bar corresponding to conveyance

Question 7. How much expenditure h;m been done in all?

  1. 21000
  2. 18000
  3. 15000
  4. 20000

Solution: 1. 21000

Total expenditure

= (1 + 6 + 4+ 5 + 2 + 3) × 1000

= ₹ 21000

Observe the following paragraph and answer the following questions:

NCERT Solutions For Class 8 Maths Chapter 4 Data Handling Number Of Books

Question 8. Of which subject are there the maximum books?

  1. Hindi
  2. English
  3. Maths
  4. Science

Solution: 1. Hindi

The height of the bar corresponding to Hindi is maximum.

Question 9. How many books are there on ofthe subject whose books are maximum?

  1. 100
  2. 200
  3. 300
  4. 400

Solution: 4.  400

Hindi  → 400

Question 10. Of which subject are there the minimum books?

  1. Social
  2. Science
  3. Hindi
  4. English

Solution: 1. Social

The height of the bar corresponding to Social Science is the minimum

Question 11. How many books are there on the subject whose books are minimum?

  1. 100
  2. 200
  3. 30
  4. 400

Solution: 1. 100

Social Science →  100

Question 12. Which two subjects have the same number of books?

  1. Maths and Hindi
  2. Hindi and English
  3. English and Science
  4. Science and Social Science

Solution: 3. English and Science

English → 200

Science → 200

Question 13. 300 books on the subject

  1. Maths
  2. English
  3. Hindi
  4. Science

Solution:  1. Maths

300 → Maths

Question 14. The difference of the number of books in English and Science is

  1. 200
  2. 100
  3. 400
  4. 0

Solution: 4. 0

200-200 = 0

Question 15. The difference of the number of books of Hindi and Social Science is

  1. 200
  2. 300
  3. 400
  4. 100

Solution: 2. 300

400-100 = 300

Question 16. The total number of books is

  1. 1200
  2. 1400
  3. 1600
  4. 1800

Solution: 1. 1200

300+400 + 200 + 200+ 100 = 1200

Question 17. The total number of books of English and Science is

  1. 200
  2. 100
  3. 400
  4. 0

Solution: 3. 400

200 + 200 = 400

Observe the pie chart given below and answer the following questions:

NCERT Solutions For Class 8 Maths Chapter 4 Data Handling Central Angle Of Sector

Question 18. The central angle for sector A is

  1. 108°
  2. 144°
  3. 72°
  4. 150°

Solution: 1. 108°

Central angle for sector A

= \(\frac{30}{100} \times 360^{\circ}\)

= 108°

Question 19. The central angle for sector B is

  1. 108°
  2. 144°
  3. 72°
  4. 150°

Solution: 2. 144°

Central angle for sector A

= \(\frac{40}{100} \times 360^{\circ}\)

= 144°

Question 20. Which sector has the greatest angle?

  1. A
  2. B
  3. C
  4. None of these

Solution: 2. B

Greatest percentage = 40% → B

Question 21. What is the difference between the central angles for Sector B and Sector C?

  1. 36°
  2. 72
  3. 81°

Solution: 1. 36°

144° – 108° = 36°

Observe the pie chart and answer the English following questions

NCERT Solutions For Class 8 Maths Chapter 4 Data Handling Types Of Colours

Question 22. Which two colours have the same central angles?

  1. Red, yellow
  2. Red, green
  3. Yellow, green
  4. Blue, red.

Solution:  1. Red, yellow

Red →  45% ; Yellow → 45%.

Question 23. Which colour has the greatest central angle?

  1. Red
  2. Yellow
  3. Green
  4. Blue

Solution: 4. Blue

Blue →  180°

Question 24. The proportion of the sector in red is

  1. \(\frac{1}{2}\)
  2. \(\frac{1}{4}\)
  3. \(\frac{1}{8}\)
  4. \(\frac{1}{3}\)

Solution: 3. \(\frac{1}{8}\)

⇒ \(\frac{45^{\circ}}{360^{\circ}}=\frac{1}{8}\)

Question 25. The difference between the central angles for green and blue is

  1. 45°
  2. 90°
  3. 180°
  4. 22 \(\frac{1}{2}\)

Solution: 2. 90°

Central angle for blue = 180°

Central angle for green = 90°

Difference = 180° – 90° = 90°.

Question 26. A child has a block in the shape of a cube with one letter written on each face as shown below

NCERT Solutions For Class 8 Maths Chapter 4 Data Handling A Child Has A Block In The Shape

The cube is thrown once. What is the probability of getting A

  1. \(\frac{1}{3}\)
  2. \(\frac{1}{6}\)
  3. \(\frac{1}{2}\)
  4. \(\frac{1}{4}\)

Solution:  1. \(\frac{1}{3}\)

Probability =\(\frac{2}{6}\)

= \(\frac{1}{3}\)

Question 27. A die is thrown. What is the probability of getting an even prime number

  1. \(\frac{1}{6}\)
  2. \(\frac{1}{4}\)
  3. \(\frac{1}{3}\)
  4. \(\frac{1}{2}\)

Solution: 1. \(\frac{1}{6}\)

Even prime number = 2

Probability =\(\frac{1}{6}\)

NCERT Solutions For Class 8 Maths Chapter 4 Data HandlingTrue Or False

1. In the n chart, a whole circle is divided into various sectors – True

2. The number of times a particular observation that occurs in a given data is called its frequency – True

3. The probability of a sure event is 0– False

4. The probability of an impossible event is 1 – False

5. In a throw of a die, the outcomes are equally likely – True

NCERT Solutions For Class 8 Maths Chapter 4 Data Handling Fill In The Blanks

1. The difference between the highest and the lowest values ofthe observations in a data is called the Range of the data.

2. A geometric representation showing the relationship between a whole and its parts is called a → Pie chart

3. The probability that it will rain tomorrow is 0. 75. What is the probability that it will not rain tomorrow  → 0.25

4. Find the range ofthe marks obtained by 10 students in class as follows   →  11, 9, 13, 18, 20, 18, 42, 41, 13,

5. In the throw of die, what is the probability of getting a number greater than 6  →  0

6. What is the total number of outcomes, when a coin is tossed →  2